abclinuxu.cz AbcLinuxu.cz itbiz.cz ITBiz.cz HDmag.cz HDmag.cz abcprace.cz AbcPráce.cz
Inzerujte na AbcPráce.cz od 950 Kč
Rozšířené hledání
×
    dnes 04:33 | IT novinky

    Společnost Espressif (ESP8266, ESP32, …) získala většinový podíl ve společnosti M5Stack, čímž posiluje ekosystém AIoT.

    Ladislav Hagara | Komentářů: 0
    včera 23:44 | Nová verze

    Byla vydána nová stabilní verze 3.5 svobodného multiplatformního softwaru pro editování a nahrávání zvukových souborů Audacity (Wikipedie). Přehled novinek také na YouTube. Nově lze využívat cloud (audio.com). Ke stažení je oficiální AppImage. Zatím starší verze Audacity lze instalovat také z Flathubu a Snapcraftu.

    Ladislav Hagara | Komentářů: 0
    včera 16:44 | Zajímavý článek

    50 let operačního systému CP/M, článek na webu Computer History Museum věnovaný operačnímu systému CP/M. Gary Kildall z Digital Research jej vytvořil v roce 1974.

    Ladislav Hagara | Komentářů: 0
    včera 16:22 | Pozvánky

    Byl zveřejněn program a spuštěna registrace na letošní konferenci Prague PostgreSQL Developer Day, která se koná 4. a 5. června. Na programu jsou 4 workshopy a 8 přednášek na různá témata o PostgreSQL, od konfigurace a zálohování po využití pro AI a vector search. Stejně jako v předchozích letech se konference koná v prostorách FIT ČVUT v Praze.

    TomasVondra | Komentářů: 0
    včera 03:00 | IT novinky

    Po 48 letech Zilog končí s výrobou 8bitového mikroprocesoru Zilog Z80 (Z84C00 Z80). Mikroprocesor byl uveden na trh v červenci 1976. Poslední objednávky jsou přijímány do 14. června [pdf].

    Ladislav Hagara | Komentářů: 6
    včera 02:00 | IT novinky

    Ještě letos vyjde Kingdom Come: Deliverance II (YouTube), pokračování počítačové hry Kingdom Come: Deliverance (Wikipedie, ProtonDB Gold).

    Ladislav Hagara | Komentářů: 3
    21.4. 19:11 | Komunita

    Thunderbird 128, příští major verze naplánovaná na červenec, přijde s nativní podporou Exchange napsanou v Rustu.

    Ladislav Hagara | Komentářů: 21
    21.4. 04:44 | Komunita

    Byly vyhlášeny výsledky letošní volby vedoucího projektu Debian (DPL, Wikipedie). Novým vedoucím je Andreas Tille.

    Ladislav Hagara | Komentářů: 7
    21.4. 00:11 | Nová verze

    Po osmi měsících vývoje byla vydána nová verze 0.12.0 programovacího jazyka Zig (GitHub, Wikipedie). Přispělo 268 vývojářů. Přehled novinek v poznámkách k vydání.

    Ladislav Hagara | Komentářů: 2
    20.4. 23:55 | Pozvánky

    Poslední měsíc byl plný zajímavých akcí, o kterých Vám bastlíři z projektu MacGyver mohou povědět, protože se na ně sami vydali. Kde všude byli, ptáte se? Objevili se na Installfestu, Arduino Day, Hackaday Europe a tajném srazu bastlířů z Twitteru. A z každé akce pro vás mají zajímavé poznatky.

    … více »
    bkralik | Komentářů: 1
    KDE Plasma 6
     (71%)
     (10%)
     (2%)
     (17%)
    Celkem 670 hlasů
     Komentářů: 4, poslední 6.4. 15:51
    Rozcestník

    Dotaz: Zašifrování HTML stránky, aby nebyl vidět zdroják

    5.1.2021 18:55 Prďoch
    Zašifrování HTML stránky, aby nebyl vidět zdroják
    Přečteno: 9914×
    Dobrý den,

    pomocí javascriptu jsem sice zakázal funkce pro kopírování a použití F-12, nicméně bych potřeboval nasměrovat jak celý HTML kód skrýt třeba pomocí base64 nebo AES 256, většina online encryptorů generuje zároveň do zacryptovaného HTML také trojany atp.

    Má někdo s tímto zkušenost? Děkuji moc, přeji poklidný Nový Rok 21'

    Řešení dotazu:


    Odpovědi

    Řešení 2× (Filip Jirsák, trilobyte)
    5.1.2021 19:06 Petr Šobáň | skóre: 80 | blog: soban | Olomouc
    Rozbalit Rozbalit vše Re: Zašifrování HTML stránky, aby nebyl vidět zdroják
    Blbost nejde to.

    Aby prohlížeč něco zobrazil potřebuje dodat data, samozřejmě je můžeš generovat jak chceš, ale prohlížeči je nějak pošleš a to lze zobrazit.

    Samozřejmě lze získání zdrojového kodu stížit, ale vždy to co server pošle lze zachytit.
    5.1.2021 19:19 Prďoch
    Rozbalit Rozbalit vše Re: Zašifrování HTML stránky, aby nebyl vidět zdroják
    To vím,

    tak jak je možné že jsem kdysi viděl stránku co neměla vidět žádný zdroják po zadání ctru -u, nebo v webkonzole nebylo nic, jen čistá bílá stránka a přece měl web obsah, obrázky..

    Existují metody jak upravit HTML do base64 atd, nemohu se s tím smířit, že to nejde. Kdysi jsem nato narazil u nějakého Inda co se tím chlubil na FB.Kdybych to neviděl tak o tom nepíši :-/
    5.1.2021 19:38 Prďoch
    Rozbalit Rozbalit vše Re: Zašifrování HTML stránky, aby nebyl vidět zdroják
    V tom případě musel nějak vše převést do bílé barvy, aby si toho člověk nevšiml.Nebylo tam nic vidět,bílo prostě.
    5.1.2021 20:00 Mti. | skóre: 31 | blog: Mti
    Rozbalit Rozbalit vše Re: Zašifrování HTML stránky, aby nebyl vidět zdroják
    scrollbar... scrollbar tam nebyl? Ze by to bylo proste jen "za rohem" posunute taby/mezerami? :-)

    Jinak ctrl-u v prohlizeci zobrazi zdrojak - neprovadi se to, takze ten je zobrazovan naopak celkem citelne a stranka by nemela ovlivnit, jak to bude barevne... :-)

    Co mi bude branit misto otevreni ve firefoxu to proste stahnout wgetem? Ten to opravdu neprovede a poslusne vse ulozi. Cim si to pak prohlidnu je moje vec. A prohlidnu.

    Takze bych se opravdu zamyslel, jestli zrovna toto "musi" zustat utajene. :-/ >kus se zamyslet, co tam bude tak tajneho, ze se to "nesmi" zobrazit. Jestli by spis nemel byt omezen pristup na celou stranku jen lidem, kteri tam maji co delat.
    Vidim harddisk mrzuty, jehoz hlava plotny se dotyka...
    5.1.2021 20:12 Prďoch
    Rozbalit Rozbalit vše Re: Zašifrování HTML stránky, aby nebyl vidět zdroják
    wgetwem ti zabrání restrikce na straně serveru, v .htaccesu atd.To je pravda to zobrazení ty barvy nejsou ovlivnit,asi by se musel upravit firefox pokud by to člověk analyzoval skrze něj.Nezkoumal jsem to :-) Šlo mi o to zda to vůbec jde, vzpomněl jsem si na toho Indocha, a tak jsem kutil a kutil a nevykutil :-)
    5.1.2021 20:16 Prďoch
    Rozbalit Rozbalit vše Re: Zašifrování HTML stránky, aby nebyl vidět zdroják
    Konkrétně ten wget :

    SetEnvIfNoCase User-Agent “^Wget” bad_bot A GET POST> Order Allow,Deny Allow from all Deny from env=bad_bot /A>

    místo < A napsat : Limit

    a šmitec :-)
    5.1.2021 20:24 Ignotus | skóre: 10
    Rozbalit Rozbalit vše Re: Zašifrování HTML stránky, aby nebyl vidět zdroják
    Tak to ti obídem pomocou

    wget -U "whatever" http://your.super.secret.webpage/

    :)
    6.1.2021 14:15 Prďoch
    Rozbalit Rozbalit vše Re: Zašifrování HTML stránky, aby nebyl vidět zdroják
    RewriteCond %{HTTP_USER_AGENT} wget.* [NC] RewriteRule .* - [F,L]
    6.1.2021 15:05 jiwopene | skóre: 31 | blog: Od každého trochu…
    Rozbalit Rozbalit vše Re: Zašifrování HTML stránky, aby nebyl vidět zdroják
    Na wget to za jistých okolností nefunguje.

    Nastavte nějaký server tak, aby stránka byla běžnému GUI prohlížeči doručena (na úrovni HTTP) a já, stejně jako mnozí další na tomto webu, Vám mohu s jistotou ukázat způsob, kterým ji získám wgetem nebo curlem. Toto řešení je poněkud absurdní – doporučuji přečíst RFC 7230 a související.
    .sig virus 3.2_cz: Prosím, okopírujte tento text do vaší patičky.
    Řešení 1× (Filip Jirsák)
    Jendа avatar 5.1.2021 20:47 Jendа | skóre: 78 | blog: Jenda | JO70FB
    Rozbalit Rozbalit vše Re: Zašifrování HTML stránky, aby nebyl vidět zdroják
    Ctrl+U myslím zobrazuje originální zdroják, ne generovaný zdroják, takže je možné, že tam bude jenom krátký javascriptový loader a zbytek stránky se vygeneruje jím. Ale aby tam „nic nebylo“ to je buď nesmysl, nebo bug v prohlížeči. Nejlepší na tyhle věci je odposlechnout si to nějakým Burp proxy. Téměř určitě ses prostě špatně díval.

    Developer console se dá nějak detekovat a ta detekce se dá zase obejít.
    Existují metody jak upravit HTML do base64 atd
    To samozřejmě jde. Ale co mi brání si do base64 dekódovat, nebo si přečíst to vygenerované HTML? Každopádně se kód dá tak zprasit, že ho nikdo číst nebude.
    5.1.2021 20:53 bigBRAMBOR | skóre: 37
    Rozbalit Rozbalit vše Re: Zašifrování HTML stránky, aby nebyl vidět zdroják
    Takze mas predstavu ze stranku zasifrujes aes256, prohlizec to zobrazi ale kod stranky bude sifrovanej takze nepujde precist? Asi jsi toho Inda nepochopil. Delali se napr prekladace PHP tak abys mohl kod distribuovat na cizi servery ale kod nebyl k precteni, ale to je jina situace.

    Co to je online encryptor ktery do kodu dava trojany?
    6.1.2021 14:06 Prďoch
    Rozbalit Rozbalit vše Re: Zašifrování HTML stránky, aby nebyl vidět zdroják
    Podívej kdybych měl odkaz a viděli jste co já tak by jste si sedli na zadek, doména abc.neco dal jsem CTRL +U a měl jsem bílou stránku tady řeším jak to, jak je to možné, a to bylo i Chrome, chvástal se že jeho stránka je šifrovaná a není možné zjistit jeho zdroják,tak asi.

    ASE jsem zkoušel, pro distribuci obsahu OK ale zašifrovat zdroják -blbost, to vím. " Co to je online encryptor ktery do kodu dava trojany? " máš google ne.
    6.1.2021 14:43 MadCatX | skóre: 28 | blog: dev_urandom
    Rozbalit Rozbalit vše Re: Zašifrování HTML stránky, aby nebyl vidět zdroják
    To bylo proto, ze CTRL+U ve Firefoxu ukaze zdrojak tak, jak jej dostal od serveru. Pokud je ten web generovany Javascriptem na klientu, bude takovy kod vypadat dost stroze a obsahovat jen odkaz na ten skript, ktery tu stranku vykresli. Resi se to tak, ze zmacknes CTRL+SHIFT+C a podivas se na vygenerovane HTML a pripadne HTTP requesty.

    Ind, ktery ti neco podobneho tvrdil se z tebe pravdepodobne jen snazil vymamit prachy za ten jeho "neprustrelny" sifrovator.
    Jendа avatar 5.1.2021 20:43 Jendа | skóre: 78 | blog: Jenda | JO70FB
    Rozbalit Rozbalit vše Re: Zašifrování HTML stránky, aby nebyl vidět zdroják
    By mě zajímalo co tam je tak převratného. Spíš jsem viděl experty, kteří se rozhodli řešit řízení přístupu na straně uživatele, a pak samozřejmě řešili, že si to lidi můžou přečíst ve zdrojáku.
    6.1.2021 14:07 Prďoch
    Rozbalit Rozbalit vše Re: Zašifrování HTML stránky, aby nebyl vidět zdroják
    Díky to je mi jasné s tím nemám problém, jen nechápu jak je to možné, že jsem neviděl zdroják ani žádné znaky a přesto stránka fungovala..
    6.1.2021 14:46
    Rozbalit Rozbalit vše Re: Zašifrování HTML stránky, aby nebyl vidět zdroják
    Jestli ty znaky nebyly vpravo dole.
    6.1.2021 16:09 chichi
    Rozbalit Rozbalit vše Re: Zašifrování HTML stránky, aby nebyl vidět zdroják
    Protoze to bylo PHP. Hele nechces radsi na pristi rocnik v C? https://www.ioccc.org/years.html
    Řešení 1× (trilobyte)
    5.1.2021 22:08 Kit | skóre: 45 | Brno
    Rozbalit Rozbalit vše Re: Zašifrování HTML stránky, aby nebyl vidět zdroják
    Nejjistějším řešením je ponechat bílou stránku bez jakéhokoliv obsahu.
    Komentáře označují místa, kde programátor udělal chybu nebo něco nedodělal.
    6.1.2021 14:08 Prďoch
    Rozbalit Rozbalit vše Re: Zašifrování HTML stránky, aby nebyl vidět zdroják
    Tak samozřejmě :-D že. :-)
    6.1.2021 12:20 LarryL | skóre: 27
    Rozbalit Rozbalit vše Re: Zašifrování HTML stránky, aby nebyl vidět zdroják
    Podle tvého popisu se zdá, že hledáš asi toto: vypnutí pravého myšítka, AES encryption (s heslem), vypnutí ctrl u F12, přidání spousty bílých mezer. Poslední odpověď je možná link na toho tvého Inda :-)
    6.1.2021 14:12 Prďoch
    Rozbalit Rozbalit vše Re: Zašifrování HTML stránky, aby nebyl vidět zdroják
    Díky LarryL tady jsem už byl předtím něž jsem psal tento post.Jenže já jsem fakt neviděl ani v FF ani v Chromu zdroják, říkám jak je to možné a lituju že jsem ztratil odkaz protože by jste si fakt jak vidím sedli na zadní/přední :-) Nic tam nebylo ani písmenko, fakt nevím nemám tucha. Vím že jsem to viděl a neměl jsem ani jedno pivo, nic, v jedné skupině na FB anglické se tím chvástal, každej mu tam psal excelentní komenty, já nevím z principu je to blbost, kašlete na to asi to nevypátrám a čas je drahý :-) Mějte super Nový Rok 21' a bez stresu pánové a dámy.
    6.1.2021 14:50 Bublal
    Rozbalit Rozbalit vše Re: Zašifrování HTML stránky, aby nebyl vidět zdroják
    To že TY jsi nic neviděl nic neznamená. A nesedli bysme si ani náhodou, jen bysme ti ukázali jak blbě koukáš. Vždyť jsi ani nepochopil, ten wget nebo BURP proxy. Takže vzdělávej se a nehraj si na člověka kterýmu ufouni dělali anální sondy, ale nemá k tomu důkazy.
    8.1.2021 14:23 Prďoch
    Rozbalit Rozbalit vše Re: Zašifrování HTML stránky, aby nebyl vidět zdroják
    Bublifuk, já ten zdroják neviděl, a neviděli to ani ostatní ze skupiny, tak jsem se nato podíval, jestli si myslíž že neznám burp suite, nebo whireshark kdy tady v česku byl byl na cedkách backtrack, to je tvoje věc, wget právě používám se spustou dalších nástrojů a právě proto že vím, tak se ptám a zkouším jestli buď víte a dokážete mě nasměrovat, je mi fakt celkem jedno kde a s kým by si seděl nebo neseděl, otázkou je aby si fakt pak neseděl.

    Konstatuji že jsi nějak " análově bukvičácky založený " čistě z toho co píšeš a mícháš jablka a hrušky,takže tito lidé patří už z jádra mého přesvědčení ke zdi nebo do vyhnanství.Čili z tvého postu je odpovídající hodnota rovnající znalostem či dokonce inteligenci.

    Kdyby si četl a netrolloval věděl bys.Přeji ti abychom se opravdu nepotkali v civilu nedopadlo by to dobře.
    18.1.2021 19:48 Bublal
    Rozbalit Rozbalit vše Re: Zašifrování HTML stránky, aby nebyl vidět zdroják
    Á pán je těžká váha. Jen jsem chtěl napsat, že si pěkný debil, ale tak nějak jako líp. No nepovedlo se. Tak znovu po lopatě, jsi debil jestli si myslíš, že prohlížeč ta data vykouzlí z prdele (zdá se, že jsi na ní zatížen), když mu nepřijdou po síti. Nejlíp bude, když se vrátíš za těma super profíkama na Facebúček.
    Gréta avatar 6.1.2021 14:39 Gréta | skóre: 36 | blog: Grétin blogísek | 🇮🇱==❤️ , 🇵🇸==💩 , 🇪🇺==☭
    Rozbalit Rozbalit vše Re: Zašifrování HTML stránky, aby nebyl vidět zdroják

    vývojář si drbe pleš

    adobe vypnulo mu flash!!!!

    má i teďko na flashi lpět??

    nee!!! appku přepsat do html5!!!!!!

    :D :D :D :D ;D ;D

     

    btw nejvíc nejbezpečnější by asi jako bylo všecko důležitý skovat na serveru a uživateloj ukazovat jenom vygenerovanou stránku jestli tam jakoby děláš něco děsně moc tajnýho :O :O :D ;D

    8.1.2021 14:30 Prďoch
    Rozbalit Rozbalit vše Re: Zašifrování HTML stránky, aby nebyl vidět zdroják
    Díky :-), ne samozřejmě, nic tajného já jsem přišel pro radu zda se s tím někdo nesetkal a zda to je či není možné :-) před dvěma lety jsem narazil v nejm.skupině na FB na ukázku webu kde se nedal, nebo jsem nějak záhadně a to pak i ostatní ze skupiny co to viděli - neviděl zdroják webu, prostě bílá stránka viz všechno hore,co jsem psal :-) s tím uživatelem a jemu pouze viditelnou stránkou to v praxi používáme pro zákazníky, asi vím co mi chceš naznačit, jen prostě nevím jak je to možné že tam bylo místo zdrojového kódu "bílo-prázdno" lidi v té skupině prováděli webdesign na " brnkačku " a přese se nemohlo tolik lidí nechat nějak nachytat, občas na to myslím. :-)

    8.1.2021 16:00 MadCatX | skóre: 28 | blog: dev_urandom
    Rozbalit Rozbalit vše Re: Zašifrování HTML stránky, aby nebyl vidět zdroják
    Neřekl bych, že zrovna na Facebooku se budou srocovat ti nejdovednější weboví vývojáři. Schválně jsem se mrkl, co za "funkční" schovávače kódu webu se dá najít. Našel jsem třeba tohle i s demo stránkou: https://www.htmlguard.com/sample.html. Používá to přesně ty techniky, které tu padly, tedy hafo mezer a generování dokumentu obfuskovaným JS. Nic z toho není složité překonat.
    16.1.2021 00:24 [Jooky]
    Rozbalit Rozbalit vše Re: Zašifrování HTML stránky, aby nebyl vidět zdroják
    Příloha:
    Co to ma vlastne robit ? (obrazok v prilohe :D ) ...


    ... este z 90tych rokov, ked sa web stranky bezne pisali v notepad-e, si pamatam tento "prazdny zdrojak". Strasne vela novych riadkov, strasne vela medzier a zhutneny kod (odstranene veci, co niesu potrebne na zobrazenie html). Na takej stranke ked si dal niekto zobrazit zdrojovy kod, tak sa zobrazilo prazdne okno. Pozornejsi si vsimli sktrolovatko a "hladali html". Sikovnejsi dali ctrl+f bud nejaku cast html (napr html, head, body, etc.), alebo text, co je na obrazovke ... osobne si myslim, ze ten "prazdny" zdrojak bude presne toto iste. So zbytkom suhlasim. Moze to byt akokolvek obfuskovane, kedze browser potrebuje ciste html na vykreslenie stranky, tak v browseri bude vysledne html a bude s dat precitat. Na tej HTML Guard stranke som stravil mozno o 2 sekundy dlhsie, nez som nasiel potrebne zobrazenie v menu, namiesto klasickeho mys + "inspect element".
    Gréta avatar 18.1.2021 21:17 Gréta | skóre: 36 | blog: Grétin blogísek | 🇮🇱==❤️ , 🇵🇸==💩 , 🇪🇺==☭
    Rozbalit Rozbalit vše Re: Zašifrování HTML stránky, aby nebyl vidět zdroják

    já hlavně jako nechápu proč by to jako někdo dělal :O :O každej kdo umí wget si to stejně stáhne celý i s těma vobrázkama skovanejma a vobyčejný lidi si akorát nebudou moct voznačit nějakej kousíček textu na vygooglení třeba :O :O

    18.1.2021 21:27 Kit | skóre: 45 | Brno
    Rozbalit Rozbalit vše Re: Zašifrování HTML stránky, aby nebyl vidět zdroják
    Proč? Protože to zákazník chce a trvá na tom.
    Komentáře označují místa, kde programátor udělal chybu nebo něco nedodělal.
    9.1.2021 20:03 Filip Jirsák | skóre: 68 | blog: Fa & Bi
    Rozbalit Rozbalit vše Re: Zašifrování HTML stránky, aby nebyl vidět zdroják
    Už vám to tady napsalo několik lidí, jak to bylo – že tam nebyl prázdný zdroják, ale jenom byl pomocí mezer nebo prázdných řádků odsunutý mimo vaše zorné pole. A možná byl zdroják co nejmenší a jen načítal nějaký skript, který zařídil samotné zobrazení stránky. Jestli jste ten zdroják studoval stejně pozorně, jako zdejší diskusi, není divu, že jste to přehlédl.
    17.1.2021 15:32 Soyo
    Rozbalit Rozbalit vše Re: Zašifrování HTML stránky, aby nebyl vidět zdroják
    Teraz by velmi pomohlo tu stranku znovu najst a ulozit lokalne (bez obrazkov - single html) a az ked vysledny subor bude mat 0B, tak ma zmysel badat.

    Je tu viacero nazorov, ze to nie je mozne - tak toto ich but vyvrati .. alebo nevyvrati. V kazdom pripade sa tym vyrazne zvysia sance na konstruktivnost dajsej debaty.
    17.1.2021 18:27 Filip Jirsák | skóre: 68 | blog: Fa & Bi
    Rozbalit Rozbalit vše Re: Zašifrování HTML stránky, aby nebyl vidět zdroják
    Jistě. A když někdo bude tvrdit, že měl v ruce obyčejný žulový kámen, který najednou začal padat nahoru, také po něm budeme chtít ten kámen jako důkaz. Sice existují názory, že to není možné, protože gravitačně se objekty vždy přitahují – ale ten kámen by ty názory mohl vyvrátit nebo nevyvrátit.

    Pořád někdo vymýšlí nějaké kompresní algoritmy, přitom je to tak jednoduché – vytvoříte stránku, schováte zdrojový kód, aby měla stránka 0 bajtů, a máte zkomprimováno na maximum.

    Konstruktivnosti debaty nejvíc prospěje, když se v ní nebudou vyskytovat bláboly jako že z 0 bajtů dokáže prohlížeč vyčarovat celou webovou stránku, kterou zobrazí.
    17.1.2021 20:43 bigBRAMBOR | skóre: 37
    Rozbalit Rozbalit vše Re: Zašifrování HTML stránky, aby nebyl vidět zdroják
    Ale nalezeni takoveho kamene by velmi zvysilo sance na konstruktivnost debaty, to bez pardonu ano.

    Me prijde zvlastni jak dlouho se debata toci kolem neceho co mi prijde jako elementarni. Pokud ma prohlizec neco zobrazit, musi neco dostat. To co dostane musi byt schopen zpracovat a precit. Pokud to je schopen precist prohlizec, je schopen to odchytnout a reprodukovat i clovek. Muze se mluvit o nejake obfuskaci, o schovani kodu v konzoly prohlizece za rohem, ale magie typu server posle neco zasifrovaneho sifrou kterou nikdo nezna a browser to zobrazi ale clovek ne, nebo dokonce server posle prazdno ale prohlizec to zobrazi, ta tady nefunguje.
    18.1.2021 08:17 Filip Jirsák | skóre: 68 | blog: Fa & Bi
    Rozbalit Rozbalit vše Re: Zašifrování HTML stránky, aby nebyl vidět zdroják
    Právě proto, že je to tak elementární, jsem to přirovnal ke gravitaci. Ani není potřeba vědět nijak podrobně, jak prohlížeč funguje. Stačí vědět akorát to, že jediné, co prohlížeč umí, je zobrazit HTML dokument, jeho vzhled načíst z CSS, v dokumentu zobrazit nějaká multimédia a umí s tím manipulovat pomocí JavaScriptu. Přesto se tu sáhodlouze debatuje, jestli se nemůže stát zázrak a prohlížeč nemůže zobrazit požadovanou stránku i jen tak, bez předpisu pro její zobrazení. A dokládá se to zcela vážně míněným argumentem „já jsem to někde viděl“. Ta neochota se nad situací zamyslet a místo toho náhodně hledat zázrak je až děsivá.
    18.1.2021 11:21 tttttttttt
    Rozbalit Rozbalit vše Re: Zašifrování HTML stránky, aby nebyl vidět zdroják
    Přidám se k těm "někde jsem to viděl", nepamatuju si pochopitelně skoro nic. Autor použil nějakou obskurní funkci konkrétního prohlížeče, pomocí které dokázal zdroják protlačit postranním kanálem, který se při view-source nezobrazil. Prohlížeč ten zdroják měl, ve vývojářských nástrojích se dal zobrazit. V praxi to bylo nepoužitelné, se šifrováním to nemělo nic společného.
    k3dAR avatar 18.1.2021 12:34 k3dAR | skóre: 62
    Rozbalit Rozbalit vše Re: Zašifrování HTML stránky, aby nebyl vidět zdroják
    jenze tazatel psal ze nekde videl prazdnej zdrojak kdy to co prohlizec zobrazil udajne nebylo ani v devel nastrojich videt, coz je proste blbost ;-)
    porad nemam telo, ale uz mam hlavu... nobody
    18.1.2021 16:38 Filip Jirsák | skóre: 68 | blog: Fa & Bi
    Rozbalit Rozbalit vše Re: Zašifrování HTML stránky, aby nebyl vidět zdroják
    Tohle nepoužívá žádnou obskurní funkci webového prohlížeče, ale standardní API. Když si to uložíte do souboru a zobrazíte v prohlížeči, bude to dělat přesně to, co popisujete. Akorát níže uvedený kód na první pohled uvidíte, když si zobrazíte zdrojový kód stránky.
    <script>fetch("/test.html").then(resp => resp.text()).then(text => document.write(text));</script>
    
    Nebo ten skript nemusíte inlinovat. Použijete externí skript, a nějak hezky ho pojmenujete, třeba mezerou nulové šířky. Takže pak celý HTML soubor bude obsahovat jenom tohle:
    <script src="​" />
    
    Spousta lidí se pak bude dušovat, že zdrojový kód stránky byl úplně prázdný…
    20.1.2021 15:43 Bherzet | skóre: 19 | blog: Bherzetův blog
    Rozbalit Rozbalit vše Re: Zašifrování HTML stránky, aby nebyl vidět zdroják
    Ohledně toho schování zdrojáku – moderní prohlížeče to asi mají všechny ošetřené, ale teoreticky je tu ještě možnost imitovat GUI prvky prohlížeče. Uživatel klikne pravým tlačítkem myši, zobrazí se něco, o čem se domnívá, že je to nativní kontextové menu, ale je to podvržené tou webovou stránkou, a po kliknutí na tlačítko, které vypadá, že zobrazuje zdroják, se otevře prázdné nové okno.

    V prohlížečích před pár lety podobné chyby byly (to je právě ta velká výhoda toho, když se zahodí desítky let vývoje stávajících operačních systémů a jejich roli začnou přebírat prohlížeče dokumentů), ale jsou to prostě chyby. Nikdy to nebude fungovat dlouhodobě a vždycky to půjde obejít wgetem se správně nastaveným User-Agentem.

    Je zbytečné na tom trávit čas a pokud to chce zákazník, jak někdo píše výše, je potřeba mu vysvětlit, že je to nesmysl. Nemá to absolutně žádný význam. Řešit nějaké blokování User-Agenta na serveru, jak naznačuje tazatel výše, je akorát způsob, jak omylem vytrestat legitimní uživatele s méně rozšířenými prohlížeči a boty (např. crawlery vyhledávačů). Přesto je stále otázkou jediného příkazu to obejít pro lidi, kterým by ten zdroják reálně k něčemu byl.

    Skrývání nebo šifrování zdrojáku je nesmysl. Jediné, co může mít nějaký význam, je obfuskace, ale i tam se díky vzniku developerské konzole, headless prohlížečů a pokrokům na poli AI prostor pro smysluplné využití neustále tenčí.

    Proč by to skrývání zdrojáku vůbec někdo řešil? Napadají mě jen samé fishy věci typu těžba bitcoinů bez vědomí uživatele.
    Jendа avatar 20.3.2021 01:05 Jendа | skóre: 78 | blog: Jenda | JO70FB
    Rozbalit Rozbalit vše Re: Zašifrování HTML stránky, aby nebyl vidět zdroják
    Tak jsem našel takovou stránku, hele.
    < Jenda`> Webová stránka dne: http://samy.pl/
    < Jenda`>  - zdroják si to přepisuje a protože browsery už neumí udělat "view page source" (ale jenom "generated source"), tak ho nemáte jak získat
    < Jenda`>  - detekce otevřené developer console
    < Jenda`>  - přes ZAP Proxy to nefunguje
    < Jenda`>  - BurpSuite to zacyklí v nekonečné smyčce a zatuhne
    
    Jendа avatar 20.3.2021 01:19 Jendа | skóre: 78 | blog: Jenda | JO70FB
    Rozbalit Rozbalit vše Re: Zašifrování HTML stránky, aby nebyl vidět zdroják
    • Detekce wgetu (i když si nastavíte user agent ze skutečného browser)
    • Kupodivu wget přes Burp to nedetekuje. Takže je to nějaká magie na nižší úrovni (TLS handshake?)
    Jendа avatar 20.3.2021 02:18 Jendа | skóre: 78 | blog: Jenda | JO70FB
    Rozbalit Rozbalit vše Re: Zašifrování HTML stránky, aby nebyl vidět zdroják
    FYI:
    • Detekce wgetu a asi ZAPu je založená na Accept hlavičce. Tohle funguje:
      wget --header="Accept-Encoding: gzip, deflate" --no-check-certificate --user-agent="Mozilla/5.0 (X11; Linux x86_64; rv:78.0) Gecko/20100101 Firefox/78.0" https://samy.pl/ -O - | gzip -d
      
      wget --header="Accept-Encoding: gzip, deflate" --header="Cookie: __utms=1" --header="Referer: https://samy.pl/" --user-agent="Mozilla/5.0 (X11; Linux x86_64; rv:78.0) Gecko/20100101 Firefox/78.0" https://samy.pl/load.js -O - | gzip -d
    • Ten Burp se předpokládám zasekne kvůli tomu množství whitespaců v tom dokumentu.
    • Co se týče divočin v samotném browseru, s tím neporadím, protože modernímu web developmentu nerozumím.
    20.3.2021 20:30 Filip Jirsák | skóre: 68 | blog: Fa & Bi
    Rozbalit Rozbalit vše Re: Zašifrování HTML stránky, aby nebyl vidět zdroják
    Já teda nevím, já tam zdroják vidím. Spousta mezer a tabulátorů, které se překládají JavaScriptem.
    Jendа avatar 20.3.2021 20:42 Jendа | skóre: 78 | blog: Jenda | JO70FB
    Rozbalit Rozbalit vše Re: Zašifrování HTML stránky, aby nebyl vidět zdroják
    Já tam ve Firefoxu vidím jenom tu přepsanou věc bez ničeho. Jak to děláte? Já pravým s altem a View Page Source.
    23.3.2021 13:54 stefan
    Rozbalit Rozbalit vše Re: Zašifrování HTML stránky, aby nebyl vidět zdroják
    Jak to vidím já, Ctrl-Shift-E načíst https://samy.pl/ kliknu na první request "/" a vyberu "Odpověď" potom přepnu z náhledu na obsah a vidím dlooouhý první řádek

    prostě jak už se tu psalo "dokonale" se to udělat nedá, tyhle snahy jsou tu od doby kdy html vzniklo a dodnes neexistuje nic co by opravdu bylo víc než zakázání zobrazení zdrojáku
    23.3.2021 21:27 ljkh
    Rozbalit Rozbalit vše Re: Zašifrování HTML stránky, aby nebyl vidět zdroják
    Tak schvalne co tam vidite, ty i Jirsak kdyz tam zdroj je jinde? Presne tady https://samy.pl/code/ a tady https://samy.pl/code/?ns=1
    24.3.2021 08:27 stefan
    Rozbalit Rozbalit vše Re: Zašifrování HTML stránky, aby nebyl vidět zdroják
    To co vidím se sem nedá vložit, redakční systém to nějak vykuchá, ale zkusím to ve zdrojáku vidím
    <!------------------------------------------------------------------------------
     ! Welcome to samy.pl.
     !
     ! You found Easter Egg #7!
     !
     ! Please note, this is NOT the source to samy.pl.
     ! You have accessed the page incorrectly. :)
     !
     ! In the various pages, you'll find benign code execution,
     ! seemingly invasive data exfiltration (that remains local
     ! to your machine and never reaches my system) and various
     ! innocuous challenges. Have fun!
     !
     ! -samy kamkar
     !------------------------------------------------------------------------------>
    <html><head>
    <META NAME="description" CONTENT="samy kamkar's open source projects, code, hardware, applied hacking, videos, talks, and other infectious technology.">
    <META NAME="easter-egg" CONTENT="You found Easter Egg #24!">
    <!-- Easter egg #7 -->
    <!-- This is NOT the source code to https://samy.pl -->
    <!-- Close the console/inspector to return to samy.pl :) -->
    <title>samy kamkar - home</title></head><body><script>
    if(navigator.userAgent.match(/Android|iPhone|iP.d/i)&&document.cookie.indexOf("iphone_redirect=false")==-1)window.location="code/";
    /*
    No source for you! Easter egg #2
    *//
    ale řádek 26 je spousta mezer a tabulátorů a na konci
    /.source.replace(/.{7}/g,function(w){document.write(String.fromCharCode(parseInt(w.replace(/ /g,'0').replace(/	/g,'1'),2)))});
    
    využívá to https://developer.mozilla.org/en-US/docs/Web/JavaScript/Reference/Global_Objects/RegExp/source
    /abcdefgh/
    je regexp a .source vrátí jeho zdroj. Výsledkem je
    <!DOCTYPE html> <html> <head> <noscript> <meta http-equiv="refresh" content="0;url=/code/?ns=1" /> </noscript> <meta http-equiv="Content-Type" content="text/html; charset=windows-1252"> <title>samy kamkar - home</title> <style> .Easter { egg: #14; this-is-not-the-source-to: samy-pl; } html, body { background:#cccccc url(/wallpapers/cn.jpg) repeat left top; background-size: 100%; font: normal 12px tahoma, arial, verdana, sans-serif; margin: 0; padding: 0; border: 0 none; overflow: hidden; height: 100%; } div { position: absolute; width: 600px; height: 300px; z-index: 15; top: 50%; left: 50%; margin: -100px 0 0 -300px; color: white; font-size: 30px; } .pb { display: none; } @media print { .pb { display: block; } .npb { display: none; } } </style> <script>var this_is_not_the_source_to_samy_pl;if(!window.$)(s=(z=document).getElementsByTagName(x="script")[0]).parentNode.insertBefore(z.createElement(x),s).src="//ajax.googleapis.com/ajax/libs/jquery/1.12.4/jquery.min.js";var ua=navigator.userAgent.toLowerCase();var dub=window;var cod=document;cod.ce=document.createElement;var msie=ua.indexOf("msie")>-1||ua.indexOf("edge/")>-1;if(!msie&&navigator.appName=="Netscape"&&ua.indexOf("trident/")>-1)msie=true;var chrome=!msie&&ua.indexOf("chrome")>-1;var safari=!chrome&&!msie&&ua.indexOf("safari")>-1;var firefox=!msie&&ua.indexOf("firefox")>-1;var hasWebRTC=navigator.getUserMedia||navigator.webkitGetUserMedia||navigator.mozGetUserMedia||navigator.msGetUserMedia||window.RTCPeerConnection;function dc(event){if(event.button==2){if(typeof stt=="function")stt("You found Easter Egg #12!");return false}}document.onmousedown=dc;(function(){if(typeof window.CustomEvent==="function")return false;function CustomEvent(event,params){params=params||{bubbles:false,cancelable:false,detail:undefined};var evt=document.createEvent("CustomEvent");evt.initCustomEvent(event,params.bubbles,params.cancelable,params.detail);return evt}CustomEvent.prototype=window.Event.prototype;window.CustomEvent=CustomEvent})();(function(){"use strict";var devtools={open:false,first:true,orientation:null};window.lq=devtools;var threshold=160;var emitEvent=function(state,orientation,eventname){if(!eventname)eventname="devtoolschange";window.dispatchEvent(new CustomEvent(eventname,{detail:{open:state,orientation:orientation}}))};setInterval(function(){var widthThreshold=window.outerWidth-window.innerWidth>threshold;var heightThreshold=window.outerHeight-window.innerHeight>threshold;var orientation=widthThreshold?"vertical":"horizontal";if(!(heightThreshold&&widthThreshold)&&(window.Firebug&&window.Firebug.chrome&&window.Firebug.chrome.isInitialized||widthThreshold||heightThreshold)){if(!devtools.open||devtools.orientation!==orientation){emitEvent(true,orientation);if(lq.first){lq.first=false;emitEvent(true,orientation,"firstevent")}}devtools.open=true;devtools.orientation=orientation}else{if(devtools.open){emitEvent(false,null);if(lq.first){lq.first=false;emitEvent(false,null,"firstevent")}}devtools.open=false;devtools.orientation=null}},1e3);if(typeof module!=="undefined"&&module.exports){module.exports=devtools}else{window.devtools=devtools}})();document.addEventListener("keydown",function(e){if(e.keyCode>=112&&e.keyCode<=123||(window.navigator.platform.match("Mac")?e.metaKey:e.ctrlKey)&&(e.keyCode==99||e.keyCode==67||e.keyCode==117||e.keyCode==85||e.keyCode==115||e.keyCode==83||e.keyCode==105||e.keyCode==73||e.keyCode==110||e.keyCode==78||e.keyCode==107||e.keyCode==75||e.keyCode==101||e.keyCode==69||e.keyCode==109||e.keyCode==77||e.keyCode==106||e.keyCode==74)){e.preventDefault();if(typeof stt=="function")stt("You found Easter Egg #10!")}},false);var deb=0;var _nc=0;var intr,_b;var timerMax=deb?2e3:500;var element=new Image;var egg8msg="No source for you! You found easter egg #8. Close the console to return to samy.pl :)";var egg8font="background: black; color: #00ff00; font-size: x-large;";var utm="__utmq";var firstload=1;var solecon=clone(console);function clone(obj){if(null==obj||"object"!=typeof obj)return obj;var copy=obj.constructor();for(var attr in obj)if(obj.hasOwnProperty(attr))copy[attr=="log"?"go":attr]=obj[attr];return copy}function al(x){if(!deb)return;solecon.go(x)}function opened(){rmbody()}function closed(){noconsole();location.reload(true)}function rmbody(){if(_nc)return;al("rm1");if(readCookie(utm)==2)return;al("rm2");createCookie(utm,2,365*10);_b=$("#allc").remove();$("noscript, script, link, div, iframe, meta, a, style").remove();$("head").append('<link rel="stylesheet" href="/css/desktop.css" type="text/css" />');$("body").append('<div style="position: absolute; width: 600px; height: 300px; z-index: 15; top: 50%; left: 50%; margin: -100px 0 0 -300px; color: white; font-size: 30px;">'+egg8msg+"</div>")}function noconsole3(){al("NOC3");noconsole()}function noconsole2(){al("NOC2");noconsole()}function noconsole(){al("noc1");if(readCookie(utm)==1)return;al("noc2");createCookie(utm,1,365*10);location.reload(true)}function egg8log(arg){if(msie)arg?solecon.go(egg8msg,arg):solecon.go(egg8msg);else arg?solecon.go("%c"+egg8msg,egg8font,arg):solecon.go("%c"+egg8msg,egg8font)}if(1){if(readCookie(utm)!=1){al("ngood");var noconyet=0;var threshold=160;var widthThreshold=window.outerWidth-window.innerWidth>threshold;var heightThreshold=window.outerHeight-window.innerHeight>threshold;var orientation=widthThreshold?"vertical":"horizontal";if(!(heightThreshold&&widthThreshold)&&(window.Firebug&&window.Firebug.chrome&&window.Firebug.chrome.isInitialized||widthThreshold||heightThreshold)){if(!devtools.open||devtools.orientation!==orientation){}}else{al("n1");noconsole();noconyet=1}}dtToStr();element.__defineGetter__("id",function(){al("dg");if(intr)clearTimeout(intr);if(!_b)rmbody();intr=setTimeout(noconsole2,timerMax*1.5)});al("elsec||s");window.addEventListener("devtoolschange",function(e){if(e.detail.open){rmbody();egg8log()}else{noconsole()}})}function dtToStr(){let cnt=0;let open=0;let timer;if(!safari&&!chrome)return;let isOpen=()=>cnt>(safari?0:chrome?1:1);let w=new Function;w.toString=()=>{cnt++;checkOpen()};let test=()=>{checkClosed();cnt=0;egg8log(w)};let checkOpen=()=>{if(isOpen()&&!open){open=1;opened()}};let checkClosed=()=>{if(!isOpen()&&open){open=0;closed()}};timer=setInterval(test,300)}function createCookie(name,value,days){if(days>0)eraseCookie(name);if(days){var date=new Date;date.setTime(date.getTime()+days*24*60*60*1e3);var expires="; expires="+date.toGMTString()}else var expires="";document.cookie=name+"="+value+expires+"; path=/"}function readCookie(name){var nameEQ=name+"=";var ca=document.cookie.split(";");for(var i=0;i<ca.length;i++){var c=ca[i];while(c.charAt(0)==" ")c=c.substring(1,c.length);if(c.indexOf(nameEQ)==0)return c.substring(nameEQ.length,c.length)}return null}function eraseCookie(name){createCookie(name,"",-1)} </script> </head> <body> <div class=npb>No source for you! You found easter egg #7. Close the console to return to samy.pl :)</div> <div class=pb>I am your printer. I have become self-aware. Easter Egg #15.</div> <script>function smr(){if(window.$)$("script,meta").remove();else setTimeout(smr,500)}smr(); </script> </body> </html>
    což už je ten kód která testuje otevření okna se zdrojákem dev tools atd. atd. prostě spousta "zbytečné" práce a výsledek k ničemu, je to asi jako ochrany her a DVD a tak podobně, pokud to má prohlížeč zobrazit "musí" to vidět a pokud to vidí prohlížeč dostane se k tomu i člověk :O)
    Jendа avatar 24.3.2021 08:50 Jendа | skóre: 78 | blog: Jenda | JO70FB
    Rozbalit Rozbalit vše Re: Zašifrování HTML stránky, aby nebyl vidět zdroják
    Ale to není ono… To o co jde je to, co ti vrátí ty dva wgety co jsem dával výše.
    Jendа avatar 24.3.2021 08:48 Jendа | skóre: 78 | blog: Jenda | JO70FB
    Rozbalit Rozbalit vše Re: Zašifrování HTML stránky, aby nebyl vidět zdroják
    To je alternativní verze pro uživatele bez JS a je bez těch eastereggů. Já mluvím o tom co se načte přímo na https://samy.pl/ (ta emulace „desktopu“), s JS.
    24.3.2021 09:47 stefan
    Rozbalit Rozbalit vše Re: Zašifrování HTML stránky, aby nebyl vidět zdroják
    wget --header="Accept-Encoding: gzip, deflate" --header="Cookie: __utms=1" --header="Referer: https://samy.pl/" --user-agent="Mozilla/5.0 (X11; Linux x86_64; rv:78.0) Gecko/20100101 Firefox/78.0" https://samy.pl/load.js -O - | gzip -d vrací na řádku 3 přesně to samé co popisuju
    /*
    No source for you! Easter egg #2
    *// ...zkráceno... /.source.replace(/.{7}/g,function(w){document.write(String.fromCharCode(parseInt(w.replace(/ /g,'0').replace(/	/g,'1'),2)))});
    
    24.3.2021 12:25 ljkh
    Rozbalit Rozbalit vše Re: Zašifrování HTML stránky, aby nebyl vidět zdroják
    !------------------------------------------------------------------------------
     ! Welcome to samy.pl.
     !
     ! In the various pages, you'll find benign code execution,
     ! seemingly invasive data exfiltration (that remains local
     ! to your machine and never reaches my system) and various
     ! innocuous challenges. Have fun!
     !
     ! -samy kamkar
     !------------------------------------------------------------------------------>
    script
    /*
    No source for you! You found Easter egg #3
    */
    /script
    Chapu nesmi tam byt nic jako
    No source for you! You found Easter egg #
    24.3.2021 12:34 ljkh
    Rozbalit Rozbalit vše Re: Zašifrování HTML stránky, aby nebyl vidět zdroják
    Kdyz jsem se dival k certifikatum vyuziva to min Cloudflare a neco z Googlu ... kdyz se podivam na info o strance pise ze je pusteny quirk mode ... na co vse vyuziva firefox ten quirk mode?..
    23.3.2021 21:39 lertimir | skóre: 64 | blog: Par_slov
    Rozbalit Rozbalit vše Re: Zašifrování HTML stránky, aby nebyl vidět zdroják
    já vidím tohle:
    <!DOCTYPE html>
    
    
        <html itemscope itemtype="https://schema.org/QAPage" class="html__responsive">
    
        <head>
    
            <title>javascript - How do I able to see the source code of this website www.samy.pl? - Stack Overflow</title>
            <link rel="shortcut icon" href="https://cdn.sstatic.net/Sites/stackoverflow/Img/favicon.ico?v=ec617d715196">
            <link rel="apple-touch-icon" href="https://cdn.sstatic.net/Sites/stackoverflow/Img/apple-touch-icon.png?v=c78bd457575a">
            <link rel="image_src" href="https://cdn.sstatic.net/Sites/stackoverflow/Img/apple-touch-icon.png?v=c78bd457575a"> 
            <link rel="search" type="application/opensearchdescription+xml" title="Stack Overflow" href="/opensearch.xml">
            <link rel="canonical" href="https://stackoverflow.com/questions/43381154/how-do-i-able-to-see-the-source-code-of-this-website-www-samy-pl" />
            <meta name="viewport" content="width=device-width, height=device-height, initial-scale=1.0, minimum-scale=1.0">
            <meta property="og:type" content= "website" />
            <meta property="og:url" content="https://stackoverflow.com/questions/43381154/how-do-i-able-to-see-the-source-code-of-this-website-www-samy-pl"/>
            <meta property="og:site_name" content="Stack Overflow" />
            <meta property="og:image" itemprop="image primaryImageOfPage" content="https://cdn.sstatic.net/Sites/stackoverflow/Img/apple-touch-icon@2.png?v=73d79a89bded" />
            <meta name="twitter:card" content="summary"/>
            <meta name="twitter:domain" content="stackoverflow.com"/>
            <meta name="twitter:title" property="og:title" itemprop="name" content="How do I able to see the source code of this website www.samy.pl?" />
            <meta name="twitter:description" property="og:description" itemprop="description" content="This website (www.samy.pl) uses some technique so that no one can inspect the code (Ctrl+Shift+I) or view source (Ctrl+U). It can detect if you have opened the console or inspector the code automat..." />
    
            <script src="https://ajax.googleapis.com/ajax/libs/jquery/1.12.4/jquery.min.js"></script>
            <script src="https://cdn.sstatic.net/Js/stub.en.js?v=f1b321be4890"></script>
        
            <link rel="stylesheet" type="text/css" href="https://cdn.sstatic.net/Shared/stacks.css?v=f0ad20c3c35c">
            <link rel="stylesheet" type="text/css" href="https://cdn.sstatic.net/Sites/stackoverflow/primary.css?v=57cf73133abb">
    
        
                <link rel="alternate" type="application/atom+xml" title="Feed for question 'How do I able to see the source code of this website www.samy.pl?'" href="/feeds/question/43381154">
                <meta name="twitter:app:country" content="US" />
                <meta name="twitter:app:name:iphone" content="Stack Exchange iOS" />
                <meta name="twitter:app:id:iphone" content="871299723" />
                <meta name="twitter:app:url:iphone" content="se-zaphod://stackoverflow.com/questions/43381154/how-do-i-able-to-see-the-source-code-of-this-website-www-samy-pl/47629515" />
                <meta name="twitter:app:name:ipad" content="Stack Exchange iOS" />
                <meta name="twitter:app:id:ipad" content="871299723" />
                <meta name="twitter:app:url:ipad" content="se-zaphod://stackoverflow.com/questions/43381154/how-do-i-able-to-see-the-source-code-of-this-website-www-samy-pl/47629515" />
                <meta name="twitter:app:name:googleplay" content="Stack Exchange Android">
                <meta name="twitter:app:url:googleplay" content="https://stackoverflow.com/questions/43381154/how-do-i-able-to-see-the-source-code-of-this-website-www-samy-pl/47629515">
                <meta name="twitter:app:id:googleplay" content="com.stackexchange.marvin">
            <script>
                StackExchange.ready(function () {
    
                        StackExchange.using("snippets", function () {
                            StackExchange.snippets.initSnippetRenderer();
                        });
                        
                    StackExchange.using("postValidation", function () {
                        StackExchange.postValidation.initOnBlurAndSubmit($('#post-form'), 2, 'answer');
                    });
    
    
                    StackExchange.question.init({showAnswerHelp:true,totalCommentCount:13,shownCommentCount:5,enableTables:true,questionId:43381154});
    
                    styleCode();
    
                        StackExchange.realtime.subscribeToQuestion('1', '43381154');
                        StackExchange.using("gps", function () { StackExchange.gps.trackOutboundClicks('#content', '.js-post-body'); });
    
    
    
                });
            </script>
    
            
            
            
            <link rel="stylesheet" type="text/css" href="https://cdn.sstatic.net/Shared/Channels/channels.css?v=a4fa343ab089">
            
            
            
    
    
        <script>
            StackExchange.init({"locale":"en","serverTime":1616531697,"routeName":"Questions/Show","stackAuthUrl":"https://stackauth.com","networkMetaHostname":"meta.stackexchange.com","site":{"name":"Stack Overflow","description":"Q&A for professional and enthusiast programmers","isNoticesTabEnabled":true,"enableNewTagCreationWarning":true,"insertSpaceAfterNameTabCompletion":false,"id":1,"cookieDomain":".stackoverflow.com","childUrl":"https://meta.stackoverflow.com","styleCodeWithHighlightjs":true,"negativeVoteScoreFloor":null,"enableSocialMediaInSharePopup":true,"protocol":"https"},"user":{"fkey":"41be0e11ff0983bf4a66e841ea7eb1d10d333a81847c19395d94a9518c426deb","tid":"4ae7c186-f08e-2d95-a1e1-cc08beb9f959","rep":0,"isAnonymous":true,"isAnonymousNetworkWide":true},"events":{"postType":{"question":1},"postEditionSection":{"title":1,"body":2,"tags":3}},"story":{"minCompleteBodyLength":75,"likedTagsMaxLength":300,"dislikedTagsMaxLength":300},"jobPreferences":{"maxNumDeveloperRoles":2,"maxNumIndustries":4},"svgIconPath":"https://cdn.sstatic.net/Img/stacks-icons","svgIconHash":"5acef7872715"}, {"userProfile":{"openGraphAPIKey":"4a307e43-b625-49bb-af15-ffadf2bda017"},"userMessaging":{"showNewFeatureNotice":true},"tags":{},"subscriptions":{"defaultBasicMaxTrueUpSeats":250,"defaultFreemiumMaxTrueUpSeats":50,"defaultMaxTrueUpSeats":1000},"snippets":{"renderDomain":"stacksnippets.net","snippetsEnabled":true},"slack":{"sidebarAdDismissCookie":"slack-sidebar-ad","sidebarAdDismissCookieExpirationDays":60.0},"site":{"allowImageUploads":true,"enableImgurHttps":true,"enableUserHovercards":true,"forceHttpsImages":true,"styleCode":true},"questions":{"enableQuestionTitleLengthLiveWarning":true,"maxTitleSize":150,"questionTitleLengthStartLiveWarningChars":50},"intercom":{"appId":"inf0secd","hostBaseUrl":"https://stacksnippets.net"},"paths":{},"monitoring":{"clientTimingsAbsoluteTimeout":30000,"clientTimingsDebounceTimeout":1000},"mentions":{"maxNumUsersInDropdown":50},"markdown":{"enableTables":true},"legal":{"oneTrustConfigId":"c3d9f1e3-55f3-4eba-b268-46cee4c6789c"},"flags":{"allowRetractingCommentFlags":true,"allowRetractingFlags":true},"comments":{},"accounts":{"currentPasswordRequiredForChangingStackIdPassword":true}});
            StackExchange.using.setCacheBreakers({"js/adops.en.js":"22a9bd59b1e9","js/ask.en.js":"91b4450eec6e","js/begin-edit-event.en.js":"7f52eac9bfd0","js/cm.en.js":"19f77005eb72","js/events.en.js":"cb258c49e9aa","js/explore-qlist.en.js":"5c7424c271ef","js/full-anon.en.js":"be6132ac7fd1","js/full.en.js":"8d69889132d5","js/help.en.js":"45dd3cca2f14","js/highlightjs-loader.en.js":"926162ad43f7","js/inline-tag-editing.en.js":"88510a5b8778","js/keyboard-shortcuts.en.js":"e32fccd0af5a","js/markdown-it-loader.en.js":"33be44e8a786","js/modElections.en.js":"fd4c5a91b3c5","js/mobile.en.js":"9400e80a5686","js/moderator.en.js":"4bdf01844847","js/postCollections-transpiled.en.js":"3ded2194b7d5","js/post-validation.en.js":"655e840d9e1c","js/prettify-full.en.js":"80c9e124fdfc","js/question-editor.en.js":"","js/review.en.js":"5704cc729d2f","js/review-v2-transpiled.en.js":"ad8fcb940e8a","js/revisions.en.js":"b4e68bf9aff9","js/stacks-editor.en.js":"dff59c911e36","js/tageditor.en.js":"7f379a506596","js/tageditornew.en.js":"cd62f702e223","js/tagsuggestions.en.js":"4d2d70ee1e16","js/wmd.en.js":"be2b839d9613","js/snippet-javascript-codemirror.en.js":"8b6cf98ec1e0"});
            StackExchange.using("gps", function() {
                 StackExchange.gps.init(true);
            });
        </script>
        <noscript id="noscript-css"><style>body,.top-bar{margin-top:1.9em}</style></noscript>
        </head>
        <body class="question-page unified-theme">
        <div id="notify-container"></div>
        <div id="custom-header"></div>
            
    <header class="top-bar js-top-bar top-bar__network _fixed">
        <div class="wmx12 mx-auto grid ai-center h100" role="menubar">
            <div class="-main grid--cell">
                    <a href="#" class="left-sidebar-toggle p0 ai-center jc-center js-left-sidebar-toggle" role="menuitem" aria-haspopup="true" aria-controls="left-sidebar" aria-expanded="false"><span class="ps-relative"></span></a>
                    <div class="topbar-dialog leftnav-dialog js-leftnav-dialog dno">
                        <div class="left-sidebar js-unpinned-left-sidebar" data-can-be="left-sidebar" data-is-here-when="sm"></div>
                    </div>
                        <a href="https://stackoverflow.com" class="-logo js-gps-track"
                            data-gps-track="top_nav.click({is_current:false, location:2, destination:8})">
                            <span class="-img _glyph">Stack Overflow</span>
                        </a>
    
    
    
            </div>
    
                <ol class="list-reset grid gs4" role="presentation">
    
                        <li class="grid--cell md:d-none">
                            <a href="/company" class="-marketing-link js-gps-track"
                               data-gps-track="top_nav.products.click({location:2, destination:7})"
                                data-ga="["top navigation","about menu click",null,null,null]">About</a>
                        </li>
    
                    <li class="grid--cell">
                        <a href="#"
                            class="-marketing-link js-gps-track js-products-menu"
                            aria-controls="products-popover"
                            data-controller="s-popover"
                            data-action="s-popover#toggle"
                            data-s-popover-placement="bottom"
                            data-s-popover-toggle-class="is-selected"
                            data-gps-track="top_nav.products.click({location:2, destination:1})"
                            data-ga="["top navigation","products menu click",null,null,null]">
                            Products
                        </a>
                    </li>
    
                        <li class="grid--cell md:d-none">
                            <a href="/teams" class="-marketing-link js-gps-track"
                               data-gps-track="top_nav.products.click({location:2, destination:7})"
                                data-ga="["top navigation","learn more - teams",null,null,null]">For Teams</a>
                        </li>
                </ol>
                <div class="s-popover ws2 mtn2 p0"
                        id="products-popover"
                        role="menu"
                        aria-hidden="true">
                    <div class="s-popover--arrow"></div>
                    <ol class="list-reset s-anchors s-anchors__inherit">
                        <li class="m6">
                            <a href="/questions" class="bar-sm p6 d-block h:bg-black-100 js-gps-track"
                               data-gps-track="top_nav.products.click({location:2, destination:2})"
                               data-ga="["top navigation","public qa submenu click",null,null,null]">
                                <span class="fs-body1 d-block">Stack Overflow</span>
                                <span class="fs-caption d-block fc-light">Public questions & answers</span>
                            </a>
                        </li>
                        <li class="m6">
                            <a href="/teams" class="bar-sm p6 d-block h:bg-black-100 js-gps-track"
                               data-gps-track="top_nav.products.click({location:2, destination:3})"
                               data-ga="["top navigation","teams submenu click",null,null,null]">
                                <span class="fs-body1 d-block">Stack Overflow for Teams</span>
                                <span class="fs-caption d-block fc-light">Where developers & technologists share private knowledge with coworkers</span>
                            </a>
                        </li>
                        <li class="m6">
                            <a href="/jobs?so_source=ProductsMenu&so_medium=StackOverflow" class="bar-sm p6 d-block h:bg-black-100 js-gps-track"
                                data-gps-track="top_nav.products.click({location:2, destination:9})"
                                data-ga="["top navigation","jobs submenu click",null,null,null]">
                                <span class="fs-body1 d-block">Jobs</span>
                                <span class="fs-caption d-block fc-light">Programming & related technical career opportunities</span>
                            </a>
                        </li>
                        <li class="m6">
                            <a href="https://stackoverflow.com/talent" class="bar-sm p6 d-block h:bg-black-100 js-gps-track"
                               data-gps-track="top_nav.products.click({location:2, destination:5})"
                               data-ga="["top navigation","talent submenu click",null,null,null]">
                                <span class="fs-body1 d-block">Talent</span>
                                <span class="fs-caption d-block fc-light">Recruit tech talent & build your employer brand</span>
                            </a>
                        </li>
                        <li class="m6">
                            <a href="https://stackoverflow.com/advertising" class="bar-sm p6 d-block h:bg-black-100 js-gps-track"
                               data-gps-track="top_nav.products.click({location:2, destination:6})"
                               data-ga="["top navigation","advertising submenu click",null,null,null]">
                                <span class="fs-body1 d-block">Advertising</span>
                                <span class="fs-caption d-block fc-light">Reach developers & technologists worldwide</span>
                            </a>
                        </li>
                        <li class="bg-black-025 bt bc-black-075 py6 px6 bbr-sm">
                            <a href="/company" class="fc-light d-block py6 px6 h:fc-black-800 js-gps-track"
                                data-gps-track="top_nav.products.click({location:2, destination:7})"
                                data-ga="["top navigation","about submenu click",null,null,null]">About the company</a>
                        </li>
                    </ol>
                </div>
    
                <form id="search" role="search" action=/search class="grid--cell fl-grow1 searchbar px12 js-searchbar " autocomplete="off">
                        <div class="ps-relative">
                            <input name="q"
                                   type="text"
                                   placeholder="Search…"
                                   value=""
                                   autocomplete="off"
                                   maxlength="240"
                                   class="s-input s-input__search js-search-field "
                                   aria-label="Search"
                                   aria-controls="top-search" 
                                   data-controller="s-popover"
                                   data-action="focus->s-popover#show"
                                   data-s-popover-placement="bottom-start"/>
                            <svg aria-hidden="true" class="s-input-icon s-input-icon__search svg-icon iconSearch" width="18" height="18" viewBox="0 0 18 18"><path d="M18 16.5l-5.14-5.18h-.35a7 7 0 10-1.19 1.19v.35L16.5 18l1.5-1.5zM12 7A5 5 0 112 7a5 5 0 0110 0z"/></svg>
                            <div class="s-popover p0 wmx100 wmn4 sm:wmn-initial js-top-search-popover" id="top-search" role="menu">
        <div class="s-popover--arrow"></div>
        <div class="js-spinner p24 grid ai-center jc-center d-none">
            <div class="s-spinner s-spinner__sm fc-orange-400">
                <div class="v-visible-sr">Loading…</div>
            </div>
        </div>
    
        <span class="v-visible-sr js-screen-reader-info"></span>
        <div class="js-ac-results overflow-y-auto hmx3 d-none"></div>
    
        <div class="js-search-hints" aria-describedby="Tips for searching"></div>
    </div>
                        </div>
                </form>
            
            
    
    <ol class="overflow-x-auto ml-auto -secondary grid ai-center list-reset h100 user-logged-out" role="presentation">
            <li class="-item searchbar-trigger"><a href="#" class="-link js-searchbar-trigger" role="button" aria-label="Search" aria-haspopup="true" aria-controls="search" title="Click to show search"><svg aria-hidden="true" class="svg-icon iconSearch" width="18" height="18" viewBox="0 0 18 18"><path d="M18 16.5l-5.14-5.18h-.35a7 7 0 10-1.19 1.19v.35L16.5 18l1.5-1.5zM12 7A5 5 0 112 7a5 5 0 0110 0z"/></svg></a></li>
    
                <li class="-ctas">
                                <a href="https://stackoverflow.com/users/login?ssrc=head&returnurl=https%3a%2f%2fstackoverflow.com%2fquestions%2f43381154%2fhow-do-i-able-to-see-the-source-code-of-this-website-www-samy-pl%2f47629515" class="login-link s-btn s-btn__filled py8 js-gps-track" rel="nofollow"
                                   data-gps-track="login.click" data-ga="["top navigation","login button click",null,null,null]">Log in</a>
                                <a href="https://stackoverflow.com/users/signup?ssrc=head&returnurl=%2fusers%2fstory%2fcurrent" class="login-link s-btn s-btn__primary py8" rel="nofollow" data-ga="["sign up","Sign Up Navigation","Header",null,null]">Sign up</a>
    
                </li>
    
        <li class="js-topbar-dialog-corral" role="presentation">
                
    
        <div class="topbar-dialog siteSwitcher-dialog dno" role="menu">
            <div class="header">
                <h3>
                    <a href="https://stackoverflow.com">current community</a>
                </h3>
            </div>
            <div class="modal-content bg-powder-050">
                <ul class="current-site">
                        <li class="grid">
                                <div class="fl1">
                    <a href="https://stackoverflow.com"
           class="current-site-link site-link js-gps-track grid gs8 gsx"
           data-id="1"
           data-gps-track="site_switcher.click({ item_type:3 })">
            <div class="favicon favicon-stackoverflow site-icon grid--cell" title="Stack Overflow"></div>
            <span class="grid--cell fl1">
                Stack Overflow
            </span>
        </a>
    
        </div>
        <div class="related-links">
                <a href="https://stackoverflow.com/help" class="js-gps-track" data-gps-track="site_switcher.click({ item_type:14 })">help</a>
                <a href="https://chat.stackoverflow.com/?tab=site&host=stackoverflow.com" class="js-gps-track" data-gps-track="site_switcher.click({ item_type:6 })">chat</a>
        </div>
    
                        </li>
                        <li class="related-site grid">
                                <div class="L-shaped-icon-container">
            <span class="L-shaped-icon"></span>
        </div>
    
                                <a href="https://meta.stackoverflow.com"
           class=" site-link js-gps-track grid gs8 gsx"
           data-id="552"
           data-gps-track="site.switch({ target_site:552, item_type:3 }),site_switcher.click({ item_type:4 })">
            <div class="favicon favicon-stackoverflowmeta site-icon grid--cell" title="Meta Stack Overflow"></div>
            <span class="grid--cell fl1">
                Meta Stack Overflow
            </span>
        </a>
    
                        </li>
                </ul>
            </div>
    
            <div class="header" id="your-communities-header">
                <h3>
    your communities            </h3>
    
            </div>
            <div class="modal-content" id="your-communities-section">
    
                    <div class="call-to-login">
    <a href="https://stackoverflow.com/users/signup?ssrc=site_switcher&returnurl=%2fusers%2fstory%2fcurrent" class="login-link js-gps-track" data-gps-track="site_switcher.click({ item_type:10 })">Sign up</a> or <a href="https://stackoverflow.com/users/login?ssrc=site_switcher&returnurl=https%3a%2f%2fstackoverflow.com%2fquestions%2f43381154%2fhow-do-i-able-to-see-the-source-code-of-this-website-www-samy-pl%2f47629515" class="login-link js-gps-track" data-gps-track="site_switcher.click({ item_type:11 })">log in</a> to customize your list.                </div>
            </div>
    
            <div class="header">
                <h3><a href="https://stackexchange.com/sites">more stack exchange communities</a>
                </h3>
                <a href="https://stackoverflow.blog" class="fr">company blog</a>
            </div>
            <div class="modal-content">
                    <div class="child-content"></div>
            </div>        
        </div>
    
        </li>
    </ol>
        </div>
    </header>
    
        <script>
            StackExchange.ready(function () { StackExchange.topbar.init(); });
    StackExchange.scrollPadding.setPaddingTop(50, 10);    </script>
    
    
    
                
    
            
        
    <div class="bg-black-025 bs-sm bt bc-black-100 ps-fixed l0 r0 b0 z-nav js-dismissable-hero" data-campaign-name="stk">
        <div class="grid wmx12 mx-auto px8 py12 jc-space-between ai-center lg:pl24 lg:pr24 md:fd-column sm:fd-row sm:ai-center">
            <div class="grid--cell fs-body2 fl1 mr16 md:mr0 md:mb12 sm:mb0 sm:mr16">
                <p class="mb0"><strong>Join Stack Overflow</strong> to learn, share knowledge, and build your career.</p>
            </div>
    
            <div id="openid-buttons" class="grid gs8 gsx ai-center sm:jc-space-between">
                <a class="grid--cell s-btn s-btn__filled ws-nowrap" href="/users/signup?ssrc=hero&returnurl=https%3a%2f%2fstackoverflow.com%2fquestions%2f43381154%2fhow-do-i-able-to-see-the-source-code-of-this-website-www-samy-pl%2f47629515" rel="nofollow" data-ga="["sign up","Sign Up Navigation","Question Hero",null,null]">
                        <span class="sm:d-none">Sign up with email</span>
                        <span class="d-none sm:d-inline">Sign up</span>
                    
                </a>
                    <button class="grid--cell ws-nowrap s-btn s-btn__icon s-btn__google ta-center js-major-provider sm:d-none" data-provider='google' data-oauthserver='https://accounts.google.com/o/oauth2/auth' data-oauthversion='2.0' data-ga="["sign up","Sign Up Started - Google","Question Hero",null,null]">
                        <svg aria-hidden="true" class="native svg-icon iconGoogle" width="18" height="18" viewBox="0 0 18 18"><path d="M16.51 8H8.98v3h4.3c-.18 1-.74 1.48-1.6 2.04v2.01h2.6a7.8 7.8 0 002.38-5.88c0-.57-.05-.66-.15-1.18z" fill="#4285F4"/><path d="M8.98 17c2.16 0 3.97-.72 5.3-1.94l-2.6-2a4.8 4.8 0 01-7.18-2.54H1.83v2.07A8 8 0 008.98 17z" fill="#34A853"/><path d="M4.5 10.52a4.8 4.8 0 010-3.04V5.41H1.83a8 8 0 000 7.18l2.67-2.07z" fill="#FBBC05"/><path d="M8.98 4.18c1.17 0 2.23.4 3.06 1.2l2.3-2.3A8 8 0 001.83 5.4L4.5 7.49a4.77 4.77 0 014.48-3.3z" fill="#EA4335"/></svg>
                        Sign up with Google
                    </button>
                    <button class="grid--cell ws-nowrap s-btn s-btn__icon s-btn__github ta-center js-major-provider sm:d-none" data-provider='github' data-oauthserver='https://github.com/login/oauth/authorize' data-oauthversion='2.0' data-ga="["sign up","Sign Up Started - GitHub","Question Hero",null,null]">
                        <svg aria-hidden="true" class="svg-icon iconGitHub" width="18" height="18" viewBox="0 0 18 18"><path d="M9 1a8 8 0 00-2.53 15.59c.4.07.55-.17.55-.38l-.01-1.49c-2.01.37-2.53-.49-2.69-.94-.09-.23-.48-.94-.82-1.13-.28-.15-.68-.52-.01-.53.63-.01 1.08.58 1.23.82.72 1.21 1.87.87 2.33.66.07-.52.28-.87.51-1.07-1.78-.2-3.64-.89-3.64-3.95 0-.87.31-1.59.82-2.15-.08-.2-.36-1.02.08-2.12 0 0 .67-.21 2.2.82a7.42 7.42 0 014 0c1.53-1.04 2.2-.82 2.2-.82.44 1.1.16 1.92.08 2.12.51.56.82 1.27.82 2.15 0 3.07-1.87 3.75-3.65 3.95.29.25.54.73.54 1.48l-.01 2.2c0 .21.15.46.55.38A8.01 8.01 0 009 1z" fill="#010101"/></svg>
                        Sign up with GitHub
                    </button>
                    <button class="grid--cell ws-nowrap s-btn s-btn__icon s-btn__facebook ta-center js-major-provider sm:d-none" data-provider='facebook' data-oauthserver='https://www.facebook.com/v2.0/dialog/oauth' data-oauthversion='2.0' data-ga="["sign up","Sign Up Started - Facebook","Question Hero",null,null]">
                        <svg aria-hidden="true" class="svg-icon iconFacebook" width="18" height="18" viewBox="0 0 18 18"><path d="M3 1a2 2 0 00-2 2v12c0 1.1.9 2 2 2h12a2 2 0 002-2V3a2 2 0 00-2-2H3zm6.55 16v-6.2H7.46V8.4h2.09V6.61c0-2.07 1.26-3.2 3.1-3.2.88 0 1.64.07 1.87.1v2.16h-1.29c-1 0-1.19.48-1.19 1.18V8.4h2.39l-.31 2.42h-2.08V17h-2.5z" fill="#4167B2"/></svg>
                        Sign up with Facebook
                    </button>
                    <button class="grid--cell s-btn s-btn__muted s-btn__icon px8 js-dismiss" title="Dismiss"><svg aria-hidden="true" class="svg-icon iconClear" width="18" height="18" viewBox="0 0 18 18"><path d="M15 4.41L13.59 3 9 7.59 4.41 3 3 4.41 7.59 9 3 13.59 4.41 15 9 10.41 13.59 15 15 13.59 10.41 9 15 4.41z"/></svg></button>
            </div>
    
            <form id="login-form" action="/users/signup?ssrc=hero&returnurl=https%3a%2f%2fstackoverflow.com%2fquestions%2f43381154%2fhow-do-i-able-to-see-the-source-code-of-this-website-www-samy-pl%2f47629515" method="POST">
    
                <input type="hidden" name="fkey" value="41be0e11ff0983bf4a66e841ea7eb1d10d333a81847c19395d94a9518c426deb">
    
                <input type="hidden" name="ssrc" value="" />
    
                <input type="hidden" id="oauth_version" name="oauth_version" />
                <input type="hidden" id="oauth_server" name="oauth_server" />
    
                <div id="se-login-fields">
                    <input type="hidden" name="legalLinksShown" value="0" />
                </div>
            </form>
            <script>
                StackExchange.ready(function () {
                    StackExchange.InlineAuth.init();
                })
            </script>
        </div>
    </div>
    
    <script>
        StackExchange.ready(function () {
            StackExchange.Hero.init("stk", "a");
    
            var location = 0;
            if ($("body").hasClass("questions-page")) {
                location = 1;
            } else if ($("body").hasClass("question-page")) {
                location = 1;
            } else if ($("body").hasClass("faq-page")) {
                location = 5;
            } else if ($("body").hasClass("home-page")) {
                location = 3;
            }
    
            $('.js-cta-button').click(function () {
                StackExchange.using("gps", function () {
                    StackExchange.gps.track("hero.action", { hero_action_type: 'cta', location: location }, true);
                });
            });
    
            // TODO: we should review the class names and whatnot in use here. Older heroes use id selectors, the newer
            // sticky question hero on SO has a .js-dismiss class instead, but it's apparently not used anywhere... 
            // It's not great. Ideally we'd have a set of classes in the partials above that would correspond to 
            // the behaviours we want here in a more clear way. 
    
            // sticky question-page hero at the bottom of the page on SO
            $('.js-dismiss').on('click', function () {
                StackExchange.using("gps", function () {
                    StackExchange.gps.track("hero.action", { hero_action_type: "close", location: location }, true);
                });
                StackExchange.Hero.dismiss();
                $(".js-dismissable-hero").fadeOut("fast");
            });
        });
    </script>
    
    
    
        <div class="container">
                
    
    
    <div id="left-sidebar" data-is-here-when="md lg" class="left-sidebar js-pinned-left-sidebar ps-relative">
        <div class="left-sidebar--sticky-container js-sticky-leftnav">
            <nav role="navigation">
                <ol class="nav-links">
            <li class="">
                <a
                   href="/"
                   class="pl8 js-gps-track nav-links--link"
                   
                   data-gps-track="top_nav.click({is_current:false, location:2, destination:8})" 
                   aria-controls="" data-controller="" data-s-popover-placement="right">
                        <div class="grid ai-center">
                            <div class="grid--cell truncate">
                                Home
                            </div>
                        </div>
                </a>
            </li>
                    <li>
                        <ol class="nav-links">
                                <li class="fs-fine tt-uppercase ml8 mt16 mb4 fc-light">Public</li>
    
            <li class=" youarehere">
                <a id="nav-questions"
                   href="/questions"
                   class="pl8 js-gps-track nav-links--link -link__with-icon"
                   
                   data-gps-track="top_nav.click({is_current:true, location:2, destination:1})" 
                   aria-controls="" data-controller="" data-s-popover-placement="right">
    <svg aria-hidden="true" class="svg-icon iconGlobe" width="18" height="18" viewBox="0 0 18 18"><path d="M9 1a8 8 0 100 16A8 8 0 009 1zM8 15.32a6.4 6.4 0 01-5.23-7.75L7 11.68v.8c0 .88.12 1.32 1 1.32v1.52zm5.72-2c-.2-.66-1-1.32-1.72-1.32h-1v-2c0-.44-.56-1-1-1H6V7h1c.44 0 1-.56 1-1V5h2c.88 0 1.4-.72 1.4-1.6v-.33a6.4 6.4 0 012.32 10.24v.01z"/></svg>                    <span class="-link--channel-name">Questions</span>
                </a>
            </li>
    
            <li class="">
                <a id="nav-tags"
                   href="/tags"
                   class=" js-gps-track nav-links--link"
                   
                   data-gps-track="top_nav.click({is_current:false, location:2, destination:2})" 
                   aria-controls="" data-controller="" data-s-popover-placement="right">
                        <div class="grid ai-center">
                            <div class="grid--cell truncate">
                                Tags
                            </div>
                        </div>
                </a>
            </li>
            <li class="">
                <a id="nav-users"
                   href="/users"
                   class=" js-gps-track nav-links--link"
                   
                   data-gps-track="top_nav.click({is_current:false, location:2, destination:3})" 
                   aria-controls="" data-controller="" data-s-popover-placement="right">
                        <div class="grid ai-center">
                            <div class="grid--cell truncate">
                                Users
                            </div>
                        </div>
                </a>
            </li>
                                <li class="fs-fine tt-uppercase ml8 mt16 mb4 fc-light">Find a Job</li>
            <li class="">
                <a id="nav-jobs"
                   href="/jobs?so_medium=StackOverflow&so_source=SiteNav"
                   class=" js-gps-track nav-links--link"
                   
                   data-gps-track="top_nav.click({is_current:false, location:2, destination:6})" 
                   aria-controls="" data-controller="" data-s-popover-placement="right">
                        <div class="grid ai-center">
                            <div class="grid--cell truncate">
                                Jobs
                            </div>
                        </div>
                </a>
            </li>
            <li class="">
                <a id="nav-companies"
                   href="/jobs/companies?so_medium=StackOverflow&so_source=SiteNav"
                   class=" js-gps-track nav-links--link"
                   
                   data-gps-track="top_nav.click({is_current:false, location:2, destination:12})" 
                   aria-controls="" data-controller="" data-s-popover-placement="right">
                        <div class="grid ai-center">
                            <div class="grid--cell truncate">
                                Companies
                            </div>
                        </div>
                </a>
            </li>
                        </ol>
                    </li>
                        <li>
                            <ol class="nav-links">
                                        
    
    <div class="js-freemium-cta ps-relative">
    
        <div class="fs-fine tt-uppercase ml8 mt16 mb8 fc-light">Teams</div>
    
        <div class="bt bl bb bc-black-075 p12 pb6 fc-black-600 blr-sm overflow-hidden">
            <strong class="fc-black-750 mb6">Stack Overflow for Teams</strong>
            – Collaborate and share knowledge with a private group.
            
            <img class="wmx100 mx-auto my8 h-auto d-block" width="139" height="114" src="https://cdn.sstatic.net/Img/teams/teams-illo-free-sidebar-promo.svg?v=47faa659a05e" alt="">
    
            <a href="https://stackoverflow.com/teams/create/free?utm_source=so-owned&utm_medium=side-bar&utm_campaign=campaign-38&utm_content=cta" 
               class="w100 s-btn s-btn__primary s-btn__xs js-gps-track"
               data-gps-track="teams.create.left-sidenav.click({ Action: 6 })"
               data-ga="["teams left navigation - anonymous","left nav free cta","stackoverflow.com/teams/create/free",null,null]">Create a free Team</a>
            <a href="https://stackoverflow.com/teams" 
               class="w100 s-btn s-btn__muted s-btn__xs js-gps-track"
               data-gps-track="teams.create.left-sidenav.click({ Action: 5 })"
               data-ga="["teams left navigation - anonymous","left nav free cta","stackoverflow.com/teams",null,null]">What is Teams?</a>
        </div>
    </div>
    
                                    <li class="grid ai-center jc-space-between ml8 mt24 mb4 js-create-team-cta d-none">
                                        <div class="grid--cell tt-uppercase fs-fine fc-light">Teams</div>
                                        <div class="grid--cell fs-fine fc-light mr4">
                                            <a href="javascript:void(0)" class="s-link s-link__inherit js-gps-track"
                                                role="button"
                                                aria-controls="popover-teams-create-cta"
                                                data-controller="s-popover"
                                                data-action="s-popover#toggle"
                                                data-s-popover-placement="bottom-start"
                                                data-s-popover-toggle-class="is-selected"
                                                data-gps-track="teams.create.left-sidenav.click({ Action: ShowInfo })"
                                                data-ga="["teams left navigation - anonymous","left nav show teams info",null,null,null]">
                                                What’s this?
                                            </a>
    
                                        </div>
                                    </li>
                                    <li class="ps-relative js-create-team-cta d-none">
                                        <a href="https://stackoverflow.com/teams/create/free?utm_source=so-owned&utm_medium=side-bar&utm_campaign=campaign-38&utm_content=cta"
                                           class="pl8 js-gps-track nav-links--link"
                                           title="Stack Overflow for Teams is a private, secure spot for your organization's questions and answers."
                                           data-gps-track="teams.create.left-sidenav.click({ Action: FreemiumTeamsCreateClick })"
                                           data-ga="["teams left navigation - anonymous","left nav team click","stackoverflow.com/teams/create/free",null,null]">
                                            <div class="grid ai-center">
                                                <div class="grid--cell s-avatar va-middle bg-orange-400">
                                                    <div class="s-avatar--letter mtn1">
                                                        <svg aria-hidden="true" class="svg-icon iconBriefcaseSm" width="14" height="14" viewBox="0 0 14 14"><path d="M4 3a1 1 0 011-1h4a1 1 0 011 1v1h.5c.83 0 1.5.67 1.5 1.5v5c0 .83-.67 1.5-1.5 1.5h-7A1.5 1.5 0 012 10.5v-5C2 4.67 2.67 4 3.5 4H4V3zm5 1V3H5v1h4z"/></svg>
                                                    </div>
                                                    <svg aria-hidden="true" class="native s-avatar--badge svg-icon iconShieldXSm" width="9" height="10" viewBox="0 0 9 10"><path d="M0 1.84L4.5 0 9 1.84v3.17C9 7.53 6.3 10 4.5 10 2.7 10 0 7.53 0 5.01V1.84z" fill="var(--white)"/><path d="M1 2.5L4.5 1 8 2.5v2.51C8 7.34 5.34 9 4.5 9 3.65 9 1 7.34 1 5.01V2.5zm2.98 3.02L3.2 7h2.6l-.78-1.48a.4.4 0 01.15-.38c.34-.24.73-.7.73-1.14 0-.71-.5-1.23-1.41-1.23-.92 0-1.39.52-1.39 1.23 0 .44.4.9.73 1.14.12.08.18.23.15.38z" fill="var(--black-500)"/></svg>
                                                </div>
                                                <div class="grid--cell pl6">
                                                    Create free Team
                                                </div>
                                            </div>
                                        </a>
                                    </li>
                            </ol>
                        </li>
                </ol>
            </nav>
        </div>
    
    
    
            <div class="s-popover"
                 id="popover-teams-create-cta"
                 role="menu"
                 aria-hidden="true">
                <div class="s-popover--arrow"></div>
    
                <div class="ps-relative overflow-hidden">
                    <p class="mb2"><strong>Teams</strong></p>
                    <p class="mb12 fs-caption fc-black-400">Q&A for work</p>
                    <p class="mb12 fs-caption fc-medium">Connect and share knowledge within a single location that is structured and easy to search.</p>
                    <a href="https://stackoverflow.com/teams"
                       class="js-gps-track s-btn s-btn__primary s-btn__xs"
                       data-gps-track="teams.create.left-sidenav.click({ Action: CtaClick })"
                       data-ga="["teams left navigation - anonymous","left nav cta","stackoverflow.com/teams",null,null]">
                        Learn more
                    </a>
                </div>
    
                <div class="ps-absolute t8 r8">
                    <svg aria-hidden="true" class="fc-orange-500 svg-spot spotPeople" width="48" height="48" viewBox="0 0 48 48"><path d="M13.5 28a4.5 4.5 0 100-9 4.5 4.5 0 000 9zM7 30a1 1 0 011-1h11a1 1 0 011 1v5h11v-5a1 1 0 011-1h12a1 1 0 011 1v10a2 2 0 01-2 2H33v5a1 1 0 01-1 1H20a1 1 0 01-1-1v-5H8a1 1 0 01-1-1V30zm25-6.5a4.5 4.5 0 109 0 4.5 4.5 0 00-9 0zM24.5 34a4.5 4.5 0 100-9 4.5 4.5 0 000 9z" opacity=".2"/><path d="M16.4 26.08A6 6 0 107.53 26C5.64 26.06 4 27.52 4 29.45V40a1 1 0 001 1h9a1 1 0 100-2h-4v-7a1 1 0 10-2 0v7H6v-9.55c0-.73.67-1.45 1.64-1.45H16a1 1 0 00.4-1.92zM12 18a4 4 0 110 8 4 4 0 010-8zm16.47 14a6 6 0 10-8.94 0A3.6 3.6 0 0016 35.5V46a1 1 0 001 1h14a1 1 0 001-1V35.5c0-1.94-1.64-3.42-3.53-3.5zM20 28a4 4 0 118 0 4 4 0 01-8 0zm-.3 6h8.6c1 0 1.7.75 1.7 1.5V45h-2v-7a1 1 0 10-2 0v7h-4v-7a1 1 0 10-2 0v7h-2v-9.5c0-.75.7-1.5 1.7-1.5zM42 22c0 1.54-.58 2.94-1.53 4A3.5 3.5 0 0144 29.45V40a1 1 0 01-1 1h-9a1 1 0 110-2h4v-7a1 1 0 112 0v7h2v-9.55A1.5 1.5 0 0040.48 28H32a1 1 0 01-.4-1.92A6 6 0 1142 22zm-2 0a4 4 0 10-8 0 4 4 0 008 0z"/><path d="M17 10a1 1 0 011-1h12a1 1 0 110 2H18a1 1 0 01-1-1zm1-5a1 1 0 100 2h12a1 1 0 100-2H18zm-4-4a1 1 0 00-1 1v12a1 1 0 001 1h5.09l4.2 4.2a1 1 0 001.46-.04l3.7-4.16H34a1 1 0 001-1V2a1 1 0 00-1-1H14zm1 12V3h18v10h-5a1 1 0 00-.75.34l-3.3 3.7-3.74-3.75a1 1 0 00-.71-.29H15z" opacity=".35"/></svg>
                </div>
            </div>
    
    </div>
    
    
    
            <div id="content" class="snippet-hidden">
    
                
    <div itemprop="mainEntity" itemscope itemtype="https://schema.org/Question">
        <link itemprop="image" href="https://cdn.sstatic.net/Sites/stackoverflow/Img/apple-touch-icon.png?v=c78bd457575a">
    
        <div class="inner-content clearfix">
    
            
    
                <div id="question-header" class="grid sm:fd-column">
                            <h1 itemprop="name" class="fs-headline1 ow-break-word mb8 grid--cell fl1"><a href="/questions/43381154/how-do-i-able-to-see-the-source-code-of-this-website-www-samy-pl" class="question-hyperlink">How do I able to see the source code of this website www.samy.pl?</a></h1>
                    <div class="ml12 aside-cta grid--cell print:d-none sm:ml0 sm:mb12 sm:order-first sm:as-end">
                            <a href="/questions/ask" class="ws-nowrap s-btn s-btn__primary">
            Ask Question
        </a>
    
                    </div>
                </div>
                <div class="grid fw-wrap pb8 mb16 bb bc-black-075">
                        <div class="grid--cell ws-nowrap mr16 mb8" title="2017-04-12 23:21:33Z">
                            <span class="fc-light mr2">Asked</span>
                            <time itemprop="dateCreated" datetime="2017-04-12T23:21:33">3 years, 11 months ago</time>
                        </div>
                        <div class="grid--cell ws-nowrap mr16 mb8">
                            <span class="fc-light mr2">Active</span>
                            <a href="?lastactivity" class="s-link s-link__inherit" title="2019-06-01 01:33:34Z">1 year, 9 months ago</a>
                        </div>
                        <div class="grid--cell ws-nowrap mb8" title="Viewed 1,937 times">
                            <span class="fc-light mr2">Viewed</span>
                            2k times
                        </div>
                </div>
                <div id="mainbar" role="main" aria-label="question and answers">
    
                    
    <div class="question" data-questionid="43381154" data-ownerid="7859169" data-score="11"  id="question">
        <style>
        </style>
    <div class="js-zone-container zone-container-main">
        <div id="dfp-tlb" class="everyonelovesstackoverflow everyoneloves__top-leaderboard everyoneloves__leaderboard"></div>
        <div class="js-report-ad-button-container " style="width: 728px"></div>
    </div>
    
        <div class="post-layout">
            <div class="votecell post-layout--left">
                <div class="js-voting-container grid jc-center fd-column ai-stretch gs4 fc-black-200" data-post-id="43381154">
            <button class="js-vote-up-btn grid--cell s-btn s-btn__unset c-pointer" data-controller="s-tooltip" data-s-tooltip-placement="right" title="This question shows research effort; it is useful and clear" aria-pressed="false" aria-label="Up vote" data-selected-classes="fc-theme-primary"><svg aria-hidden="true" class="m0 svg-icon iconArrowUpLg" width="36" height="36" viewBox="0 0 36 36"><path d="M2 26h32L18 10 2 26z"/></svg></button>
            <div class="js-vote-count grid--cell fc-black-500 fs-title grid fd-column ai-center" itemprop="upvoteCount" data-value="11">11</div>
            <button class="js-vote-down-btn grid--cell s-btn s-btn__unset c-pointer" data-controller="s-tooltip" data-s-tooltip-placement="right" title="This question does not show any research effort; it is unclear or not useful" aria-pressed="false" aria-label="Down vote" data-selected-classes="fc-theme-primary"><svg aria-hidden="true" class="m0 svg-icon iconArrowDownLg" width="36" height="36" viewBox="0 0 36 36"><path d="M2 10h32L18 26 2 10z"/></svg></button>
    
            <button class="js-bookmark-btn s-btn s-btn__unset c-pointer py4 js-gps-track" 
                    data-controller="s-tooltip" data-s-tooltip-placement="right" title="Bookmark this question."
                    aria-pressed="false" aria-label="Bookmark (3)" data-selected-classes="fc-yellow-600"
                    data-gps-track="post.click({ item: 1, priv: 0, post_type: 1 })">
                <svg aria-hidden="true" class="svg-icon iconBookmark" width="18" height="18" viewBox="0 0 18 18"><path d="M6 1a2 2 0 00-2 2v14l5-4 5 4V3a2 2 0 00-2-2H6zm3.9 3.83h2.9l-2.35 1.7.9 2.77L9 7.59l-2.35 1.7.9-2.76-2.35-1.7h2.9L9 2.06l.9 2.77z"/></svg>
                <div class="js-bookmark-count mt4" data-value="3">3</div>
            </button>
        
    
        
            <a class="js-post-issue grid--cell s-btn s-btn__unset c-pointer py6 mx-auto" href="/posts/43381154/timeline" data-shortcut="T" data-ks-title="timeline" data-controller="s-tooltip" data-s-tooltip-placement="right" title="Show activity on this post." aria-label="Timeline"><svg aria-hidden="true" class="mln2 mr0 svg-icon iconHistory" width="19" height="18" viewBox="0 0 19 18"><path d="M3 9a8 8 0 113.73 6.77L8.2 14.3A6 6 0 105 9l3.01-.01-4 4-4-4h3L3 9zm7-4h1.01L11 9.36l3.22 2.1-.6.93L10 10V5z"/></svg></a>
    
    </div>
            </div>
    
            
    
    <div class="postcell post-layout--right">
        
        <div class="s-prose js-post-body" itemprop="text">
                    
    <p>This <a href="http://www.samy.pl" rel="noreferrer">website (www.samy.pl)</a> uses some technique so that no one can inspect the code <br />(<kbd>Ctrl</kbd>+<kbd>Shift</kbd>+<kbd>I</kbd>) or view source (<kbd>Ctrl</kbd>+<kbd>U</kbd>). It can detect if you have opened the console or inspector the code automatically changes.</p>
    
    <p>How this is possible?</p>
        </div>
    
            <div class="mt24 mb12">
                <div class="post-taglist grid gs4 gsy fd-column">
                    <div class="grid ps-relative">
                        <a href="/questions/tagged/javascript" class="post-tag" title="show questions tagged 'javascript'" rel="tag">javascript</a> <a href="/questions/tagged/html" class="post-tag" title="show questions tagged 'html'" rel="tag">html</a> <a href="/questions/tagged/iframe" class="post-tag" title="show questions tagged 'iframe'" rel="tag">iframe</a> <a href="/questions/tagged/web" class="post-tag" title="show questions tagged 'web'" rel="tag">web</a> 
                    </div>
                </div>
            </div>
    
        <div class="mb0 ">
            <div class="mt16 grid gs8 gsy fw-wrap jc-end ai-start pt4 mb16">
                <div class="grid--cell mr16 fl1 w96">
                    
    
    <div class="js-post-menu pt2" data-post-id="43381154">
        <div class="grid d-flex gs8 s-anchors s-anchors__muted fw-wrap">
    
            <div class="grid--cell">
                <a href="/q/43381154"
                   rel="nofollow"
                   itemprop="url"
                   class="js-share-link js-gps-track"
                   title="Short permalink to this question"
                   data-gps-track="post.click({ item: 2, priv: 0, post_type: 1 })"
                   data-controller="se-share-sheet"
                   data-se-share-sheet-title="Share a link to this question"
                   data-se-share-sheet-subtitle=""
                   data-se-share-sheet-post-type="question"
                   data-se-share-sheet-social="facebook twitter devto"
                   data-se-share-sheet-location="1"
                   data-se-share-sheet-license-url="https%3a%2f%2fcreativecommons.org%2flicenses%2fby-sa%2f3.0%2f"
                   data-se-share-sheet-license-name="CC BY-SA 3.0"
                   data-s-popover-placement="bottom-start">Share</a>
            </div>
    
    
                    <div class="grid--cell">
                        <a href="/posts/43381154/edit" class="js-suggest-edit-post js-gps-track" data-gps-track="post.click({ item: 6, priv: 0, post_type: 1 })" title="">Improve this question</a>
                    </div>
    
    
            <div class="grid--cell">
                <button type="button"
                        id="btnFollowPost-43381154" class="s-btn s-btn__link js-follow-post js-follow-question js-gps-track"
                        data-gps-track="post.click({ item: 14, priv: 0, post_type: 1 })"
                        data-controller="s-tooltip " data-s-tooltip-placement="bottom"
                        data-s-popover-placement="bottom" aria-controls=""
                        title="Follow this question to receive notifications">
                    Follow
                </button>
            </div>
    
    
    
    
        </div>
        <div class="js-menu-popup-container"></div>
    </div>
    
                </div>
    
                    <div class="post-signature grid--cell">
    <div class="user-info ">
        <div class="user-action-time">
            <a href="/posts/43381154/revisions" title="show all edits to this post"
                             class="js-gps-track"
                             data-gps-track="post.click({ item: 4, priv: 0, post_type: 1 })">edited <span title="2017-04-13 04:57:40Z" class="relativetime">Apr 13 '17 at 4:57</span></a>
        </div>
        <div class="user-gravatar32">
            <a href="/users/7023590/mariand"><div class="gravatar-wrapper-32"><img src="https://www.gravatar.com/avatar/82052fbf0815ce82c4eded8fd941be14?s=32&d=identicon&r=PG&f=1" alt="" width="32" height="32" class="bar-sm"></div></a>
        </div>
        <div class="user-details">
            <a href="/users/7023590/mariand">MarianD</a>
            <div class="-flair">
                <span class="reputation-score" title="reputation score " dir="ltr">9,436</span><span title="8 gold badges" aria-hidden="true"><span class="badge1"></span><span class="badgecount">8</span></span><span class="v-visible-sr">8 gold badges</span><span title="27 silver badges" aria-hidden="true"><span class="badge2"></span><span class="badgecount">27</span></span><span class="v-visible-sr">27 silver badges</span><span title="42 bronze badges" aria-hidden="true"><span class="badge3"></span><span class="badgecount">42</span></span><span class="v-visible-sr">42 bronze badges</span>
            </div>
        </div>
    </div>
                    </div>
                <div class="post-signature owner grid--cell">
                    <div class="user-info ">
        <div class="user-action-time">
            asked <span title="2017-04-12 23:21:33Z" class="relativetime">Apr 12 '17 at 23:21</span>
        </div>
        <div class="user-gravatar32">
            <a href="/users/7859169/tps-kisan"><div class="gravatar-wrapper-32"><img src="https://lh5.googleusercontent.com/-XJQyTk0vTCE/AAAAAAAAAAI/AAAAAAAAABc/mJ4AyT8gxCY/photo.jpg?sz=32" alt="" width="32" height="32" class="bar-sm"></div></a>
        </div>
        <div class="user-details" itemprop="author" itemscope itemtype="http://schema.org/Person">
            <a href="/users/7859169/tps-kisan">TPS kisan</a><span class="d-none" itemprop="name">TPS kisan</span>
            <div class="-flair">
                <span class="reputation-score" title="reputation score " dir="ltr">119</span><span title="5 bronze badges" aria-hidden="true"><span class="badge3"></span><span class="badgecount">5</span></span><span class="v-visible-sr">5 bronze badges</span>
            </div>
        </div>
    </div>
    
    
                </div>
            </div>
        </div>
        
    </div>
    
    
    
    
                <span class="d-none" itemprop="commentCount">13</span> 
        <div class="post-layout--right js-post-comments-component">
            <div id="comments-43381154" class="comments js-comments-container bt bc-black-075 mt12 " data-post-id="43381154" data-min-length="15">
                <ul class="comments-list js-comments-list"
                        data-remaining-comments-count="8"
                        data-canpost="false"
                        data-cansee="true"
                        data-comments-unavailable="false"
                        data-addlink-disabled="true">
    
                            <li id="comment-73824447" class="comment js-comment " data-comment-id="73824447" data-comment-owner-id="259725" data-comment-score="1">
            <div class="js-comment-actions comment-actions">
                <div class="comment-score js-comment-edit-hide">
                        <span title="number of 'useful comment' votes received"
                                class="cool">1</span>
                </div>
            </div>
            <div class="comment-text  js-comment-text-and-form">
                <div class="comment-body js-comment-edit-hide">
                    
                    <span class="comment-copy">Possible duplicate of <a href="http://stackoverflow.com/questions/6597224/how-to-hide-html-source-disable-right-click-and-text-copy">How to hide html source & disable right click and text copy?</a></span>
                    
    – <a href="/users/259725/adam"
                           title="1,395 reputation"
                           class="comment-user">Adam</a>
                    <span class="comment-date" dir="ltr"><span title="2017-04-12 23:27:39Z, License: CC BY-SA 3.0" class="relativetime-clean">Apr 12 '17 at 23:27</span></span>
                </div>
            </div>
        </li>
        <li id="comment-73824456" class="comment js-comment " data-comment-id="73824456" data-comment-owner-id="1491895" data-comment-score="0">
            <div class="js-comment-actions comment-actions">
                <div class="comment-score js-comment-edit-hide">
                </div>
            </div>
            <div class="comment-text  js-comment-text-and-form">
                <div class="comment-body js-comment-edit-hide">
                    
                    <span class="comment-copy">@Adam No, it's not the same as that site.</span>
                    
    – <a href="/users/1491895/barmar"
                           title="583,686 reputation"
                           class="comment-user">Barmar</a>
                    <span class="comment-date" dir="ltr"><span title="2017-04-12 23:28:21Z, License: CC BY-SA 3.0" class="relativetime-clean">Apr 12 '17 at 23:28</span></span>
                </div>
            </div>
        </li>
        <li id="comment-73824474" class="comment js-comment " data-comment-id="73824474" data-comment-owner-id="5752562" data-comment-score="0">
            <div class="js-comment-actions comment-actions">
                <div class="comment-score js-comment-edit-hide">
                </div>
            </div>
            <div class="comment-text  js-comment-text-and-form">
                <div class="comment-body js-comment-edit-hide">
                    
                    <span class="comment-copy">Press F12 to open your console. Use IE11 to fool their script.</span>
                    
    – <a href="/users/5752562/yezzz"
                           title="2,810 reputation"
                           class="comment-user">yezzz</a>
                    <span class="comment-date" dir="ltr"><span title="2017-04-12 23:28:59Z, License: CC BY-SA 3.0" class="relativetime-clean">Apr 12 '17 at 23:28</span></span>
                </div>
            </div>
        </li>
        <li id="comment-73824527" class="comment js-comment " data-comment-id="73824527" data-comment-owner-id="7859169" data-comment-score="0">
            <div class="js-comment-actions comment-actions">
                <div class="comment-score js-comment-edit-hide">
                </div>
            </div>
            <div class="comment-text  js-comment-text-and-form">
                <div class="comment-body js-comment-edit-hide">
                    
                    <span class="comment-copy">what about if i goto More Tools>Developers Tools or (ctrl+shift+i) option on chrome this website redirect to another page so that no one can see the code of homepage. How to do it then?</span>
                    
    – <a href="/users/7859169/tps-kisan"
                           title="119 reputation"
                           class="comment-user owner">TPS kisan</a>
                    <span class="comment-date" dir="ltr"><span title="2017-04-12 23:32:01Z, License: CC BY-SA 3.0" class="relativetime-clean">Apr 12 '17 at 23:32</span></span>
                </div>
            </div>
        </li>
        <li id="comment-73824528" class="comment js-comment " data-comment-id="73824528" data-comment-owner-id="1491895" data-comment-score="0">
            <div class="js-comment-actions comment-actions">
                <div class="comment-score js-comment-edit-hide">
                </div>
            </div>
            <div class="comment-text  js-comment-text-and-form">
                <div class="comment-body js-comment-edit-hide">
                    
                    <span class="comment-copy">The site somehow detects when you open the console, and the source changes.</span>
                    
    – <a href="/users/1491895/barmar"
                           title="583,686 reputation"
                           class="comment-user">Barmar</a>
                    <span class="comment-date" dir="ltr"><span title="2017-04-12 23:32:10Z, License: CC BY-SA 3.0" class="relativetime-clean">Apr 12 '17 at 23:32</span></span>
                </div>
            </div>
        </li>
    
                </ul>
    	    </div>
    
            <div id="comments-link-43381154" data-rep=50 data-anon=true>
                        <a class="js-add-link comments-link dno" title="Use comments to ask for more information or suggest improvements. Avoid answering questions in comments."  href="#" role="button"></a>
                    <span class="js-link-separator dno"> | </span>
                <a class="js-show-link comments-link " title="Expand to show all comments on this post" href=# onclick="" role="button">Show <b>8</b> more comments</a>
            </div>         
        </div>
        </div>
    </div>
    
    
    <div class="js-zone-container zone-container-responsive">
        <div id="dfp-isb" class="everyonelovesstackoverflow everyoneloves__inline-sidebar mx-auto"></div>
        <div class="js-report-ad-button-container mx-auto" style="width: 300px"></div>
    </div>
    
                    <div id="answers">
    
                        <a name="tab-top"></a>
                        <div id="answers-header">
                            <div class="answers-subheader grid ai-center mb8">
                                <div class="grid--cell fl1">
                                    <h2 class="mb0" data-answercount="8">
                                            8 Answers
                                        <span style="display:none;" itemprop="answerCount">8</span>
                                    </h2>
                                </div>
                                <div class="grid--cell">
                                    <div class=" grid s-btn-group js-filter-btn">
            <a class="grid--cell s-btn s-btn__muted s-btn__outlined" href="/questions/43381154/how-do-i-able-to-see-the-source-code-of-this-website-www-samy-pl?answertab=active#tab-top" data-nav-xhref="" title="Answers with the latest activity first" data-value="active" data-shortcut="A">
                Active</a>
            <a class="grid--cell s-btn s-btn__muted s-btn__outlined" href="/questions/43381154/how-do-i-able-to-see-the-source-code-of-this-website-www-samy-pl?answertab=oldest#tab-top" data-nav-xhref="" title="Answers in the order they were provided" data-value="oldest" data-shortcut="O">
                Oldest</a>
            <a class="youarehere is-selected grid--cell s-btn s-btn__muted s-btn__outlined" href="/questions/43381154/how-do-i-able-to-see-the-source-code-of-this-website-www-samy-pl?answertab=votes#tab-top" data-nav-xhref="" title="Answers with the highest score first" data-value="votes" data-shortcut="V">
                Votes</a>
    </div>
    
                                </div>
                            </div>
                                
                        </div>
    
    
                                              
    <a name="43381381"></a>
    <div id="answer-43381381" class="answer" data-answerid="43381381" data-ownerid="1095581" data-score="4" itemprop="suggestedAnswer" itemscope itemtype="https://schema.org/Answer">
        <div class="post-layout">
            <div class="votecell post-layout--left">
                <div class="js-voting-container grid jc-center fd-column ai-stretch gs4 fc-black-200" data-post-id="43381381">
            <button class="js-vote-up-btn grid--cell s-btn s-btn__unset c-pointer" data-controller="s-tooltip" data-s-tooltip-placement="right" title="This answer is useful" aria-pressed="false" aria-label="Up vote" data-selected-classes="fc-theme-primary"><svg aria-hidden="true" class="m0 svg-icon iconArrowUpLg" width="36" height="36" viewBox="0 0 36 36"><path d="M2 26h32L18 10 2 26z"/></svg></button>
            <div class="js-vote-count grid--cell fc-black-500 fs-title grid fd-column ai-center" itemprop="upvoteCount" data-value="4">4</div>
            <button class="js-vote-down-btn grid--cell s-btn s-btn__unset c-pointer" data-controller="s-tooltip" data-s-tooltip-placement="right" title="This answer is not useful" aria-pressed="false" aria-label="Down vote" data-selected-classes="fc-theme-primary"><svg aria-hidden="true" class="m0 svg-icon iconArrowDownLg" width="36" height="36" viewBox="0 0 36 36"><path d="M2 10h32L18 26 2 10z"/></svg></button>
    
        
                <div class="js-accepted-answer-indicator grid--cell fc-green-500 py6 mtn8 d-none" data-s-tooltip-placement="right" title="Loading when this answer was accepted…" tabindex="0" role="note" aria-label="Accepted">
                    <div class="ta-center">
                        <svg aria-hidden="true" class="svg-icon iconCheckmarkLg" width="36" height="36" viewBox="0 0 36 36"><path d="M6 14l8 8L30 6v8L14 30l-8-8v-8z"/></svg>
                    </div>
                </div>
    
        
            <a class="js-post-issue grid--cell s-btn s-btn__unset c-pointer py6 mx-auto" href="/posts/43381381/timeline" data-shortcut="T" data-ks-title="timeline" data-controller="s-tooltip" data-s-tooltip-placement="right" title="Show activity on this post." aria-label="Timeline"><svg aria-hidden="true" class="mln2 mr0 svg-icon iconHistory" width="19" height="18" viewBox="0 0 19 18"><path d="M3 9a8 8 0 113.73 6.77L8.2 14.3A6 6 0 105 9l3.01-.01-4 4-4-4h3L3 9zm7-4h1.01L11 9.36l3.22 2.1-.6.93L10 10V5z"/></svg></a>
    
    </div>
            </div>
    
            
    
    <div class="answercell post-layout--right">
        
        <div class="s-prose js-post-body" itemprop="text">
    <p>You could always save the website.
    In chrome open the menu -> more tools -> Save page as..</p>
    
    <p>You should be able to run the html file in chrome, and use Dev Tools without it blocking you.</p>
        </div>
        <div class="mt24">
            <div class="grid fw-wrap ai-start jc-end gs8 gsy">
                <time itemprop="dateCreated" datetime="2017-04-12T23:49:14"></time>
                <div class="grid--cell mr16" style="flex: 1 1 100px;">
                    
    
    <div class="js-post-menu pt2" data-post-id="43381381">
        <div class="grid d-flex gs8 s-anchors s-anchors__muted fw-wrap">
    
            <div class="grid--cell">
                <a href="/a/43381381"
                   rel="nofollow"
                   itemprop="url"
                   class="js-share-link js-gps-track"
                   title="Short permalink to this answer"
                   data-gps-track="post.click({ item: 2, priv: 0, post_type: 2 })"
                   data-controller="se-share-sheet"
                   data-se-share-sheet-title="Share a link to this answer"
                   data-se-share-sheet-subtitle=""
                   data-se-share-sheet-post-type="answer"
                   data-se-share-sheet-social="facebook twitter devto"
                   data-se-share-sheet-location="2"
                   data-se-share-sheet-license-url="https%3a%2f%2fcreativecommons.org%2flicenses%2fby-sa%2f3.0%2f"
                   data-se-share-sheet-license-name="CC BY-SA 3.0"
                   data-s-popover-placement="bottom-start">Share</a>
            </div>
    
    
                    <div class="grid--cell">
                        <a href="/posts/43381381/edit" class="js-suggest-edit-post js-gps-track" data-gps-track="post.click({ item: 6, priv: 0, post_type: 2 })" title="">Improve this answer</a>
                    </div>
    
    
            <div class="grid--cell">
                <button type="button"
                        id="btnFollowPost-43381381" class="s-btn s-btn__link js-follow-post js-follow-answer js-gps-track"
                        data-gps-track="post.click({ item: 14, priv: 0, post_type: 2 })"
                        data-controller="s-tooltip " data-s-tooltip-placement="bottom"
                        data-s-popover-placement="bottom" aria-controls=""
                        title="Follow this answer to receive notifications">
                    Follow
                </button>
            </div>
    
    
    
    
        </div>
        <div class="js-menu-popup-container"></div>
    </div>
    
                </div>
    
    
                <div class="post-signature grid--cell fl0">
                    <div class="user-info ">
        <div class="user-action-time">
            answered <span title="2017-04-12 23:49:14Z" class="relativetime">Apr 12 '17 at 23:49</span>
        </div>
        <div class="user-gravatar32">
            <a href="/users/1095581/fio"><div class="gravatar-wrapper-32"><img src="https://www.gravatar.com/avatar/a788527a8ba89f41b3885e763a283c91?s=32&d=identicon&r=PG" alt="" width="32" height="32" class="bar-sm"></div></a>
        </div>
        <div class="user-details" itemprop="author" itemscope itemtype="http://schema.org/Person">
            <a href="/users/1095581/fio">Fio</a><span class="d-none" itemprop="name">Fio</span>
            <div class="-flair">
                <span class="reputation-score" title="reputation score " dir="ltr">79</span><span title="7 bronze badges" aria-hidden="true"><span class="badge3"></span><span class="badgecount">7</span></span><span class="v-visible-sr">7 bronze badges</span>
            </div>
        </div>
    </div>
    
    
                </div>
            </div>
        </div>
        
    </div>
    
    
    
    
                <span class="d-none" itemprop="commentCount">2</span> 
        <div class="post-layout--right js-post-comments-component">
            <div id="comments-43381381" class="comments js-comments-container bt bc-black-075 mt12 " data-post-id="43381381" data-min-length="15">
                <ul class="comments-list js-comments-list"
                        data-remaining-comments-count="0"
                        data-canpost="false"
                        data-cansee="true"
                        data-comments-unavailable="false"
                        data-addlink-disabled="true">
    
                            <li id="comment-73824846" class="comment js-comment " data-comment-id="73824846" data-comment-owner-id="6563293" data-comment-score="0">
            <div class="js-comment-actions comment-actions">
                <div class="comment-score js-comment-edit-hide">
                </div>
            </div>
            <div class="comment-text  js-comment-text-and-form">
                <div class="comment-body js-comment-edit-hide">
                    
                    <span class="comment-copy">It would still be blocked because the code is what is being downloaded and is what is blocking it. But you WILL be able to open it in a text editor to view the code.</span>
                    
    – <a href="/users/6563293/cameron"
                           title="949 reputation"
                           class="comment-user">Cameron</a>
                    <span class="comment-date" dir="ltr"><span title="2017-04-12 23:51:04Z, License: CC BY-SA 3.0" class="relativetime-clean">Apr 12 '17 at 23:51</span></span>
                </div>
            </div>
        </li>
        <li id="comment-73825043" class="comment js-comment " data-comment-id="73825043" data-comment-owner-id="6563293" data-comment-score="1">
            <div class="js-comment-actions comment-actions">
                <div class="comment-score js-comment-edit-hide">
                        <span title="number of 'useful comment' votes received"
                                class="cool">1</span>
                </div>
            </div>
            <div class="comment-text  js-comment-text-and-form">
                <div class="comment-body js-comment-edit-hide">
                    
                    <span class="comment-copy">Yeah opening the file to see the code actually works!</span>
                    
    – <a href="/users/6563293/cameron"
                           title="949 reputation"
                           class="comment-user">Cameron</a>
                    <span class="comment-date" dir="ltr"><span title="2017-04-13 00:03:47Z, License: CC BY-SA 3.0" class="relativetime-clean">Apr 13 '17 at 0:03</span></span>
                </div>
            </div>
        </li>
    
                </ul>
    	    </div>
    
            <div id="comments-link-43381381" data-rep=50 data-anon=true>
                        <a class="js-add-link comments-link disabled-link" title="Use comments to ask for more information or suggest improvements. Avoid comments like “+1” or “thanks”."  href="#" role="button">Add a comment</a>
                    <span class="js-link-separator dno"> | </span>
                <a class="js-show-link comments-link dno" title="Expand to show all comments on this post" href=# onclick="" role="button"></a>
            </div>         
        </div>
        </div>
    </div>
    <div class="js-zone-container zone-container-main">
        <div id="dfp-mlb" class="everyonelovesstackoverflow everyoneloves__mid-leaderboard everyoneloves__leaderboard"></div>
        <div class="js-report-ad-button-container " style="width: 728px"></div>
    </div>
                                              
    <a name="43381541"></a>
    <div id="answer-43381541" class="answer" data-answerid="43381541" data-ownerid="7024476" data-score="3" itemprop="suggestedAnswer" itemscope itemtype="https://schema.org/Answer">
        <div class="post-layout">
            <div class="votecell post-layout--left">
                <div class="js-voting-container grid jc-center fd-column ai-stretch gs4 fc-black-200" data-post-id="43381541">
            <button class="js-vote-up-btn grid--cell s-btn s-btn__unset c-pointer" data-controller="s-tooltip" data-s-tooltip-placement="right" title="This answer is useful" aria-pressed="false" aria-label="Up vote" data-selected-classes="fc-theme-primary"><svg aria-hidden="true" class="m0 svg-icon iconArrowUpLg" width="36" height="36" viewBox="0 0 36 36"><path d="M2 26h32L18 10 2 26z"/></svg></button>
            <div class="js-vote-count grid--cell fc-black-500 fs-title grid fd-column ai-center" itemprop="upvoteCount" data-value="3">3</div>
            <button class="js-vote-down-btn grid--cell s-btn s-btn__unset c-pointer" data-controller="s-tooltip" data-s-tooltip-placement="right" title="This answer is not useful" aria-pressed="false" aria-label="Down vote" data-selected-classes="fc-theme-primary"><svg aria-hidden="true" class="m0 svg-icon iconArrowDownLg" width="36" height="36" viewBox="0 0 36 36"><path d="M2 10h32L18 26 2 10z"/></svg></button>
    
        
                <div class="js-accepted-answer-indicator grid--cell fc-green-500 py6 mtn8 d-none" data-s-tooltip-placement="right" title="Loading when this answer was accepted…" tabindex="0" role="note" aria-label="Accepted">
                    <div class="ta-center">
                        <svg aria-hidden="true" class="svg-icon iconCheckmarkLg" width="36" height="36" viewBox="0 0 36 36"><path d="M6 14l8 8L30 6v8L14 30l-8-8v-8z"/></svg>
                    </div>
                </div>
    
        
            <a class="js-post-issue grid--cell s-btn s-btn__unset c-pointer py6 mx-auto" href="/posts/43381541/timeline" data-shortcut="T" data-ks-title="timeline" data-controller="s-tooltip" data-s-tooltip-placement="right" title="Show activity on this post." aria-label="Timeline"><svg aria-hidden="true" class="mln2 mr0 svg-icon iconHistory" width="19" height="18" viewBox="0 0 19 18"><path d="M3 9a8 8 0 113.73 6.77L8.2 14.3A6 6 0 105 9l3.01-.01-4 4-4-4h3L3 9zm7-4h1.01L11 9.36l3.22 2.1-.6.93L10 10V5z"/></svg></a>
    
    </div>
            </div>
    
            
    
    <div class="answercell post-layout--right">
        
        <div class="s-prose js-post-body" itemprop="text">
    <p>There is a <code>console</code> "clearing" API that is mentioned in <a href="https://stackoverflow.com/a/31261830/7024476">this answer</a>.</p>
    
    <p>Here's what I think is happening:</p>
    
    <p>First, I think Google Chrome DevTools emits an event when it is open/launched.
    The page author listens for this 'launch' event with the following handler logic:</p>
    
    <ol>
    <li>Run a <code>chrome.devtools.inspectedWindow.eval</code> <a href="https://developer.chrome.com/extensions/devtools_inspectedWindow" rel="nofollow noreferrer">DevTools inspection API</a> </li>
    <li>Set <code>document.body.innerHTML</code> to the winky-face div</li>
    <li>Also, a <a href="https://developer.mozilla.org/en-US/docs/Web/API/WindowOrWorkerGlobalScope/setInterval" rel="nofollow noreferrer"><code>setInterval</code></a> that executes a <code>console.clear()</code> before the <code>console.log()</code> of "no source for you".</li>
    </ol>
    
    <p>I also inspected the EventListeners tab of Chrome DevTools while reading the "no source for you"-page.</p>
    
    <p>Unfortunately, the two event listeners you can view don't seem to do <em>anything useful</em>: </p>
    
    <ul>
    <li>one returns <code>false</code></li>
    <li>the other wires together forms and their associated submit actions.</li>
    </ul>
    
    <p>Hope this helped</p>
        </div>
        <div class="mt24">
            <div class="grid fw-wrap ai-start jc-end gs8 gsy">
                <time itemprop="dateCreated" datetime="2017-04-13T00:08:05"></time>
                <div class="grid--cell mr16" style="flex: 1 1 100px;">
                    
    
    <div class="js-post-menu pt2" data-post-id="43381541">
        <div class="grid d-flex gs8 s-anchors s-anchors__muted fw-wrap">
    
            <div class="grid--cell">
                <a href="/a/43381541"
                   rel="nofollow"
                   itemprop="url"
                   class="js-share-link js-gps-track"
                   title="Short permalink to this answer"
                   data-gps-track="post.click({ item: 2, priv: 0, post_type: 2 })"
                   data-controller="se-share-sheet"
                   data-se-share-sheet-title="Share a link to this answer"
                   data-se-share-sheet-subtitle=""
                   data-se-share-sheet-post-type="answer"
                   data-se-share-sheet-social="facebook twitter devto"
                   data-se-share-sheet-location="2"
                   data-se-share-sheet-license-url="https%3a%2f%2fcreativecommons.org%2flicenses%2fby-sa%2f3.0%2f"
                   data-se-share-sheet-license-name="CC BY-SA 3.0"
                   data-s-popover-placement="bottom-start">Share</a>
            </div>
    
    
                    <div class="grid--cell">
                        <a href="/posts/43381541/edit" class="js-suggest-edit-post js-gps-track" data-gps-track="post.click({ item: 6, priv: 0, post_type: 2 })" title="">Improve this answer</a>
                    </div>
    
    
            <div class="grid--cell">
                <button type="button"
                        id="btnFollowPost-43381541" class="s-btn s-btn__link js-follow-post js-follow-answer js-gps-track"
                        data-gps-track="post.click({ item: 14, priv: 0, post_type: 2 })"
                        data-controller="s-tooltip " data-s-tooltip-placement="bottom"
                        data-s-popover-placement="bottom" aria-controls=""
                        title="Follow this answer to receive notifications">
                    Follow
                </button>
            </div>
    
    
    
    
        </div>
        <div class="js-menu-popup-container"></div>
    </div>
    
                </div>
                <div class="post-signature grid--cell fl0">
    <div class="user-info user-hover">
        <div class="user-action-time">
            <a href="/posts/43381541/revisions" title="show all edits to this post"
                             class="js-gps-track"
                             data-gps-track="post.click({ item: 4, priv: 0, post_type: 2 })">edited <span title="2017-05-23 11:55:03Z" class="relativetime">May 23 '17 at 11:55</span></a>
        </div>
        <div class="user-gravatar32">
            <a href="/users/-1/community"><div class="gravatar-wrapper-32"><img src="https://www.gravatar.com/avatar/a007be5a61f6aa8f3e85ae2fc18dd66e?s=32&d=identicon&r=PG" alt="" width="32" height="32" class="bar-sm"></div></a>
        </div>
        <div class="user-details">
            <a href="/users/-1/community">Community</a><span class="mod-flair " title="moderator">♦</span>
            <div class="-flair">
                <span class="reputation-score" title="reputation score " dir="ltr">1</span><span title="1 silver badge" aria-hidden="true"><span class="badge2"></span><span class="badgecount">1</span></span><span class="v-visible-sr">1 silver badge</span>
            </div>
        </div>
    </div>
                </div>
    
    
                <div class="post-signature grid--cell fl0">
                    <div class="user-info user-hover">
        <div class="user-action-time">
            answered <span title="2017-04-13 00:08:05Z" class="relativetime">Apr 13 '17 at 0:08</span>
        </div>
        <div class="user-gravatar32">
            <a href="/users/7024476/stephenjfox"><div class="gravatar-wrapper-32"><img src="https://i.stack.imgur.com/rPF53.jpg?s=32&g=1" alt="" width="32" height="32" class="bar-sm"></div></a>
        </div>
        <div class="user-details" itemprop="author" itemscope itemtype="http://schema.org/Person">
            <a href="/users/7024476/stephenjfox">stephenjfox</a><span class="d-none" itemprop="name">stephenjfox</span>
            <div class="-flair">
                <span class="reputation-score" title="reputation score " dir="ltr">73</span><span title="3 silver badges" aria-hidden="true"><span class="badge2"></span><span class="badgecount">3</span></span><span class="v-visible-sr">3 silver badges</span><span title="8 bronze badges" aria-hidden="true"><span class="badge3"></span><span class="badgecount">8</span></span><span class="v-visible-sr">8 bronze badges</span>
            </div>
        </div>
    </div>
    
    
                </div>
            </div>
        </div>
        
    </div>
    
    
    
    
                <span class="d-none" itemprop="commentCount">1</span> 
        <div class="post-layout--right js-post-comments-component">
            <div id="comments-43381541" class="comments js-comments-container bt bc-black-075 mt12 " data-post-id="43381541" data-min-length="15">
                <ul class="comments-list js-comments-list"
                        data-remaining-comments-count="0"
                        data-canpost="false"
                        data-cansee="true"
                        data-comments-unavailable="false"
                        data-addlink-disabled="true">
    
                            <li id="comment-73842901" class="comment js-comment " data-comment-id="73842901" data-comment-owner-id="5752562" data-comment-score="1">
            <div class="js-comment-actions comment-actions">
                <div class="comment-score js-comment-edit-hide">
                        <span title="number of 'useful comment' votes received"
                                class="cool">1</span>
                </div>
            </div>
            <div class="comment-text  js-comment-text-and-form">
                <div class="comment-body js-comment-edit-hide">
                    
                    <span class="comment-copy">So they also clear their script tags, and remove any event listeners and/or callbacks that mess with the console.</span>
                    
    – <a href="/users/5752562/yezzz"
                           title="2,810 reputation"
                           class="comment-user">yezzz</a>
                    <span class="comment-date" dir="ltr"><span title="2017-04-13 11:29:17Z, License: CC BY-SA 3.0" class="relativetime-clean">Apr 13 '17 at 11:29</span></span>
                </div>
            </div>
        </li>
    
                </ul>
    	    </div>
    
            <div id="comments-link-43381541" data-rep=50 data-anon=true>
                        <a class="js-add-link comments-link disabled-link" title="Use comments to ask for more information or suggest improvements. Avoid comments like “+1” or “thanks”."  href="#" role="button">Add a comment</a>
                    <span class="js-link-separator dno"> | </span>
                <a class="js-show-link comments-link dno" title="Expand to show all comments on this post" href=# onclick="" role="button"></a>
            </div>         
        </div>
        </div>
    </div>
                                              
    <a name="47629515"></a>
    <div id="answer-47629515" class="answer" data-answerid="47629515" data-ownerid="766521" data-score="2" itemprop="suggestedAnswer" itemscope itemtype="https://schema.org/Answer">
        <div class="post-layout">
            <div class="votecell post-layout--left">
                <div class="js-voting-container grid jc-center fd-column ai-stretch gs4 fc-black-200" data-post-id="47629515">
            <button class="js-vote-up-btn grid--cell s-btn s-btn__unset c-pointer" data-controller="s-tooltip" data-s-tooltip-placement="right" title="This answer is useful" aria-pressed="false" aria-label="Up vote" data-selected-classes="fc-theme-primary"><svg aria-hidden="true" class="m0 svg-icon iconArrowUpLg" width="36" height="36" viewBox="0 0 36 36"><path d="M2 26h32L18 10 2 26z"/></svg></button>
            <div class="js-vote-count grid--cell fc-black-500 fs-title grid fd-column ai-center" itemprop="upvoteCount" data-value="2">2</div>
            <button class="js-vote-down-btn grid--cell s-btn s-btn__unset c-pointer" data-controller="s-tooltip" data-s-tooltip-placement="right" title="This answer is not useful" aria-pressed="false" aria-label="Down vote" data-selected-classes="fc-theme-primary"><svg aria-hidden="true" class="m0 svg-icon iconArrowDownLg" width="36" height="36" viewBox="0 0 36 36"><path d="M2 10h32L18 26 2 10z"/></svg></button>
    
        
                <div class="js-accepted-answer-indicator grid--cell fc-green-500 py6 mtn8 d-none" data-s-tooltip-placement="right" title="Loading when this answer was accepted…" tabindex="0" role="note" aria-label="Accepted">
                    <div class="ta-center">
                        <svg aria-hidden="true" class="svg-icon iconCheckmarkLg" width="36" height="36" viewBox="0 0 36 36"><path d="M6 14l8 8L30 6v8L14 30l-8-8v-8z"/></svg>
                    </div>
                </div>
    
        
            <a class="js-post-issue grid--cell s-btn s-btn__unset c-pointer py6 mx-auto" href="/posts/47629515/timeline" data-shortcut="T" data-ks-title="timeline" data-controller="s-tooltip" data-s-tooltip-placement="right" title="Show activity on this post." aria-label="Timeline"><svg aria-hidden="true" class="mln2 mr0 svg-icon iconHistory" width="19" height="18" viewBox="0 0 19 18"><path d="M3 9a8 8 0 113.73 6.77L8.2 14.3A6 6 0 105 9l3.01-.01-4 4-4-4h3L3 9zm7-4h1.01L11 9.36l3.22 2.1-.6.93L10 10V5z"/></svg></a>
    
    </div>
            </div>
    
            
    
    <div class="answercell post-layout--right">
        
        <div class="s-prose js-post-body" itemprop="text">
    <p>hahaha, this is a good one, you should not expect less from samy!</p>
    
    <p>Now I see why people think it is the wrong answer (the source get replaced if you do otherwise) </p>
    
    <p>1.Navigate to site with inspector open there you end up with "No source for you! You found easter egg #7."</p>
    
    <p>2.and then check the source to find Easter egg #2:</p>
    
    <pre><code>view-source:https://samy.pl
    </code></pre>
    
    <p>you will see:</p>
    
    <pre><code>/*
    No source for you! Easter egg #2
    *// AFTER SOME LONG WHITE SPACES HERE  /.source.replace(/.{7}/g,function(w){document.write(String.fromCharCode(parseInt(w.replace(/ /g,'0').replace(/   /g,'1'),2)))});
    </code></pre>
    
    <p>This is the way he loads the site(by injecting js), by replacing two type of white spaces for 1 and 0 and parsing that to char code by some regex and then that to string of course :D</p>
    
    <p><a href="https://de.wikipedia.org/wiki/Whitespace_(Programmiersprache)" rel="nofollow noreferrer">https://de.wikipedia.org/wiki/Whitespace_(Programmiersprache)</a></p>
    
    <p>If you paste that part to a js console, you will get the tags that loads the page.</p>
    
    <p><a href="https://i.stack.imgur.com/aQ6wj.png" rel="nofollow noreferrer"><img src="https://i.stack.imgur.com/aQ6wj.png" alt="enter image description here"></a></p>
    
    <p>ps. to see the rendered source, just load the page normaly and hit F12 after disabling javascript.</p>
    
    <p>Have fun!</p>
        </div>
        <div class="mt24">
            <div class="grid fw-wrap ai-start jc-end gs8 gsy">
                <time itemprop="dateCreated" datetime="2017-12-04T08:48:22"></time>
                <div class="grid--cell mr16" style="flex: 1 1 100px;">
                    
    
    <div class="js-post-menu pt2" data-post-id="47629515">
        <div class="grid d-flex gs8 s-anchors s-anchors__muted fw-wrap">
    
            <div class="grid--cell">
                <a href="/a/47629515"
                   rel="nofollow"
                   itemprop="url"
                   class="js-share-link js-gps-track"
                   title="Short permalink to this answer"
                   data-gps-track="post.click({ item: 2, priv: 0, post_type: 2 })"
                   data-controller="se-share-sheet"
                   data-se-share-sheet-title="Share a link to this answer"
                   data-se-share-sheet-subtitle=""
                   data-se-share-sheet-post-type="answer"
                   data-se-share-sheet-social="facebook twitter devto"
                   data-se-share-sheet-location="2"
                   data-se-share-sheet-license-url="https%3a%2f%2fcreativecommons.org%2flicenses%2fby-sa%2f3.0%2f"
                   data-se-share-sheet-license-name="CC BY-SA 3.0"
                   data-s-popover-placement="bottom-start">Share</a>
            </div>
    
    
                    <div class="grid--cell">
                        <a href="/posts/47629515/edit" class="js-suggest-edit-post js-gps-track" data-gps-track="post.click({ item: 6, priv: 0, post_type: 2 })" title="">Improve this answer</a>
                    </div>
    
    
            <div class="grid--cell">
                <button type="button"
                        id="btnFollowPost-47629515" class="s-btn s-btn__link js-follow-post js-follow-answer js-gps-track"
                        data-gps-track="post.click({ item: 14, priv: 0, post_type: 2 })"
                        data-controller="s-tooltip " data-s-tooltip-placement="bottom"
                        data-s-popover-placement="bottom" aria-controls=""
                        title="Follow this answer to receive notifications">
                    Follow
                </button>
            </div>
    
    
    
    
        </div>
        <div class="js-menu-popup-container"></div>
    </div>
    
                </div>
                <div class="post-signature grid--cell fl0">
    <div class="user-info ">
        <div class="user-action-time">
            <a href="/posts/47629515/revisions" title="show all edits to this post"
                             class="js-gps-track"
                             data-gps-track="post.click({ item: 4, priv: 0, post_type: 2 })">edited <span title="2017-12-05 09:29:32Z" class="relativetime">Dec 5 '17 at 9:29</span></a>
        </div>
        <div class="user-gravatar32">
            
        </div>
        <div class="user-details">
            
            <div class="-flair">
                
            </div>
        </div>
    </div>
                </div>
    
    
                <div class="post-signature grid--cell fl0">
                    <div class="user-info user-hover">
        <div class="user-action-time">
            answered <span title="2017-12-04 08:48:22Z" class="relativetime">Dec 4 '17 at 8:48</span>
        </div>
        <div class="user-gravatar32">
            <a href="/users/766521/pabbasian"><div class="gravatar-wrapper-32"><img src="https://www.gravatar.com/avatar/83abbd03fbbdaad3441ddee7189632b6?s=32&d=identicon&r=PG" alt="" width="32" height="32" class="bar-sm"></div></a>
        </div>
        <div class="user-details" itemprop="author" itemscope itemtype="http://schema.org/Person">
            <a href="/users/766521/pabbasian">pabbasian</a><span class="d-none" itemprop="name">pabbasian</span>
            <div class="-flair">
                <span class="reputation-score" title="reputation score " dir="ltr">79</span><span title="11 bronze badges" aria-hidden="true"><span class="badge3"></span><span class="badgecount">11</span></span><span class="v-visible-sr">11 bronze badges</span>
            </div>
        </div>
    </div>
    
    
                </div>
            </div>
        </div>
        
    </div>
    
    
    
    
                <span class="d-none" itemprop="commentCount">2</span> 
        <div class="post-layout--right js-post-comments-component">
            <div id="comments-47629515" class="comments js-comments-container bt bc-black-075 mt12 " data-post-id="47629515" data-min-length="15">
                <ul class="comments-list js-comments-list"
                        data-remaining-comments-count="0"
                        data-canpost="false"
                        data-cansee="true"
                        data-comments-unavailable="false"
                        data-addlink-disabled="true">
    
                            <li id="comment-99885384" class="comment js-comment " data-comment-id="99885384" data-comment-owner-id="704894" data-comment-score="0">
            <div class="js-comment-actions comment-actions">
                <div class="comment-score js-comment-edit-hide">
                </div>
            </div>
            <div class="comment-text  js-comment-text-and-form">
                <div class="comment-body js-comment-edit-hide">
                    
                    <span class="comment-copy">This is no longer valid.</span>
                    
    – <a href="/users/704894/micha%c5%82-miszczyszyn"
                           title="8,721 reputation"
                           class="comment-user">Michał Miszczyszyn</a>
                    <span class="comment-date" dir="ltr"><span title="2019-06-18 22:26:26Z, License: CC BY-SA 4.0" class="relativetime-clean">Jun 18 '19 at 22:26</span></span>
                </div>
            </div>
        </li>
        <li id="comment-100061089" class="comment js-comment " data-comment-id="100061089" data-comment-owner-id="766521" data-comment-score="0">
            <div class="js-comment-actions comment-actions">
                <div class="comment-score js-comment-edit-hide">
                </div>
            </div>
            <div class="comment-text  js-comment-text-and-form">
                <div class="comment-body js-comment-edit-hide">
                    
                    <span class="comment-copy">"<!DOCTYPE html>  <html>  <head>  	<noscript>  	<meta http-equiv=\"refresh\" content=\"0;url=/code/?ns=1\" />  	</noscript>  	<script>  		var YOU_FOUND_THE_SOURCE_TO_SAMY_PL = \"you are awesome!\"  		var total_easter_eggs = 21  		/*  			Thanks for playing!  			There are currently 21 Easter Eggs on this site,  			hope you've enjoyed running into a few, and perhaps  			you can find a few more!    			Your internet friend, -samy  		*/  	</script>  	...</span>
                    
    – <a href="/users/766521/pabbasian"
                           title="79 reputation"
                           class="comment-user">pabbasian</a>
                    <span class="comment-date" dir="ltr"><span title="2019-06-25 10:17:25Z, License: CC BY-SA 4.0" class="relativetime-clean">Jun 25 '19 at 10:17</span></span>
                </div>
            </div>
        </li>
    
                </ul>
    	    </div>
    
            <div id="comments-link-47629515" data-rep=50 data-anon=true>
                        <a class="js-add-link comments-link disabled-link" title="Use comments to ask for more information or suggest improvements. Avoid comments like “+1” or “thanks”."  href="#" role="button">Add a comment</a>
                    <span class="js-link-separator dno"> | </span>
                <a class="js-show-link comments-link dno" title="Expand to show all comments on this post" href=# onclick="" role="button"></a>
            </div>         
        </div>
        </div>
    </div>
    <div class="js-zone-container zone-container-main">
        <div id="dfp-smlb" class="everyonelovesstackoverflow everyoneloves__mid-second-leaderboard everyoneloves__leaderboard"></div>
        <div class="js-report-ad-button-container " style="width: 728px"></div>
    </div>
                                              
    <a name="45537537"></a>
    <div id="answer-45537537" class="answer" data-answerid="45537537" data-ownerid="6565157" data-score="1" itemprop="suggestedAnswer" itemscope itemtype="https://schema.org/Answer">
        <div class="post-layout">
            <div class="votecell post-layout--left">
                <div class="js-voting-container grid jc-center fd-column ai-stretch gs4 fc-black-200" data-post-id="45537537">
            <button class="js-vote-up-btn grid--cell s-btn s-btn__unset c-pointer" data-controller="s-tooltip" data-s-tooltip-placement="right" title="This answer is useful" aria-pressed="false" aria-label="Up vote" data-selected-classes="fc-theme-primary"><svg aria-hidden="true" class="m0 svg-icon iconArrowUpLg" width="36" height="36" viewBox="0 0 36 36"><path d="M2 26h32L18 10 2 26z"/></svg></button>
            <div class="js-vote-count grid--cell fc-black-500 fs-title grid fd-column ai-center" itemprop="upvoteCount" data-value="1">1</div>
            <button class="js-vote-down-btn grid--cell s-btn s-btn__unset c-pointer" data-controller="s-tooltip" data-s-tooltip-placement="right" title="This answer is not useful" aria-pressed="false" aria-label="Down vote" data-selected-classes="fc-theme-primary"><svg aria-hidden="true" class="m0 svg-icon iconArrowDownLg" width="36" height="36" viewBox="0 0 36 36"><path d="M2 10h32L18 26 2 10z"/></svg></button>
    
        
                <div class="js-accepted-answer-indicator grid--cell fc-green-500 py6 mtn8 d-none" data-s-tooltip-placement="right" title="Loading when this answer was accepted…" tabindex="0" role="note" aria-label="Accepted">
                    <div class="ta-center">
                        <svg aria-hidden="true" class="svg-icon iconCheckmarkLg" width="36" height="36" viewBox="0 0 36 36"><path d="M6 14l8 8L30 6v8L14 30l-8-8v-8z"/></svg>
                    </div>
                </div>
    
        
            <a class="js-post-issue grid--cell s-btn s-btn__unset c-pointer py6 mx-auto" href="/posts/45537537/timeline" data-shortcut="T" data-ks-title="timeline" data-controller="s-tooltip" data-s-tooltip-placement="right" title="Show activity on this post." aria-label="Timeline"><svg aria-hidden="true" class="mln2 mr0 svg-icon iconHistory" width="19" height="18" viewBox="0 0 19 18"><path d="M3 9a8 8 0 113.73 6.77L8.2 14.3A6 6 0 105 9l3.01-.01-4 4-4-4h3L3 9zm7-4h1.01L11 9.36l3.22 2.1-.6.93L10 10V5z"/></svg></a>
    
    </div>
            </div>
    
            
    
    <div class="answercell post-layout--right">
        
        <div class="s-prose js-post-body" itemprop="text">
    <p>Create an <code>index.html</code> somewhere, containing an <code><iframe src='https://samy.pl/'></code>. Load it in your browser and then open devtools. You will see the source inside the iframe.</p>
        </div>
        <div class="mt24">
            <div class="grid fw-wrap ai-start jc-end gs8 gsy">
                <time itemprop="dateCreated" datetime="2017-08-06T23:17:40"></time>
                <div class="grid--cell mr16" style="flex: 1 1 100px;">
                    
    
    <div class="js-post-menu pt2" data-post-id="45537537">
        <div class="grid d-flex gs8 s-anchors s-anchors__muted fw-wrap">
    
            <div class="grid--cell">
                <a href="/a/45537537"
                   rel="nofollow"
                   itemprop="url"
                   class="js-share-link js-gps-track"
                   title="Short permalink to this answer"
                   data-gps-track="post.click({ item: 2, priv: 0, post_type: 2 })"
                   data-controller="se-share-sheet"
                   data-se-share-sheet-title="Share a link to this answer"
                   data-se-share-sheet-subtitle=""
                   data-se-share-sheet-post-type="answer"
                   data-se-share-sheet-social="facebook twitter devto"
                   data-se-share-sheet-location="2"
                   data-se-share-sheet-license-url="https%3a%2f%2fcreativecommons.org%2flicenses%2fby-sa%2f3.0%2f"
                   data-se-share-sheet-license-name="CC BY-SA 3.0"
                   data-s-popover-placement="bottom-start">Share</a>
            </div>
    
    
                    <div class="grid--cell">
                        <a href="/posts/45537537/edit" class="js-suggest-edit-post js-gps-track" data-gps-track="post.click({ item: 6, priv: 0, post_type: 2 })" title="">Improve this answer</a>
                    </div>
    
    
            <div class="grid--cell">
                <button type="button"
                        id="btnFollowPost-45537537" class="s-btn s-btn__link js-follow-post js-follow-answer js-gps-track"
                        data-gps-track="post.click({ item: 14, priv: 0, post_type: 2 })"
                        data-controller="s-tooltip " data-s-tooltip-placement="bottom"
                        data-s-popover-placement="bottom" aria-controls=""
                        title="Follow this answer to receive notifications">
                    Follow
                </button>
            </div>
    
    
    
    
        </div>
        <div class="js-menu-popup-container"></div>
    </div>
    
                </div>
    
    
                <div class="post-signature grid--cell fl0">
                    <div class="user-info ">
        <div class="user-action-time">
            answered <span title="2017-08-06 23:17:40Z" class="relativetime">Aug 6 '17 at 23:17</span>
        </div>
        <div class="user-gravatar32">
            <a href="/users/6565157/nadal-gonzalo-garc%c3%ada-zavala"><div class="gravatar-wrapper-32"><img src="https://www.gravatar.com/avatar/b3128ec73a08bcf7f36f47236c8ba9ef?s=32&d=identicon&r=PG&f=1" alt="" width="32" height="32" class="bar-sm"></div></a>
        </div>
        <div class="user-details" itemprop="author" itemscope itemtype="http://schema.org/Person">
            <a href="/users/6565157/nadal-gonzalo-garc%c3%ada-zavala">Nadal Gonzalo García Zavala</a><span class="d-none" itemprop="name">Nadal Gonzalo García Zavala</span>
            <div class="-flair">
                <span class="reputation-score" title="reputation score " dir="ltr">11</span><span title="1 bronze badge" aria-hidden="true"><span class="badge3"></span><span class="badgecount">1</span></span><span class="v-visible-sr">1 bronze badge</span>
            </div>
        </div>
    </div>
    
    
                </div>
            </div>
        </div>
        
    </div>
    
    
    
    
                <span class="d-none" itemprop="commentCount">1</span> 
        <div class="post-layout--right js-post-comments-component">
            <div id="comments-45537537" class="comments js-comments-container bt bc-black-075 mt12 " data-post-id="45537537" data-min-length="15">
                <ul class="comments-list js-comments-list"
                        data-remaining-comments-count="0"
                        data-canpost="false"
                        data-cansee="true"
                        data-comments-unavailable="false"
                        data-addlink-disabled="true">
    
                            <li id="comment-78035490" class="comment js-comment " data-comment-id="78035490" data-comment-owner-id="1273996" data-comment-score="0">
            <div class="js-comment-actions comment-actions">
                <div class="comment-score js-comment-edit-hide">
                </div>
            </div>
            <div class="comment-text  js-comment-text-and-form">
                <div class="comment-body js-comment-edit-hide">
                    
                    <span class="comment-copy">this doesn't really add to the answer does it? its just shows how you could see the websites source, but doesn't answer how to recreate this?</span>
                    
    – <a href="/users/1273996/lucas-reppe-welander"
                           title="886 reputation"
                           class="comment-user">Lucas Reppe Welander</a>
                    <span class="comment-date" dir="ltr"><span title="2017-08-06 23:56:48Z, License: CC BY-SA 3.0" class="relativetime-clean">Aug 6 '17 at 23:56</span></span>
                </div>
            </div>
        </li>
    
                </ul>
    	    </div>
    
            <div id="comments-link-45537537" data-rep=50 data-anon=true>
                        <a class="js-add-link comments-link disabled-link" title="Use comments to ask for more information or suggest improvements. Avoid comments like “+1” or “thanks”."  href="#" role="button">Add a comment</a>
                    <span class="js-link-separator dno"> | </span>
                <a class="js-show-link comments-link dno" title="Expand to show all comments on this post" href=# onclick="" role="button"></a>
            </div>         
        </div>
        </div>
    </div>
                                              
    <a name="43381291"></a>
    <div id="answer-43381291" class="answer" data-answerid="43381291" data-ownerid="6563293" data-score="0" itemprop="suggestedAnswer" itemscope itemtype="https://schema.org/Answer">
        <div class="post-layout">
            <div class="votecell post-layout--left">
                <div class="js-voting-container grid jc-center fd-column ai-stretch gs4 fc-black-200" data-post-id="43381291">
            <button class="js-vote-up-btn grid--cell s-btn s-btn__unset c-pointer" data-controller="s-tooltip" data-s-tooltip-placement="right" title="This answer is useful" aria-pressed="false" aria-label="Up vote" data-selected-classes="fc-theme-primary"><svg aria-hidden="true" class="m0 svg-icon iconArrowUpLg" width="36" height="36" viewBox="0 0 36 36"><path d="M2 26h32L18 10 2 26z"/></svg></button>
            <div class="js-vote-count grid--cell fc-black-500 fs-title grid fd-column ai-center" itemprop="upvoteCount" data-value="0">0</div>
            <button class="js-vote-down-btn grid--cell s-btn s-btn__unset c-pointer" data-controller="s-tooltip" data-s-tooltip-placement="right" title="This answer is not useful" aria-pressed="false" aria-label="Down vote" data-selected-classes="fc-theme-primary"><svg aria-hidden="true" class="m0 svg-icon iconArrowDownLg" width="36" height="36" viewBox="0 0 36 36"><path d="M2 10h32L18 26 2 10z"/></svg></button>
    
        
                <div class="js-accepted-answer-indicator grid--cell fc-green-500 py6 mtn8 d-none" data-s-tooltip-placement="right" title="Loading when this answer was accepted…" tabindex="0" role="note" aria-label="Accepted">
                    <div class="ta-center">
                        <svg aria-hidden="true" class="svg-icon iconCheckmarkLg" width="36" height="36" viewBox="0 0 36 36"><path d="M6 14l8 8L30 6v8L14 30l-8-8v-8z"/></svg>
                    </div>
                </div>
    
        
            <a class="js-post-issue grid--cell s-btn s-btn__unset c-pointer py6 mx-auto" href="/posts/43381291/timeline" data-shortcut="T" data-ks-title="timeline" data-controller="s-tooltip" data-s-tooltip-placement="right" title="Show activity on this post." aria-label="Timeline"><svg aria-hidden="true" class="mln2 mr0 svg-icon iconHistory" width="19" height="18" viewBox="0 0 19 18"><path d="M3 9a8 8 0 113.73 6.77L8.2 14.3A6 6 0 105 9l3.01-.01-4 4-4-4h3L3 9zm7-4h1.01L11 9.36l3.22 2.1-.6.93L10 10V5z"/></svg></a>
    
    </div>
            </div>
    
            
    
    <div class="answercell post-layout--right">
        
        <div class="s-prose js-post-body" itemprop="text">
    <p><strong>Update:</strong>
    Go to view-source:http://samy.pl which is the link of what the view source right click brings you to. It only works on Chrome I think.</p>
    
    <p>If you are using chrome, go to the menu at the top right. Then go to more tools, and click developer tools.</p>
        </div>
        <div class="mt24">
            <div class="grid fw-wrap ai-start jc-end gs8 gsy">
                <time itemprop="dateCreated" datetime="2017-04-12T23:37:42"></time>
                <div class="grid--cell mr16" style="flex: 1 1 100px;">
                    
    
    <div class="js-post-menu pt2" data-post-id="43381291">
        <div class="grid d-flex gs8 s-anchors s-anchors__muted fw-wrap">
    
            <div class="grid--cell">
                <a href="/a/43381291"
                   rel="nofollow"
                   itemprop="url"
                   class="js-share-link js-gps-track"
                   title="Short permalink to this answer"
                   data-gps-track="post.click({ item: 2, priv: 0, post_type: 2 })"
                   data-controller="se-share-sheet"
                   data-se-share-sheet-title="Share a link to this answer"
                   data-se-share-sheet-subtitle=""
                   data-se-share-sheet-post-type="answer"
                   data-se-share-sheet-social="facebook twitter devto"
                   data-se-share-sheet-location="2"
                   data-se-share-sheet-license-url="https%3a%2f%2fcreativecommons.org%2flicenses%2fby-sa%2f3.0%2f"
                   data-se-share-sheet-license-name="CC BY-SA 3.0"
                   data-s-popover-placement="bottom-start">Share</a>
            </div>
    
    
                    <div class="grid--cell">
                        <a href="/posts/43381291/edit" class="js-suggest-edit-post js-gps-track" data-gps-track="post.click({ item: 6, priv: 0, post_type: 2 })" title="">Improve this answer</a>
                    </div>
    
    
            <div class="grid--cell">
                <button type="button"
                        id="btnFollowPost-43381291" class="s-btn s-btn__link js-follow-post js-follow-answer js-gps-track"
                        data-gps-track="post.click({ item: 14, priv: 0, post_type: 2 })"
                        data-controller="s-tooltip " data-s-tooltip-placement="bottom"
                        data-s-popover-placement="bottom" aria-controls=""
                        title="Follow this answer to receive notifications">
                    Follow
                </button>
            </div>
    
    
    
    
        </div>
        <div class="js-menu-popup-container"></div>
    </div>
    
                </div>
                <div class="post-signature grid--cell fl0">
    <div class="user-info ">
        <div class="user-action-time">
            <a href="/posts/43381291/revisions" title="show all edits to this post"
                             class="js-gps-track"
                             data-gps-track="post.click({ item: 4, priv: 0, post_type: 2 })">edited <span title="2017-04-12 23:46:05Z" class="relativetime">Apr 12 '17 at 23:46</span></a>
        </div>
        <div class="user-gravatar32">
            
        </div>
        <div class="user-details">
            
            <div class="-flair">
                
            </div>
        </div>
    </div>
                </div>
    
    
                <div class="post-signature grid--cell fl0">
                    <div class="user-info ">
        <div class="user-action-time">
            answered <span title="2017-04-12 23:37:42Z" class="relativetime">Apr 12 '17 at 23:37</span>
        </div>
        <div class="user-gravatar32">
            <a href="/users/6563293/cameron"><div class="gravatar-wrapper-32"><img src="https://www.gravatar.com/avatar/09a704158b2c5e0ee9c7af1e3be2594c?s=32&d=identicon&r=PG&f=1" alt="" width="32" height="32" class="bar-sm"></div></a>
        </div>
        <div class="user-details" itemprop="author" itemscope itemtype="http://schema.org/Person">
            <a href="/users/6563293/cameron">Cameron</a><span class="d-none" itemprop="name">Cameron</span>
            <div class="-flair">
                <span class="reputation-score" title="reputation score " dir="ltr">949</span><span title="10 silver badges" aria-hidden="true"><span class="badge2"></span><span class="badgecount">10</span></span><span class="v-visible-sr">10 silver badges</span><span title="21 bronze badges" aria-hidden="true"><span class="badge3"></span><span class="badgecount">21</span></span><span class="v-visible-sr">21 bronze badges</span>
            </div>
        </div>
    </div>
    
    
                </div>
            </div>
        </div>
        
    </div>
    
    
    
    
                <span class="d-none" itemprop="commentCount">0</span> 
        <div class="post-layout--right js-post-comments-component">
            <div id="comments-43381291" class="comments js-comments-container bt bc-black-075 mt12  dno" data-post-id="43381291" data-min-length="15">
                <ul class="comments-list js-comments-list"
                        data-remaining-comments-count="0"
                        data-canpost="false"
                        data-cansee="true"
                        data-comments-unavailable="false"
                        data-addlink-disabled="true">
    
                </ul>
    	    </div>
    
            <div id="comments-link-43381291" data-rep=50 data-anon=true>
                        <a class="js-add-link comments-link disabled-link" title="Use comments to ask for more information or suggest improvements. Avoid comments like “+1” or “thanks”."  href="#" role="button">Add a comment</a>
                    <span class="js-link-separator dno"> | </span>
                <a class="js-show-link comments-link dno" title="Expand to show all comments on this post" href=# onclick="" role="button"></a>
            </div>         
        </div>
        </div>
    </div>
                                              
    <a name="43390012"></a>
    <div id="answer-43390012" class="answer" data-answerid="43390012" data-ownerid="3804991" data-score="0" itemprop="suggestedAnswer" itemscope itemtype="https://schema.org/Answer">
        <div class="post-layout">
            <div class="votecell post-layout--left">
                <div class="js-voting-container grid jc-center fd-column ai-stretch gs4 fc-black-200" data-post-id="43390012">
            <button class="js-vote-up-btn grid--cell s-btn s-btn__unset c-pointer" data-controller="s-tooltip" data-s-tooltip-placement="right" title="This answer is useful" aria-pressed="false" aria-label="Up vote" data-selected-classes="fc-theme-primary"><svg aria-hidden="true" class="m0 svg-icon iconArrowUpLg" width="36" height="36" viewBox="0 0 36 36"><path d="M2 26h32L18 10 2 26z"/></svg></button>
            <div class="js-vote-count grid--cell fc-black-500 fs-title grid fd-column ai-center" itemprop="upvoteCount" data-value="0">0</div>
            <button class="js-vote-down-btn grid--cell s-btn s-btn__unset c-pointer" data-controller="s-tooltip" data-s-tooltip-placement="right" title="This answer is not useful" aria-pressed="false" aria-label="Down vote" data-selected-classes="fc-theme-primary"><svg aria-hidden="true" class="m0 svg-icon iconArrowDownLg" width="36" height="36" viewBox="0 0 36 36"><path d="M2 10h32L18 26 2 10z"/></svg></button>
    
        
                <div class="js-accepted-answer-indicator grid--cell fc-green-500 py6 mtn8 d-none" data-s-tooltip-placement="right" title="Loading when this answer was accepted…" tabindex="0" role="note" aria-label="Accepted">
                    <div class="ta-center">
                        <svg aria-hidden="true" class="svg-icon iconCheckmarkLg" width="36" height="36" viewBox="0 0 36 36"><path d="M6 14l8 8L30 6v8L14 30l-8-8v-8z"/></svg>
                    </div>
                </div>
    
        
            <a class="js-post-issue grid--cell s-btn s-btn__unset c-pointer py6 mx-auto" href="/posts/43390012/timeline" data-shortcut="T" data-ks-title="timeline" data-controller="s-tooltip" data-s-tooltip-placement="right" title="Show activity on this post." aria-label="Timeline"><svg aria-hidden="true" class="mln2 mr0 svg-icon iconHistory" width="19" height="18" viewBox="0 0 19 18"><path d="M3 9a8 8 0 113.73 6.77L8.2 14.3A6 6 0 105 9l3.01-.01-4 4-4-4h3L3 9zm7-4h1.01L11 9.36l3.22 2.1-.6.93L10 10V5z"/></svg></a>
    
    </div>
            </div>
    
            
    
    <div class="answercell post-layout--right">
        
        <div class="s-prose js-post-body" itemprop="text">
    <p>After a lot of tinkering and speculating together, my roommate succeeded in the end with quite a naïve approach.</p>
    
    <p>Simply open the Chrome devtools and copy out the source before it gets replaced. Giving Chrome processes low priority and doing some heavy-lifting with your CPU (compilation or other) helps.</p>
    
    <p>Chop, chop.</p>
        </div>
        <div class="mt24">
            <div class="grid fw-wrap ai-start jc-end gs8 gsy">
                <time itemprop="dateCreated" datetime="2017-04-13T10:37:39"></time>
                <div class="grid--cell mr16" style="flex: 1 1 100px;">
                    
    
    <div class="js-post-menu pt2" data-post-id="43390012">
        <div class="grid d-flex gs8 s-anchors s-anchors__muted fw-wrap">
    
            <div class="grid--cell">
                <a href="/a/43390012"
                   rel="nofollow"
                   itemprop="url"
                   class="js-share-link js-gps-track"
                   title="Short permalink to this answer"
                   data-gps-track="post.click({ item: 2, priv: 0, post_type: 2 })"
                   data-controller="se-share-sheet"
                   data-se-share-sheet-title="Share a link to this answer"
                   data-se-share-sheet-subtitle=""
                   data-se-share-sheet-post-type="answer"
                   data-se-share-sheet-social="facebook twitter devto"
                   data-se-share-sheet-location="2"
                   data-se-share-sheet-license-url="https%3a%2f%2fcreativecommons.org%2flicenses%2fby-sa%2f3.0%2f"
                   data-se-share-sheet-license-name="CC BY-SA 3.0"
                   data-s-popover-placement="bottom-start">Share</a>
            </div>
    
    
                    <div class="grid--cell">
                        <a href="/posts/43390012/edit" class="js-suggest-edit-post js-gps-track" data-gps-track="post.click({ item: 6, priv: 0, post_type: 2 })" title="">Improve this answer</a>
                    </div>
    
    
            <div class="grid--cell">
                <button type="button"
                        id="btnFollowPost-43390012" class="s-btn s-btn__link js-follow-post js-follow-answer js-gps-track"
                        data-gps-track="post.click({ item: 14, priv: 0, post_type: 2 })"
                        data-controller="s-tooltip " data-s-tooltip-placement="bottom"
                        data-s-popover-placement="bottom" aria-controls=""
                        title="Follow this answer to receive notifications">
                    Follow
                </button>
            </div>
    
    
    
    
        </div>
        <div class="js-menu-popup-container"></div>
    </div>
    
                </div>
    
    
                <div class="post-signature grid--cell fl0">
                    <div class="user-info user-hover">
        <div class="user-action-time">
            answered <span title="2017-04-13 10:37:39Z" class="relativetime">Apr 13 '17 at 10:37</span>
        </div>
        <div class="user-gravatar32">
            <a href="/users/3804991/viktor-sec"><div class="gravatar-wrapper-32"><img src="https://i.stack.imgur.com/5XZw1.png?s=32&g=1" alt="" width="32" height="32" class="bar-sm"></div></a>
        </div>
        <div class="user-details" itemprop="author" itemscope itemtype="http://schema.org/Person">
            <a href="/users/3804991/viktor-sec">Viktor Sec</a><span class="d-none" itemprop="name">Viktor Sec</span>
            <div class="-flair">
                <span class="reputation-score" title="reputation score " dir="ltr">2,278</span><span title="1 gold badge" aria-hidden="true"><span class="badge1"></span><span class="badgecount">1</span></span><span class="v-visible-sr">1 gold badge</span><span title="19 silver badges" aria-hidden="true"><span class="badge2"></span><span class="badgecount">19</span></span><span class="v-visible-sr">19 silver badges</span><span title="28 bronze badges" aria-hidden="true"><span class="badge3"></span><span class="badgecount">28</span></span><span class="v-visible-sr">28 bronze badges</span>
            </div>
        </div>
    </div>
    
    
                </div>
            </div>
        </div>
        
    </div>
    
    
    
    
                <span class="d-none" itemprop="commentCount"></span> 
        <div class="post-layout--right js-post-comments-component">
            <div id="comments-43390012" class="comments js-comments-container bt bc-black-075 mt12  dno" data-post-id="43390012" data-min-length="15">
                <ul class="comments-list js-comments-list"
                        data-remaining-comments-count="0"
                        data-canpost="false"
                        data-cansee="true"
                        data-comments-unavailable="false"
                        data-addlink-disabled="true">
    
                </ul>
    	    </div>
    
            <div id="comments-link-43390012" data-rep=50 data-anon=true>
                        <a class="js-add-link comments-link disabled-link" title="Use comments to ask for more information or suggest improvements. Avoid comments like “+1” or “thanks”."  href="#" role="button">Add a comment</a>
                    <span class="js-link-separator dno"> | </span>
                <a class="js-show-link comments-link dno" title="Expand to show all comments on this post" href=# onclick="" role="button"></a>
            </div>         
        </div>
        </div>
    </div>
                                              
    <a name="43989801"></a>
    <div id="answer-43989801" class="answer" data-answerid="43989801" data-ownerid="7288364" data-score="0" itemprop="suggestedAnswer" itemscope itemtype="https://schema.org/Answer">
        <div class="post-layout">
            <div class="votecell post-layout--left">
                <div class="js-voting-container grid jc-center fd-column ai-stretch gs4 fc-black-200" data-post-id="43989801">
            <button class="js-vote-up-btn grid--cell s-btn s-btn__unset c-pointer" data-controller="s-tooltip" data-s-tooltip-placement="right" title="This answer is useful" aria-pressed="false" aria-label="Up vote" data-selected-classes="fc-theme-primary"><svg aria-hidden="true" class="m0 svg-icon iconArrowUpLg" width="36" height="36" viewBox="0 0 36 36"><path d="M2 26h32L18 10 2 26z"/></svg></button>
            <div class="js-vote-count grid--cell fc-black-500 fs-title grid fd-column ai-center" itemprop="upvoteCount" data-value="0">0</div>
            <button class="js-vote-down-btn grid--cell s-btn s-btn__unset c-pointer" data-controller="s-tooltip" data-s-tooltip-placement="right" title="This answer is not useful" aria-pressed="false" aria-label="Down vote" data-selected-classes="fc-theme-primary"><svg aria-hidden="true" class="m0 svg-icon iconArrowDownLg" width="36" height="36" viewBox="0 0 36 36"><path d="M2 10h32L18 26 2 10z"/></svg></button>
    
        
                <div class="js-accepted-answer-indicator grid--cell fc-green-500 py6 mtn8 d-none" data-s-tooltip-placement="right" title="Loading when this answer was accepted…" tabindex="0" role="note" aria-label="Accepted">
                    <div class="ta-center">
                        <svg aria-hidden="true" class="svg-icon iconCheckmarkLg" width="36" height="36" viewBox="0 0 36 36"><path d="M6 14l8 8L30 6v8L14 30l-8-8v-8z"/></svg>
                    </div>
                </div>
    
        
            <a class="js-post-issue grid--cell s-btn s-btn__unset c-pointer py6 mx-auto" href="/posts/43989801/timeline" data-shortcut="T" data-ks-title="timeline" data-controller="s-tooltip" data-s-tooltip-placement="right" title="Show activity on this post." aria-label="Timeline"><svg aria-hidden="true" class="mln2 mr0 svg-icon iconHistory" width="19" height="18" viewBox="0 0 19 18"><path d="M3 9a8 8 0 113.73 6.77L8.2 14.3A6 6 0 105 9l3.01-.01-4 4-4-4h3L3 9zm7-4h1.01L11 9.36l3.22 2.1-.6.93L10 10V5z"/></svg></a>
    
    </div>
            </div>
    
            
    
    <div class="answercell post-layout--right">
        
        <div class="s-prose js-post-body" itemprop="text">
    <p>Ah! I have found an <b><i>extremely</i></b> easy way to do it.<br>
    1. Open Firefox (Chrome only shows you part of it).<br>
    2. In the URL bar, type in <code>javascript:alert(document.documentElement.outerHTML);</code><br>
    3. Press <kbd>Enter</kbd>.<br>
    4. Voilà!<br></p>
        </div>
        <div class="mt24">
            <div class="grid fw-wrap ai-start jc-end gs8 gsy">
                <time itemprop="dateCreated" datetime="2017-05-15T22:32:05"></time>
                <div class="grid--cell mr16" style="flex: 1 1 100px;">
                    
    
    <div class="js-post-menu pt2" data-post-id="43989801">
        <div class="grid d-flex gs8 s-anchors s-anchors__muted fw-wrap">
    
            <div class="grid--cell">
                <a href="/a/43989801"
                   rel="nofollow"
                   itemprop="url"
                   class="js-share-link js-gps-track"
                   title="Short permalink to this answer"
                   data-gps-track="post.click({ item: 2, priv: 0, post_type: 2 })"
                   data-controller="se-share-sheet"
                   data-se-share-sheet-title="Share a link to this answer"
                   data-se-share-sheet-subtitle=""
                   data-se-share-sheet-post-type="answer"
                   data-se-share-sheet-social="facebook twitter devto"
                   data-se-share-sheet-location="2"
                   data-se-share-sheet-license-url="https%3a%2f%2fcreativecommons.org%2flicenses%2fby-sa%2f3.0%2f"
                   data-se-share-sheet-license-name="CC BY-SA 3.0"
                   data-s-popover-placement="bottom-start">Share</a>
            </div>
    
    
                    <div class="grid--cell">
                        <a href="/posts/43989801/edit" class="js-suggest-edit-post js-gps-track" data-gps-track="post.click({ item: 6, priv: 0, post_type: 2 })" title="">Improve this answer</a>
                    </div>
    
    
            <div class="grid--cell">
                <button type="button"
                        id="btnFollowPost-43989801" class="s-btn s-btn__link js-follow-post js-follow-answer js-gps-track"
                        data-gps-track="post.click({ item: 14, priv: 0, post_type: 2 })"
                        data-controller="s-tooltip " data-s-tooltip-placement="bottom"
                        data-s-popover-placement="bottom" aria-controls=""
                        title="Follow this answer to receive notifications">
                    Follow
                </button>
            </div>
    
    
    
    
        </div>
        <div class="js-menu-popup-container"></div>
    </div>
    
                </div>
    
    
                <div class="post-signature grid--cell fl0">
                    <div class="user-info ">
        <div class="user-action-time">
            answered <span title="2017-05-15 22:32:05Z" class="relativetime">May 15 '17 at 22:32</span>
        </div>
        <div class="user-gravatar32">
            <a href="/users/7288364/cerebraldatabank"><div class="gravatar-wrapper-32"><img src="https://www.gravatar.com/avatar/7e7d735991f8ad31199ea4c754d01934?s=32&d=identicon&r=PG&f=1" alt="" width="32" height="32" class="bar-sm"></div></a>
        </div>
        <div class="user-details" itemprop="author" itemscope itemtype="http://schema.org/Person">
            <a href="/users/7288364/cerebraldatabank">CerebralDatabank</a><span class="d-none" itemprop="name">CerebralDatabank</span>
            <div class="-flair">
                <span class="reputation-score" title="reputation score " dir="ltr">37</span><span title="1 silver badge" aria-hidden="true"><span class="badge2"></span><span class="badgecount">1</span></span><span class="v-visible-sr">1 silver badge</span><span title="6 bronze badges" aria-hidden="true"><span class="badge3"></span><span class="badgecount">6</span></span><span class="v-visible-sr">6 bronze badges</span>
            </div>
        </div>
    </div>
    
    
                </div>
            </div>
        </div>
        
    </div>
    
    
    
    
                <span class="d-none" itemprop="commentCount"></span> 
        <div class="post-layout--right js-post-comments-component">
            <div id="comments-43989801" class="comments js-comments-container bt bc-black-075 mt12  dno" data-post-id="43989801" data-min-length="15">
                <ul class="comments-list js-comments-list"
                        data-remaining-comments-count="0"
                        data-canpost="false"
                        data-cansee="true"
                        data-comments-unavailable="false"
                        data-addlink-disabled="true">
    
                </ul>
    	    </div>
    
            <div id="comments-link-43989801" data-rep=50 data-anon=true>
                        <a class="js-add-link comments-link disabled-link" title="Use comments to ask for more information or suggest improvements. Avoid comments like “+1” or “thanks”."  href="#" role="button">Add a comment</a>
                    <span class="js-link-separator dno"> | </span>
                <a class="js-show-link comments-link dno" title="Expand to show all comments on this post" href=# onclick="" role="button"></a>
            </div>         
        </div>
        </div>
    </div>
                                              
    <a name="43381294"></a>
    <div id="answer-43381294" class="answer" data-answerid="43381294" data-ownerid="7607671" data-score="-2" itemprop="suggestedAnswer" itemscope itemtype="https://schema.org/Answer">
        <div class="post-layout">
            <div class="votecell post-layout--left">
                <div class="js-voting-container grid jc-center fd-column ai-stretch gs4 fc-black-200" data-post-id="43381294">
            <button class="js-vote-up-btn grid--cell s-btn s-btn__unset c-pointer" data-controller="s-tooltip" data-s-tooltip-placement="right" title="This answer is useful" aria-pressed="false" aria-label="Up vote" data-selected-classes="fc-theme-primary"><svg aria-hidden="true" class="m0 svg-icon iconArrowUpLg" width="36" height="36" viewBox="0 0 36 36"><path d="M2 26h32L18 10 2 26z"/></svg></button>
            <div class="js-vote-count grid--cell fc-black-500 fs-title grid fd-column ai-center" itemprop="upvoteCount" data-value="-2">-2</div>
            <button class="js-vote-down-btn grid--cell s-btn s-btn__unset c-pointer" data-controller="s-tooltip" data-s-tooltip-placement="right" title="This answer is not useful" aria-pressed="false" aria-label="Down vote" data-selected-classes="fc-theme-primary"><svg aria-hidden="true" class="m0 svg-icon iconArrowDownLg" width="36" height="36" viewBox="0 0 36 36"><path d="M2 10h32L18 26 2 10z"/></svg></button>
    
        
                <div class="js-accepted-answer-indicator grid--cell fc-green-500 py6 mtn8 d-none" data-s-tooltip-placement="right" title="Loading when this answer was accepted…" tabindex="0" role="note" aria-label="Accepted">
                    <div class="ta-center">
                        <svg aria-hidden="true" class="svg-icon iconCheckmarkLg" width="36" height="36" viewBox="0 0 36 36"><path d="M6 14l8 8L30 6v8L14 30l-8-8v-8z"/></svg>
                    </div>
                </div>
    
        
            <a class="js-post-issue grid--cell s-btn s-btn__unset c-pointer py6 mx-auto" href="/posts/43381294/timeline" data-shortcut="T" data-ks-title="timeline" data-controller="s-tooltip" data-s-tooltip-placement="right" title="Show activity on this post." aria-label="Timeline"><svg aria-hidden="true" class="mln2 mr0 svg-icon iconHistory" width="19" height="18" viewBox="0 0 19 18"><path d="M3 9a8 8 0 113.73 6.77L8.2 14.3A6 6 0 105 9l3.01-.01-4 4-4-4h3L3 9zm7-4h1.01L11 9.36l3.22 2.1-.6.93L10 10V5z"/></svg></a>
    
    </div>
            </div>
    
            
    
    <div class="answercell post-layout--right">
        
        <div class="s-prose js-post-body" itemprop="text">
    <p>You can simply use the developer tools in almost all the browsers. Just Press F12 and the developer tools section will be there.</p>
    
    <p>Thanks</p>
        </div>
        <div class="mt24">
            <div class="grid fw-wrap ai-start jc-end gs8 gsy">
                <time itemprop="dateCreated" datetime="2017-04-12T23:38:22"></time>
                <div class="grid--cell mr16" style="flex: 1 1 100px;">
                    
    
    <div class="js-post-menu pt2" data-post-id="43381294">
        <div class="grid d-flex gs8 s-anchors s-anchors__muted fw-wrap">
    
            <div class="grid--cell">
                <a href="/a/43381294"
                   rel="nofollow"
                   itemprop="url"
                   class="js-share-link js-gps-track"
                   title="Short permalink to this answer"
                   data-gps-track="post.click({ item: 2, priv: 0, post_type: 2 })"
                   data-controller="se-share-sheet"
                   data-se-share-sheet-title="Share a link to this answer"
                   data-se-share-sheet-subtitle=""
                   data-se-share-sheet-post-type="answer"
                   data-se-share-sheet-social="facebook twitter devto"
                   data-se-share-sheet-location="2"
                   data-se-share-sheet-license-url="https%3a%2f%2fcreativecommons.org%2flicenses%2fby-sa%2f3.0%2f"
                   data-se-share-sheet-license-name="CC BY-SA 3.0"
                   data-s-popover-placement="bottom-start">Share</a>
            </div>
    
    
                    <div class="grid--cell">
                        <a href="/posts/43381294/edit" class="js-suggest-edit-post js-gps-track" data-gps-track="post.click({ item: 6, priv: 0, post_type: 2 })" title="">Improve this answer</a>
                    </div>
    
    
            <div class="grid--cell">
                <button type="button"
                        id="btnFollowPost-43381294" class="s-btn s-btn__link js-follow-post js-follow-answer js-gps-track"
                        data-gps-track="post.click({ item: 14, priv: 0, post_type: 2 })"
                        data-controller="s-tooltip " data-s-tooltip-placement="bottom"
                        data-s-popover-placement="bottom" aria-controls=""
                        title="Follow this answer to receive notifications">
                    Follow
                </button>
            </div>
    
    
    
    
        </div>
        <div class="js-menu-popup-container"></div>
    </div>
    
                </div>
    
    
                <div class="post-signature grid--cell fl0">
                    <div class="user-info user-hover">
        <div class="user-action-time">
            answered <span title="2017-04-12 23:38:22Z" class="relativetime">Apr 12 '17 at 23:38</span>
        </div>
        <div class="user-gravatar32">
            <a href="/users/7607671/harsheet"><div class="gravatar-wrapper-32"><img src="https://www.gravatar.com/avatar/1f37ca2821c6926f7a40501854a5e052?s=32&d=identicon&r=PG&f=1" alt="" width="32" height="32" class="bar-sm"></div></a>
        </div>
        <div class="user-details" itemprop="author" itemscope itemtype="http://schema.org/Person">
            <a href="/users/7607671/harsheet">Harsheet</a><span class="d-none" itemprop="name">Harsheet</span>
            <div class="-flair">
                <span class="reputation-score" title="reputation score " dir="ltr">699</span><span title="9 silver badges" aria-hidden="true"><span class="badge2"></span><span class="badgecount">9</span></span><span class="v-visible-sr">9 silver badges</span><span title="22 bronze badges" aria-hidden="true"><span class="badge3"></span><span class="badgecount">22</span></span><span class="v-visible-sr">22 bronze badges</span>
            </div>
        </div>
    </div>
    
    
                </div>
            </div>
        </div>
        
    </div>
    
    
    
    
                <span class="d-none" itemprop="commentCount">2</span> 
        <div class="post-layout--right js-post-comments-component">
            <div id="comments-43381294" class="comments js-comments-container bt bc-black-075 mt12 " data-post-id="43381294" data-min-length="15">
                <ul class="comments-list js-comments-list"
                        data-remaining-comments-count="0"
                        data-canpost="false"
                        data-cansee="true"
                        data-comments-unavailable="false"
                        data-addlink-disabled="true">
    
                            <li id="comment-73824691" class="comment js-comment " data-comment-id="73824691" data-comment-owner-id="6563293" data-comment-score="0">
            <div class="js-comment-actions comment-actions">
                <div class="comment-score js-comment-edit-hide">
                </div>
            </div>
            <div class="comment-text  js-comment-text-and-form">
                <div class="comment-body js-comment-edit-hide">
                    
                    <span class="comment-copy">That might also be blocked by the code on the website because if right click is blocked, and those keyboard commands are blocked, then F12 is probably blocked too.</span>
                    
    – <a href="/users/6563293/cameron"
                           title="949 reputation"
                           class="comment-user">Cameron</a>
                    <span class="comment-date" dir="ltr"><span title="2017-04-12 23:41:33Z, License: CC BY-SA 3.0" class="relativetime-clean">Apr 12 '17 at 23:41</span></span>
                </div>
            </div>
        </li>
        <li id="comment-73824774" class="comment js-comment " data-comment-id="73824774" data-comment-owner-id="5752562" data-comment-score="0">
            <div class="js-comment-actions comment-actions">
                <div class="comment-score js-comment-edit-hide">
                </div>
            </div>
            <div class="comment-text  js-comment-text-and-form">
                <div class="comment-body js-comment-edit-hide">
                    
                    <span class="comment-copy">F12 does work, it brings the dev tools, but the site detects this and loads other content. Sites have limited control over browsers. Why do you think they're limiting js xss.</span>
                    
    – <a href="/users/5752562/yezzz"
                           title="2,810 reputation"
                           class="comment-user">yezzz</a>
                    <span class="comment-date" dir="ltr"><span title="2017-04-12 23:46:32Z, License: CC BY-SA 3.0" class="relativetime-clean">Apr 12 '17 at 23:46</span></span>
                </div>
            </div>
        </li>
    
                </ul>
    	    </div>
    
            <div id="comments-link-43381294" data-rep=50 data-anon=true>
                        <a class="js-add-link comments-link disabled-link" title="Use comments to ask for more information or suggest improvements. Avoid comments like “+1” or “thanks”."  href="#" role="button">Add a comment</a>
                    <span class="js-link-separator dno"> | </span>
                <a class="js-show-link comments-link dno" title="Expand to show all comments on this post" href=# onclick="" role="button"></a>
            </div>         
        </div>
        </div>
    </div>
    
                            <a name='new-answer'></a>
                                <form id="post-form" action="/questions/43381154/answer/submit" method="post" class="js-add-answer-component post-form">
                                    <input type="hidden" id="post-id" value="43381154" />
                                    <input type="hidden" id="qualityBanWarningShown" name="qualityBanWarningShown" value="false" />
                                    <input type="hidden" name="referrer" value="https://www.abclinuxu.cz/" />
                                    <h2 class="space">
                                        Your Answer
                                    </h2>
                                        
    
        <script>
            StackExchange.ifUsing("editor", function () {
                StackExchange.using("externalEditor", function () {
                    StackExchange.using("snippets", function () {
                        StackExchange.snippets.init();
                    });
                });
            }, "code-snippets");
        </script>
    
    
    <script>
        StackExchange.ready(function() {
            var channelOptions = {
                tags: "".split(" "),
                id: "1"
            };
            initTagRenderer("".split(" "), "".split(" "), channelOptions);
    
            StackExchange.using("externalEditor", function() {
                // Have to fire editor after snippets, if snippets enabled
                if (StackExchange.settings.snippets.snippetsEnabled) {
                    StackExchange.using("snippets", function() {
                        createEditor();
                    });
                }
                else {
                    createEditor();
                }
            });
    
            function createEditor() {
                StackExchange.prepareEditor({
                    useStacksEditor: false,
                    heartbeatType: 'answer',
                    autoActivateHeartbeat: false,
                    convertImagesToLinks: true,
                    noModals: true,
                    showLowRepImageUploadWarning: true,
                    reputationToPostImages: 10,
                    bindNavPrevention: true,
                    postfix: "",
                    imageUploader: {
                        brandingHtml: "Powered by \u003ca href=\"https://imgur.com/\"\u003e\u003csvg class=\"svg-icon\" width=\"50\" height=\"18\" viewBox=\"0 0 50 18\" fill=\"none\" xmlns=\"http://www.w3.org/2000/svg\"\u003e\u003cpath d=\"M46.1709 9.17788C46.1709 8.26454 46.2665 7.94324 47.1084 7.58816C47.4091 7.46349 47.7169 7.36433 48.0099 7.26993C48.9099 6.97997 49.672 6.73443 49.672 5.93063C49.672 5.22043 48.9832 4.61182 48.1414 4.61182C47.4335 4.61182 46.7256 4.91628 46.0943 5.50789C45.7307 4.9328 45.2525 4.66231 44.6595 4.66231C43.6264 4.66231 43.1481 5.28821 43.1481 6.59048V11.9512C43.1481 13.2535 43.6264 13.8962 44.6595 13.8962C45.6924 13.8962 46.1709 13.2535 46.1709 11.9512V9.17788Z\"/\u003e\u003cpath d=\"M32.492 10.1419C32.492 12.6954 34.1182 14.0484 37.0451 14.0484C39.9723 14.0484 41.5985 12.6954 41.5985 10.1419V6.59049C41.5985 5.28821 41.1394 4.66232 40.1061 4.66232C39.0732 4.66232 38.5948 5.28821 38.5948 6.59049V9.60062C38.5948 10.8521 38.2696 11.5455 37.0451 11.5455C35.8209 11.5455 35.4954 10.8521 35.4954 9.60062V6.59049C35.4954 5.28821 35.0173 4.66232 34.0034 4.66232C32.9703 4.66232 32.492 5.28821 32.492 6.59049V10.1419Z\" /\u003e\u003cpath fill-rule=\"evenodd\" clip-rule=\"evenodd\" d=\"M25.6622 17.6335C27.8049 17.6335 29.3739 16.9402 30.2537 15.6379C30.8468 14.7755 30.9615 13.5579 30.9615 11.9512V6.59049C30.9615 5.28821 30.4833 4.66231 29.4502 4.66231C28.9913 4.66231 28.4555 4.94978 28.1109 5.50789C27.499 4.86533 26.7335 4.56087 25.7005 4.56087C23.1369 4.56087 21.0134 6.57349 21.0134 9.27932C21.0134 11.9852 23.003 13.913 25.3754 13.913C26.5612 13.913 27.4607 13.4902 28.1109 12.6616C28.1109 12.7229 28.1161 12.7799 28.121 12.8346C28.1256 12.8854 28.1301 12.9342 28.1301 12.983C28.1301 14.4373 27.2502 15.2321 25.777 15.2321C24.8349 15.2321 24.1352 14.9821 23.5661 14.7787C23.176 14.6393 22.8472 14.5218 22.5437 14.5218C21.7977 14.5218 21.2429 15.0123 21.2429 15.6887C21.2429 16.7375 22.9072 17.6335 25.6622 17.6335ZM24.1317 9.27932C24.1317 7.94324 24.9928 7.09766 26.1024 7.09766C27.2119 7.09766 28.0918 7.94324 28.0918 9.27932C28.0918 10.6321 27.2311 11.5116 26.1024 11.5116C24.9737 11.5116 24.1317 10.6491 24.1317 9.27932Z\"/\u003e\u003cpath d=\"M16.8045 11.9512C16.8045 13.2535 17.2637 13.8962 18.2965 13.8962C19.3298 13.8962 19.8079 13.2535 19.8079 11.9512V8.12928C19.8079 5.82936 18.4879 4.62866 16.4027 4.62866C15.1594 4.62866 14.279 4.98375 13.3609 5.88013C12.653 5.05154 11.6581 4.62866 10.3573 4.62866C9.34336 4.62866 8.57809 4.89931 7.9466 5.5079C7.58314 4.9328 7.10506 4.66232 6.51203 4.66232C5.47873 4.66232 5.00066 5.28821 5.00066 6.59049V11.9512C5.00066 13.2535 5.47873 13.8962 6.51203 13.8962C7.54479 13.8962 8.0232 13.2535 8.0232 11.9512V8.90741C8.0232 7.58817 8.44431 6.91179 9.53458 6.91179C10.5104 6.91179 10.893 7.58817 10.893 8.94108V11.9512C10.893 13.2535 11.3711 13.8962 12.4044 13.8962C13.4375 13.8962 13.9157 13.2535 13.9157 11.9512V8.90741C13.9157 7.58817 14.3365 6.91179 15.4269 6.91179C16.4027 6.91179 16.8045 7.58817 16.8045 8.94108V11.9512Z\"/\u003e\u003cpath d=\"M3.31675 6.59049C3.31675 5.28821 2.83866 4.66232 1.82471 4.66232C0.791758 4.66232 0.313354 5.28821 0.313354 6.59049V11.9512C0.313354 13.2535 0.791758 13.8962 1.82471 13.8962C2.85798 13.8962 3.31675 13.2535 3.31675 11.9512V6.59049Z\" /\u003e\u003cpath d=\"M1.87209 0.400291C0.843612 0.400291 0 1.1159 0 1.98861C0 2.87869 0.822846 3.57676 1.87209 3.57676C2.90056 3.57676 3.7234 2.87869 3.7234 1.98861C3.7234 1.1159 2.90056 0.400291 1.87209 0.400291Z\" fill=\"#1BB76E\"/\u003e\u003c/svg\u003e\u003c/a\u003e",
                        contentPolicyHtml: "User contributions licensed under \u003ca href=\"https://stackoverflow.com/help/licensing\"\u003ecc by-sa\u003c/a\u003e \u003ca href=\"https://stackoverflow.com/legal/content-policy\"\u003e(content policy)\u003c/a\u003e",
                        allowUrls: true
                    },
                    onDemand: true,
                    discardSelector: ".discard-answer"
                    ,immediatelyShowMarkdownHelp:true,enableTables:true,enableSnippets:true
                });
                        }
        });
    </script>
    <div id="post-editor" class="post-editor js-post-editor">
    
    
            <div class="ps-relative">
                <div class="wmd-container mb8">
                    <div id="wmd-button-bar" class="wmd-button-bar btr-sm"></div>
                    <div class="js-stacks-validation">
                        <div class="ps-relative">
                            <textarea id="wmd-input"
                                      name="post-text"
                                      class="wmd-input s-input bar0 js-post-body-field"
                                      data-post-type-id="2"
                                      cols="92" rows="15"
                                      tabindex="101"
                                      data-min-length=""></textarea>
                        </div>
                        <div class="s-input-message mt4 d-none js-stacks-validation-message"></div>
                    </div>
                </div>
            </div>
    
        <aside class="grid ai-start jc-space-between js-answer-help s-notice s-notice__warning pb0 pr4 pt4 mb8 d-none" role="status" aria-hidden="true">
        <div class="grid--cell pt8">
            <p>Thanks for contributing an answer to Stack Overflow!</p><ul><li>Please be sure to <em>answer the question</em>. Provide details and share your research!</li></ul><p>But <em>avoid</em> …</p><ul><li>Asking for help, clarification, or responding to other answers.</li><li>Making statements based on opinion; back them up with references or personal experience.</li></ul><p>To learn more, see our <a href="/help/how-to-answer">tips on writing great answers</a>.</p>
        </div>
        <button class="grid--cell js-answer-help-close-btn s-btn s-btn__muted fc-dark">
            <svg aria-hidden="true" class="svg-icon iconClear" width="18" height="18" viewBox="0 0 18 18"><path d="M15 4.41L13.59 3 9 7.59 4.41 3 3 4.41 7.59 9 3 13.59 4.41 15 9 10.41 13.59 15 15 13.59 10.41 9 15 4.41z"/></svg>
        </button>
    </aside>
    
    
    
        <div>
            <div id="draft-saved" class="fc-success h24" style="display:none;">Draft saved</div>
            <div id="draft-discarded" class="fc-error h24" style="display:none;">Draft discarded</div>
        </div>
    
    
                <div id="wmd-preview" class="s-prose mb16 wmd-preview js-wmd-preview"></div>
                <div></div>
    
            <div class="edit-block">
                <input id="fkey" name="fkey" type="hidden" value="41be0e11ff0983bf4a66e841ea7eb1d10d333a81847c19395d94a9518c426deb">
                <input id="author" name="author" type="text">
            </div>
    
    </div>
    
    
                                    <div class="ps-relative">
                                                    <div class="form-item dno new-post-login p0 my16">
                    <div class="grid gs16 md:fd-column new-login-form">
                        <div class="grid fd-column w50 md:w-auto gsy gs8 jc-space-between new-login-left">
                            <h3 class="grid--cell fs-title">Sign up or <a id="login-link" href="/users/login?ssrc=question_page&returnurl=https%3a%2f%2fstackoverflow.com%2fquestions%2f43381154%2fhow-do-i-able-to-see-the-source-code-of-this-website-www-samy-pl%23new-answer">log in</a></h3>
                            <script>
                                StackExchange.ready(function () {
                                    StackExchange.helpers.onClickDraftSave('#login-link');
                                });
                            </script>
                            <div class="grid--cell s-btn s-btn__muted s-btn__outlined s-btn__icon google-login" data-ga="["sign up","Sign Up Started - Google","New Post",null,null]">
                                <svg aria-hidden="true" class="native svg-icon iconGoogle" width="18" height="18" viewBox="0 0 18 18"><path d="M16.51 8H8.98v3h4.3c-.18 1-.74 1.48-1.6 2.04v2.01h2.6a7.8 7.8 0 002.38-5.88c0-.57-.05-.66-.15-1.18z" fill="#4285F4"/><path d="M8.98 17c2.16 0 3.97-.72 5.3-1.94l-2.6-2a4.8 4.8 0 01-7.18-2.54H1.83v2.07A8 8 0 008.98 17z" fill="#34A853"/><path d="M4.5 10.52a4.8 4.8 0 010-3.04V5.41H1.83a8 8 0 000 7.18l2.67-2.07z" fill="#FBBC05"/><path d="M8.98 4.18c1.17 0 2.23.4 3.06 1.2l2.3-2.3A8 8 0 001.83 5.4L4.5 7.49a4.77 4.77 0 014.48-3.3z" fill="#EA4335"/></svg> Sign up using Google
                            </div>
                            <div class="grid--cell s-btn s-btn__muted s-btn__icon facebook-login" data-ga="["sign up","Sign Up Started - Facebook","New Post",null,null]">
                                <svg aria-hidden="true" class="svg-icon iconFacebook" width="18" height="18" viewBox="0 0 18 18"><path d="M3 1a2 2 0 00-2 2v12c0 1.1.9 2 2 2h12a2 2 0 002-2V3a2 2 0 00-2-2H3zm6.55 16v-6.2H7.46V8.4h2.09V6.61c0-2.07 1.26-3.2 3.1-3.2.88 0 1.64.07 1.87.1v2.16h-1.29c-1 0-1.19.48-1.19 1.18V8.4h2.39l-.31 2.42h-2.08V17h-2.5z" fill="#4167B2"/></svg> Sign up using Facebook
                            </div>
                            <div class="grid--cell s-btn s-btn__muted s-btn__outlined s-btn__icon stackexchange-login" data-ga="["sign up","Sign Up Navigation","New Post",null,null]">
                                <svg aria-hidden="true" class="native svg-icon iconLogoGlyphXSm" width="18" height="18" viewBox="0 0 18 18"><path d="M14 16v-5h2v7H2v-7h2v5h10z" fill="#BCBBBB"/><path d="M12.09.72l-1.21.9 4.5 6.07 1.22-.9L12.09.71zM5 15h8v-2H5v2zm9.15-5.87L8.35 4.3l.96-1.16 5.8 4.83-.96 1.16zm-7.7-1.47l6.85 3.19.63-1.37-6.85-3.2-.63 1.38zm6.53 5L5.4 11.39l.38-1.67 7.42 1.48-.22 1.46z" fill="#F48024"/></svg> Sign up using Email and Password
                            </div>
                        </div>
                        <input type="hidden" name="use-facebook" class="use-facebook" value="false" />
                        <input type="hidden" name="use-google" class="use-google" value="false" />
                        <button type="button" class="d-none js-submit-openid">Submit</button>
                        <div class="grid gsy gs8 fd-column w50 md:w-auto new-login-right form-item p0">
                                    <h3 class="grid--cell fs-title">Post as a guest</h3>
                <div class="grid--cell">
                    <div class="grid gs4 gsy fd-column">
                        <label class="s-label" for="display-name">Name</label>
                        <div class="grid ps-relative">
                            <input class="s-input" id="display-name" name="display-name" maxlength="30" type="text" value="" tabindex="105" placeholder="" />
                        </div>
                    </div>
                </div>
                <div class="grid--cell">
                    <div class="grid gs4 gsy fd-column">
                        <div class="grid--cell">
                            <div class="grid gs2 gsy fd-column">
                                <label class="grid--cell s-label" for="m-address">Email</label>
                                <p class="grid--cell s-description">Required, but never shown</p>
                            </div>
                        </div>
                        <div class="grid ps-relative">
                            <input class="s-input js-post-email-field" id="m-address" name="m-address" type="text" value="" size="40" tabindex="106" placeholder="" />
                        </div>
                    </div>
                </div>
    
                        </div>
                    </div>
                </div>
                <script>
                    StackExchange.ready(
                        function () {
                            StackExchange.openid.initPostLogin('.new-post-login', 'https%3a%2f%2fstackoverflow.com%2fquestions%2f43381154%2fhow-do-i-able-to-see-the-source-code-of-this-website-www-samy-pl%23new-answer', 'question_page');
                        }
                    );
                </script>
                <noscript>
                            <h3 class="grid--cell fs-title">Post as a guest</h3>
                <div class="grid--cell">
                    <div class="grid gs4 gsy fd-column">
                        <label class="s-label" for="display-name">Name</label>
                        <div class="grid ps-relative">
                            <input class="s-input" id="display-name" name="display-name" maxlength="30" type="text" value="" tabindex="105" placeholder="" />
                        </div>
                    </div>
                </div>
                <div class="grid--cell">
                    <div class="grid gs4 gsy fd-column">
                        <div class="grid--cell">
                            <div class="grid gs2 gsy fd-column">
                                <label class="grid--cell s-label" for="m-address">Email</label>
                                <p class="grid--cell s-description">Required, but never shown</p>
                            </div>
                        </div>
                        <div class="grid ps-relative">
                            <input class="s-input js-post-email-field" id="m-address" name="m-address" type="text" value="" size="40" tabindex="106" placeholder="" />
                        </div>
                    </div>
                </div>
    
                </noscript>
    
                                    </div>
    
                                        <div class="form-submit cbt grid gsx gs4">
                                            <button id="submit-button" class="grid--cell s-btn s-btn__primary s-btn__icon" type="submit" tabindex="120" autocomplete="off">
    Post Your Answer                                        </button>
                                            <button class="grid--cell s-btn s-btn__danger discard-answer dno">
                                                Discard
                                            </button>
                                                <p class="privacy-policy-agreement">
                                                    By clicking “Post Your Answer”, you agree to our <a href='https://stackoverflow.com/legal/terms-of-service/public' name='tos' target='_blank' class='-link'>terms of service</a>, <a href='https://stackoverflow.com/legal/privacy-policy' name='privacy' target='_blank' class='-link'>privacy policy</a> and <a href='https://stackoverflow.com/legal/cookie-policy' name='cookie' target='_blank' class='-link'>cookie policy</a><input type="hidden" name="legalLinksShown" value="1" />
                                                </p>
                                        </div>
                                        <div class="js-general-error general-error cbt d-none"></div>
                                </form>
    
    
    
                                <h2 class="bottom-notice" data-loc="1">
    Not the answer you're looking for? Browse other questions tagged <a href="/questions/tagged/javascript" class="post-tag" title="show questions tagged 'javascript'" rel="tag">javascript</a> <a href="/questions/tagged/html" class="post-tag" title="show questions tagged 'html'" rel="tag">html</a> <a href="/questions/tagged/iframe" class="post-tag" title="show questions tagged 'iframe'" rel="tag">iframe</a> <a href="/questions/tagged/web" class="post-tag" title="show questions tagged 'web'" rel="tag">web</a>  or <a href="/questions/ask">ask your own question</a>.                            </h2>
                    </div>
                </div>
                <div id="sidebar" class="show-votes" role="complementary" aria-label="sidebar">
                        
    
                    
    <div class="s-sidebarwidget s-sidebarwidget__yellow s-anchors s-anchors__grayscale mb16" data-tracker="cb=1">
        <ul class="d-block p0 m0">
                        <div class="s-sidebarwidget--header s-sidebarwidget__small-bold-text fc-light d:fc-black-900 bb bbw1">
                            The Overflow Blog
                        </div>
            <li class="s-sidebarwidget--item grid px16">
                <div class="grid--cell1 fl-shrink0">
    <svg aria-hidden="true" class="va-text-top svg-icon iconPencilSm" width="14" height="14" viewBox="0 0 14 14"><path d="M11.1 1.71l1.13 1.12c.2.2.2.51 0 .71L11.1 4.7 9.21 2.86l1.17-1.15c.2-.2.51-.2.71 0zM2 10.12l6.37-6.43 1.88 1.88L3.88 12H2v-1.88z"/></svg>            </div>
                <div class="grid--cell wmn0 ow-break-word">
                    <a href="https://stackoverflow.blog/2021/03/23/podcast-323-a-director-of-engineering-explains-scaling-from-dozens-of-employees-to-thousands/" class="js-gps-track" title="Podcast 323: A director of engineering explains scaling from dozens of employees to thousands" data-ga="["community bulletin board","The Overflow Blog","https://stackoverflow.blog/2021/03/23/podcast-323-a-director-of-engineering-explains-scaling-from-dozens-of-employees-to-thousands/",null,null]" data-gps-track="communitybulletin.click({ priority: 1, position: 0 })">Podcast 323: A director of engineering explains scaling from dozens of…</a>
                </div>
            </li>
            <li class="s-sidebarwidget--item grid px16">
                <div class="grid--cell1 fl-shrink0">
    <svg aria-hidden="true" class="va-text-top svg-icon iconPencilSm" width="14" height="14" viewBox="0 0 14 14"><path d="M11.1 1.71l1.13 1.12c.2.2.2.51 0 .71L11.1 4.7 9.21 2.86l1.17-1.15c.2-.2.51-.2.71 0zM2 10.12l6.37-6.43 1.88 1.88L3.88 12H2v-1.88z"/></svg>            </div>
                <div class="grid--cell wmn0 ow-break-word">
                    <a href="https://stackoverflow.blog/2021/03/23/level-up-creative-coding-with-p5-js-part-2/" class="js-gps-track" data-ga="["community bulletin board","The Overflow Blog","https://stackoverflow.blog/2021/03/23/level-up-creative-coding-with-p5-js-part-2/",null,null]" data-gps-track="communitybulletin.click({ priority: 1, position: 1 })">Level Up: creative coding with p5.js – part 2</a>
                </div>
            </li>
                        <div class="s-sidebarwidget--header s-sidebarwidget__small-bold-text fc-light d:fc-black-900 bb bbw1">
                            Featured on Meta
                        </div>
            <li class="s-sidebarwidget--item grid px16">
                <div class="grid--cell1 fl-shrink0">
    <div class="favicon favicon-stackexchangemeta" title="Meta Stack Exchange"></div>            </div>
                <div class="grid--cell wmn0 ow-break-word">
                    <a href="https://meta.stackexchange.com/questions/362203/stack-overflow-for-teams-is-now-free-for-up-to-50-users-forever" class="js-gps-track" data-ga="["community bulletin board","Featured on Meta","https://meta.stackexchange.com/questions/362203/stack-overflow-for-teams-is-now-free-for-up-to-50-users-forever",null,null]" data-gps-track="communitybulletin.click({ priority: 3, position: 2 })">Stack Overflow for Teams is now free for up to 50 users, forever</a>
                </div>
            </li>
            <li class="s-sidebarwidget--item grid px16">
                <div class="grid--cell1 fl-shrink0">
    <div class="favicon favicon-stackexchangemeta" title="Meta Stack Exchange"></div>            </div>
                <div class="grid--cell wmn0 ow-break-word">
                    <a href="https://meta.stackexchange.com/questions/362269/planned-maintenance-scheduled-for-saturday-march-27-2021-at-100-utc-friday-m" class="js-gps-track" title="Planned maintenance scheduled for Saturday, March 27, 2021 at 1:00 UTC (Friday March 26 9:00 PM US/EDT)" data-ga="["community bulletin board","Featured on Meta","https://meta.stackexchange.com/questions/362269/planned-maintenance-scheduled-for-saturday-march-27-2021-at-100-utc-friday-m",null,null]" data-gps-track="communitybulletin.click({ priority: 3, position: 3 })">Planned maintenance scheduled for Saturday, March 27, 2021 at 1:00 UTC…</a>
                </div>
            </li>
        </ul>
    </div>
    
    
    <div class="js-zone-container zone-container-sidebar">
        <div id="dfp-tsb" class="everyonelovesstackoverflow everyoneloves__top-sidebar"></div>
        <div class="js-report-ad-button-container " style="width: 300px"></div>
    </div>
    <div class="js-zone-container zone-container-sidebar">
        <div id="dfp-msb" class="everyonelovesstackoverflow everyoneloves__mid-sidebar"></div>
        <div class="js-report-ad-button-container " style="width: 300px"></div>
    </div>
    <div id="hireme"></div>                    <div class="module sidebar-linked">
    	<h4 id="h-linked">Linked</h4>
    	<div class="linked" data-tracker="lq=1">
                <div class="spacer js-gps-track" data-gps-track="linkedquestion.click({ source_post_id: 43381154, target_question_id: 6597224, position: 0 })">
    				<a href="/q/6597224" title="Vote score (upvotes - downvotes)">
    					<div class="answer-votes answered-accepted default">39</div>
    				</a>
    				<a href="/questions/6597224/how-to-hide-html-source-disable-right-click-and-text-copy?noredirect=1" class="question-hyperlink">How to hide html source & disable right click and text copy?</a>
    			</div>
                <div class="spacer js-gps-track" data-gps-track="linkedquestion.click({ source_post_id: 43381154, target_question_id: 42193700, position: 1 })">
    				<a href="/q/42193700" title="Vote score (upvotes - downvotes)">
    					<div class="answer-votes answered-accepted default">45</div>
    				</a>
    				<a href="/questions/42193700/detect-when-inspect-element-is-open?noredirect=1" class="question-hyperlink">Detect when “Inspect Element” is open</a>
    			</div>
                <div class="spacer js-gps-track" data-gps-track="linkedquestion.click({ source_post_id: 43381154, target_question_id: 31261667, position: 2 })">
    				<a href="/q/31261667" title="Vote score (upvotes - downvotes)">
    					<div class="answer-votes  default">9</div>
    				</a>
    				<a href="/questions/31261667/how-to-clear-the-javascript-console-programmatically?noredirect=1" class="question-hyperlink">How to clear the javascript console programmatically?</a>
    			</div>
    	</div>
    </div>
    
    
                        <div class="module sidebar-related">
                            <h4 id="h-related">Related</h4>
                            <div class="related js-gps-related-questions" data-tracker="rq=1">
                                    <div class="spacer">
                                        <a href="/q/105034" title="Vote score (upvotes - downvotes)">
                                            <div class="answer-votes answered-accepted extra-large">4496</div>
                                        </a>
                                        <a href="/questions/105034/how-to-create-a-guid-uuid" class="question-hyperlink">How to create a GUID / UUID</a>
                                    </div>
                                    <div class="spacer">
                                        <a href="/q/111102" title="Vote score (upvotes - downvotes)">
                                            <div class="answer-votes answered-accepted extra-large">7631</div>
                                        </a>
                                        <a href="/questions/111102/how-do-javascript-closures-work" class="question-hyperlink">How do JavaScript closures work?</a>
                                    </div>
                                    <div class="spacer">
                                        <a href="/q/195951" title="Vote score (upvotes - downvotes)">
                                            <div class="answer-votes answered-accepted extra-large">2946</div>
                                        </a>
                                        <a href="/questions/195951/how-can-i-change-an-elements-class-with-javascript" class="question-hyperlink">How can I change an element's class with JavaScript?</a>
                                    </div>
                                    <div class="spacer">
                                        <a href="/q/208105" title="Vote score (upvotes - downvotes)">
                                            <div class="answer-votes answered-accepted extra-large">6504</div>
                                        </a>
                                        <a href="/questions/208105/how-do-i-remove-a-property-from-a-javascript-object" class="question-hyperlink">How do I remove a property from a JavaScript object?</a>
                                    </div>
                                    <div class="spacer">
                                        <a href="/q/901712" title="Vote score (upvotes - downvotes)">
                                            <div class="answer-votes answered-accepted extra-large">4794</div>
                                        </a>
                                        <a href="/questions/901712/how-do-i-check-whether-a-checkbox-is-checked-in-jquery" class="question-hyperlink">How do I check whether a checkbox is checked in jQuery?</a>
                                    </div>
                                    <div class="spacer">
                                        <a href="/q/950087" title="Vote score (upvotes - downvotes)">
                                            <div class="answer-votes answered-accepted extra-large">5557</div>
                                        </a>
                                        <a href="/questions/950087/how-do-i-include-a-javascript-file-in-another-javascript-file" class="question-hyperlink">How do I include a JavaScript file in another JavaScript file?</a>
                                    </div>
                                    <div class="spacer">
                                        <a href="/q/1144783" title="Vote score (upvotes - downvotes)">
                                            <div class="answer-votes answered-accepted extra-large">4743</div>
                                        </a>
                                        <a href="/questions/1144783/how-to-replace-all-occurrences-of-a-string-in-javascript" class="question-hyperlink">How to replace all occurrences of a string in Javascript?</a>
                                    </div>
                                    <div class="spacer">
                                        <a href="/q/1789945" title="Vote score (upvotes - downvotes)">
                                            <div class="answer-votes answered-accepted extra-large">7421</div>
                                        </a>
                                        <a href="/questions/1789945/how-to-check-whether-a-string-contains-a-substring-in-javascript" class="question-hyperlink">How to check whether a string contains a substring in JavaScript?</a>
                                    </div>
                                    <div class="spacer">
                                        <a href="/q/5062614" title="Vote score (upvotes - downvotes)">
                                            <div class="answer-votes answered-accepted extra-large">2194</div>
                                        </a>
                                        <a href="/questions/5062614/how-to-decide-when-to-use-node-js" class="question-hyperlink">How to decide when to use Node.js?</a>
                                    </div>
                                    <div class="spacer">
                                        <a href="/q/5767325" title="Vote score (upvotes - downvotes)">
                                            <div class="answer-votes answered-accepted extra-large">9228</div>
                                        </a>
                                        <a href="/questions/5767325/how-can-i-remove-a-specific-item-from-an-array" class="question-hyperlink">How can I remove a specific item from an array?</a>
                                    </div>
                            </div>
                        </div>
    
                    <div id="hot-network-questions" class="module tex2jax_ignore">
        <h4>
            <a href="https://stackexchange.com/questions?tab=hot"
               class="js-gps-track s-link s-link__inherit" 
               data-gps-track="posts_hot_network.click({ item_type:1, location:11 })">
                Hot Network Questions
            </a>
        </h4>
        <ul>
                <li >
                    <div class="favicon favicon-japanese" title="Japanese Language Stack Exchange"></div><a href="https://japanese.stackexchange.com/questions/84740/what-is-the-difference-between-%e5%a4%96%e7%a7%91-and-%e6%89%8b%e8%a1%93" class="js-gps-track question-hyperlink mb0" data-gps-track="site.switch({ item_type:11, target_site:257 }); posts_hot_network.click({ item_type:2, location:11 })">
                        What is the difference between 外科 and 手術?
                    </a>
    
                </li>
                <li >
                    <div class="favicon favicon-astronomy" title="Astronomy Stack Exchange"></div><a href="https://astronomy.stackexchange.com/questions/42076/why-wont-the-sun-set-for-days-at-n66-2-which-is-below-the-arctic-circle" class="js-gps-track question-hyperlink mb0" data-gps-track="site.switch({ item_type:11, target_site:514 }); posts_hot_network.click({ item_type:2, location:11 })">
                        Why won't the Sun set for days at N66.2 which is below the arctic circle?
                    </a>
    
                </li>
                <li >
                    <div class="favicon favicon-unix" title="Unix & Linux Stack Exchange"></div><a href="https://unix.stackexchange.com/questions/640681/how-to-only-keep-line-that-start-with-a-character-and-the-line-after" class="js-gps-track question-hyperlink mb0" data-gps-track="site.switch({ item_type:11, target_site:106 }); posts_hot_network.click({ item_type:2, location:11 })">
                        How to only keep line that start with a character and the line after
                    </a>
    
                </li>
                <li >
                    <div class="favicon favicon-space" title="Space Exploration Stack Exchange"></div><a href="https://space.stackexchange.com/questions/50980/astronauts-make-a-lot-of-ch%e2%82%84-and-some-h%e2%82%82-as-well-do-space-capsules-and-space-st" class="js-gps-track question-hyperlink mb0" data-gps-track="site.switch({ item_type:11, target_site:508 }); posts_hot_network.click({ item_type:2, location:11 })">
                        Astronauts make a lot of CH₄ and some H₂ as well; do space capsules and space stations have systems to remove these?
                    </a>
    
                </li>
                <li >
                    <div class="favicon favicon-skeptics" title="Skeptics Stack Exchange"></div><a href="https://skeptics.stackexchange.com/questions/50490/does-sci-hub-use-malware-and-phishing-to-obtain-researchers-login-credentials" class="js-gps-track question-hyperlink mb0" data-gps-track="site.switch({ item_type:11, target_site:212 }); posts_hot_network.click({ item_type:2, location:11 })">
                        Does Sci-Hub use malware and phishing to obtain researchers' login credentials?
                    </a>
    
                </li>
                <li class="dno js-hidden">
                    <div class="favicon favicon-unix" title="Unix & Linux Stack Exchange"></div><a href="https://unix.stackexchange.com/questions/640457/how-can-i-prevent-the-second-command-in-a-chain-while-keeping-the-first-one-runn" class="js-gps-track question-hyperlink mb0" data-gps-track="site.switch({ item_type:11, target_site:106 }); posts_hot_network.click({ item_type:2, location:11 })">
                        How can I prevent the second command in a chain while keeping the first one running?
                    </a>
    
                </li>
                <li class="dno js-hidden">
                    <div class="favicon favicon-serverfault" title="Server Fault"></div><a href="https://serverfault.com/questions/1057780/dns-mx-spf-dmarc-records-without-actuall-emails-on-domain" class="js-gps-track question-hyperlink mb0" data-gps-track="site.switch({ item_type:11, target_site:2 }); posts_hot_network.click({ item_type:2, location:11 })">
                        DNS MX/SPF/DMARC records without actuall emails on domain
                    </a>
    
                </li>
                <li class="dno js-hidden">
                    <div class="favicon favicon-worldbuilding" title="Worldbuilding Stack Exchange"></div><a href="https://worldbuilding.stackexchange.com/questions/198188/what-would-it-have-taken-for-navies-to-control-air-forces-historically" class="js-gps-track question-hyperlink mb0" data-gps-track="site.switch({ item_type:11, target_site:579 }); posts_hot_network.click({ item_type:2, location:11 })">
                        What would it have taken for navies to control air forces historically?
                    </a>
    
                </li>
                <li class="dno js-hidden">
                    <div class="favicon favicon-superuser" title="Super User"></div><a href="https://superuser.com/questions/1635090/what-happened-to-ecc-ram" class="js-gps-track question-hyperlink mb0" data-gps-track="site.switch({ item_type:11, target_site:3 }); posts_hot_network.click({ item_type:2, location:11 })">
                        What happened to ECC RAM?
                    </a>
    
                </li>
                <li class="dno js-hidden">
                    <div class="favicon favicon-academia" title="Academia Stack Exchange"></div><a href="https://academia.stackexchange.com/questions/164251/students-admit-illicit-behavior-in-private-communication-how-should-i-proceed" class="js-gps-track question-hyperlink mb0" data-gps-track="site.switch({ item_type:11, target_site:415 }); posts_hot_network.click({ item_type:2, location:11 })">
                        Students admit illicit behavior in private communication: how should I proceed?
                    </a>
    
                </li>
                <li class="dno js-hidden">
                    <div class="favicon favicon-travel" title="Travel Stack Exchange"></div><a href="https://travel.stackexchange.com/questions/162982/my-bus-hasnt-left-salem-for-several-hours-no-eta-updated-its-hours-behind-s" class="js-gps-track question-hyperlink mb0" data-gps-track="site.switch({ item_type:11, target_site:273 }); posts_hot_network.click({ item_type:2, location:11 })">
                        My bus hasn't left Salem for several hours, no ETA updated - It's hours behind schedule, What should I do?
                    </a>
    
                </li>
                <li class="dno js-hidden">
                    <div class="favicon favicon-worldbuilding" title="Worldbuilding Stack Exchange"></div><a href="https://worldbuilding.stackexchange.com/questions/198183/how-would-amazon-a-female-only-subspecies-genetics-work" class="js-gps-track question-hyperlink mb0" data-gps-track="site.switch({ item_type:11, target_site:579 }); posts_hot_network.click({ item_type:2, location:11 })">
                        How would "Amazon" (a female only subspecies) genetics work?
                    </a>
    
                </li>
                <li class="dno js-hidden">
                    <div class="favicon favicon-ell" title="English Language Learners Stack Exchange"></div><a href="https://ell.stackexchange.com/questions/278442/what-would-you-call-for-the-lack-of-a-better-way-to-put-it-benign-nationalism" class="js-gps-track question-hyperlink mb0" data-gps-track="site.switch({ item_type:11, target_site:481 }); posts_hot_network.click({ item_type:2, location:11 })">
                        What would you call, for the lack of a better way to put it, "benign nationalism"?
                    </a>
    
                </li>
                <li class="dno js-hidden">
                    <div class="favicon favicon-music" title="Music: Practice & Theory Stack Exchange"></div><a href="https://music.stackexchange.com/questions/112093/what-are-good-beginner-pedals" class="js-gps-track question-hyperlink mb0" data-gps-track="site.switch({ item_type:11, target_site:240 }); posts_hot_network.click({ item_type:2, location:11 })">
                        What are good beginner pedals
                    </a>
    
                </li>
                <li class="dno js-hidden">
                    <div class="favicon favicon-electronics" title="Electrical Engineering Stack Exchange"></div><a href="https://electronics.stackexchange.com/questions/554441/why-do-consumer-ovens-use-thermostats-instead-of-pid-pwm" class="js-gps-track question-hyperlink mb0" data-gps-track="site.switch({ item_type:11, target_site:135 }); posts_hot_network.click({ item_type:2, location:11 })">
                        Why do consumer ovens use thermostats instead of PID + PWM?
                    </a>
    
                </li>
                <li class="dno js-hidden">
                    <div class="favicon favicon-android" title="Android Enthusiasts Stack Exchange"></div><a href="https://android.stackexchange.com/questions/234830/android-com-google-android-trichromelibrary-causes-other-apps-to-crash" class="js-gps-track question-hyperlink mb0" data-gps-track="site.switch({ item_type:11, target_site:139 }); posts_hot_network.click({ item_type:2, location:11 })">
                        Android com.google.android.trichromelibrary causes other apps to crash
                    </a>
    
                </li>
                <li class="dno js-hidden">
                    <div class="favicon favicon-mathematica" title="Mathematica Stack Exchange"></div><a href="https://mathematica.stackexchange.com/questions/242297/copy-graphical-style-of-slider2d" class="js-gps-track question-hyperlink mb0" data-gps-track="site.switch({ item_type:11, target_site:387 }); posts_hot_network.click({ item_type:2, location:11 })">
                        Copy graphical style of Slider2D
                    </a>
    
                </li>
                <li class="dno js-hidden">
                    <div class="favicon favicon-askubuntu" title="Ask Ubuntu"></div><a href="https://askubuntu.com/questions/1325479/how-to-rename-applications-in-show-apps-such-as-gnu-image-manipulation-progr" class="js-gps-track question-hyperlink mb0" data-gps-track="site.switch({ item_type:11, target_site:89 }); posts_hot_network.click({ item_type:2, location:11 })">
                        how to rename applications in "show apps", such as "GNU Image Manipulation Program" to "GIMP" 20.04.2?
                    </a>
    
                </li>
                <li class="dno js-hidden">
                    <div class="favicon favicon-chemistry" title="Chemistry Stack Exchange"></div><a href="https://chemistry.stackexchange.com/questions/147952/iupac-name-of-haucl4" class="js-gps-track question-hyperlink mb0" data-gps-track="site.switch({ item_type:11, target_site:431 }); posts_hot_network.click({ item_type:2, location:11 })">
                        IUPAC name of H[AuCl4]
                    </a>
    
                </li>
                <li class="dno js-hidden">
                    <div class="favicon favicon-chess" title="Chess Stack Exchange"></div><a href="https://chess.stackexchange.com/questions/34576/what-does-white-is-down-a-rook-for-a-pawn-mean" class="js-gps-track question-hyperlink mb0" data-gps-track="site.switch({ item_type:11, target_site:435 }); posts_hot_network.click({ item_type:2, location:11 })">
                        What does "White is down a rook for a pawn" mean?
                    </a>
    
                </li>
                <li class="dno js-hidden">
                    <div class="favicon favicon-money" title="Personal Finance & Money Stack Exchange"></div><a href="https://money.stackexchange.com/questions/137967/formula-like-kelly-criterion-for-choosing-how-much-of-an-investment-to-liquidate" class="js-gps-track question-hyperlink mb0" data-gps-track="site.switch({ item_type:11, target_site:93 }); posts_hot_network.click({ item_type:2, location:11 })">
                        Formula like Kelly criterion for choosing how much of an investment to liquidate?
                    </a>
    
                </li>
                <li class="dno js-hidden">
                    <div class="favicon favicon-mathoverflow" title="MathOverflow"></div><a href="https://mathoverflow.net/questions/387142/where-do-some-energy-identities-in-pde-theory-come-from" class="js-gps-track question-hyperlink mb0" data-gps-track="site.switch({ item_type:11, target_site:504 }); posts_hot_network.click({ item_type:2, location:11 })">
                        Where do some "energy identities" in PDE theory come from?
                    </a>
    
                </li>
                <li class="dno js-hidden">
                    <div class="favicon favicon-codegolf" title="Code Golf Stack Exchange"></div><a href="https://codegolf.stackexchange.com/questions/221110/counting-polydominoes" class="js-gps-track question-hyperlink mb0" data-gps-track="site.switch({ item_type:11, target_site:200 }); posts_hot_network.click({ item_type:2, location:11 })">
                        Counting polydominoes
                    </a>
    
                </li>
                <li class="dno js-hidden">
                    <div class="favicon favicon-dba" title="Database Administrators Stack Exchange"></div><a href="https://dba.stackexchange.com/questions/287475/sql-server-instance-how-to-recover-memory-used" class="js-gps-track question-hyperlink mb0" data-gps-track="site.switch({ item_type:11, target_site:182 }); posts_hot_network.click({ item_type:2, location:11 })">
                        SQL Server Instance How to recover memory used
                    </a>
    
                </li>
        </ul>
    
            <a href="#" 
               class="show-more js-show-more js-gps-track" 
               data-gps-track="posts_hot_network.click({ item_type:3, location:11 })">
                more hot questions
            </a>
    </div>
    
                                <div id="feed-link" class="js-feed-link">
            <a href="/feeds/question/43381154" title="Feed of this question and its answers">
                <svg aria-hidden="true" class="fc-orange-400 svg-icon iconRss" width="18" height="18" viewBox="0 0 18 18"><path d="M1 3c0-1.1.9-2 2-2h12a2 2 0 012 2v12a2 2 0 01-2 2H3a2 2 0 01-2-2V3zm14.5 12C15.5 8.1 9.9 2.5 3 2.5V5a10 10 0 0110 10h2.5zm-5 0A7.5 7.5 0 003 7.5V10a5 5 0 015 5h2.5zm-5 0A2.5 2.5 0 003 12.5V15h2.5z"/></svg>
                Question feed
            </a>
        </div>
        <aside class="s-modal js-feed-link-modal" tabindex="-1" role="dialog" aria-labelledby="feed-modal-title" aria-describedby="feed-modal-description" aria-hidden="true">
            <div class="s-modal--dialog js-modal-dialog wmx4" role="document"  data-controller="se-draggable">
                <h1 class="s-modal--header fw-bold js-first-tabbable" id="feed-modal-title" data-target="se-draggable.handle" tabindex="0">
                    Subscribe to RSS
                </h1>
                <div class="grid gs4 gsy fd-column">
                    <div class="grid--cell">
                        <label class="d-block s-label c-default" for="feed-url">
                            Question feed
                            <p class="s-description mt2" id="feed-modal-description">To subscribe to this RSS feed, copy and paste this URL into your RSS reader.</p>
                        </label>
                    </div>
                    <div class="grid ps-relative">
                        <input class="s-input" type="text" name="feed-url" id="feed-url" readonly="readonly" value="https://stackoverflow.com/feeds/question/43381154" />
                        <svg aria-hidden="true" class="s-input-icon fc-orange-400 svg-icon iconRss" width="18" height="18" viewBox="0 0 18 18"><path d="M1 3c0-1.1.9-2 2-2h12a2 2 0 012 2v12a2 2 0 01-2 2H3a2 2 0 01-2-2V3zm14.5 12C15.5 8.1 9.9 2.5 3 2.5V5a10 10 0 0110 10h2.5zm-5 0A7.5 7.5 0 003 7.5V10a5 5 0 015 5h2.5zm-5 0A2.5 2.5 0 003 12.5V15h2.5z"/></svg>
                    </div>
                </div>
                <a class="s-modal--close s-btn s-btn__muted js-modal-close js-last-tabbable" href="#" aria-label="Close">
                    <svg aria-hidden="true" class="svg-icon iconClearSm" width="14" height="14" viewBox="0 0 14 14"><path d="M12 3.41L10.59 2 7 5.59 3.41 2 2 3.41 5.59 7 2 10.59 3.41 12 7 8.41 10.59 12 12 10.59 8.41 7 12 3.41z"/></svg>
                </a>
            </div>
        </aside>
    
                </div>
        </div>
    <script>StackExchange.ready(function(){$.get('/posts/43381154/ivc/b20a');});</script>
    <noscript><div><img src="/posts/43381154/ivc/b20a" class="dno" alt="" width="0" height="0"></div></noscript><div style="display:none" id="js-codeblock-lang">default</div></div>
    
    
            </div>
        </div>
            
    <script>try{(function(a){function b(a){return'string'==typeof a?document.getElementById(a):a}function c(a){return a=b(a),!!a&&'none'===getComputedStyle(a).display}function d(a){return!c(a)}function e(a){return!!a}function f(a){return /^\s*$/.test(b(a).innerHTML)}function g(a){var b=a.style;b.height=b.maxHeight=b.minHeight='auto',b.display='none'}function h(a){var b=a.style;b.height=b.maxHeight=b.minHeight='auto',b.display='none',[].forEach.call(a.children,h)}function i(a){var b=a.style;b.height=b.maxHeight=b.minHeight='auto',b.removeProperty('display')}function j(a,b){var c;return function(){return a&&(c=a.call(b||this,arguments),a=null),c}}function k(a){var b=document.createElement('script');b.src=a,document.body.appendChild(b)}function l(a){return m([],a)}function m(a,b){return a.push=function(a){return b(),delete this.push,this.push(a)},a}function n(){try{return!new Function('return async()=>{};')}catch(a){return!0}}function o(){return'undefined'!=typeof googletag&&!!googletag.apiReady}function p(){o()||(googletag={cmd:l(B)})}function q(){var a=document.createElement('div');a.className='adsbox',a.id='clc-abd',a.style.position='absolute',a.style.pointerEvents='none',a.innerHTML=' ',document.body.appendChild(a)}function r(){return Object.keys(F.ids).filter(function(a){return'clc-cpa'!=a})}function s(a){var b=a.split('_')[0],c=F.ids[b],d=F.slots[c];'function'==typeof d&&(d=d(b));return{path:'/'+C+'/'+E+'/'+c+'/'+D,sizes:d,zone:c}}function t(a){try{Array.isArray(clc.dfp.slotsRenderedEvents)||(clc.dfp.slotsRenderedEvents=[]),clc.dfp.slotsRenderedEvents.push(a);var b=a.slot.getSlotElementId(),c=[];b||c.push('id=0');var d=document.getElementById(b);if(!b||d?d.hasAttribute('data-clc-stalled')&&c.push('st=1'):c.push('el=0'),0!==c.length)return void G(c.join('&'));var e=s(b),f=e.zone;if(clc.collapse&&clc.collapse[f]&&a.isEmpty)return h(d),void d.setAttribute('data-clc-ready','true');if(-1!==y.dh.indexOf(a.lineItemId))h(d);else if(a.lineItemId){d.setAttribute('data-clc-prefilled','true');var j=d.parentElement;if(j.classList.contains('js-zone-container')){g(j);var k=j.querySelectorAll('.js-report-ad-button-container'),l=k[0];switch(l.style.height='24px',b){case'dfp-tlb':case'dfp-tag':{j.classList.add('mb8');break}case'dfp-mlb':case'dfp-smlb':case'dfp-bmlb':{j.classList.add('my8');break}case'dfp-isb':{j.classList.add('mt24');break}case'dfp-m-aq':{j.classList.add('my12'),j.classList.add('mx-auto');break}default:}i(j),i(d)}else i(d);if('dfp-msb'==b){var m=document.getElementById('hireme');h(m)}}d.setAttribute('data-clc-ready','true')}catch(a){var n=document.querySelector('#dfp-tsb, #dfp-isb, #clc-tsb');n&&n.setAttribute('data-clc-ready','true'),G('e=1')}}function u(a,b){'dfp-isb'===a&&b.setTargeting('Sidebar',['Inline']),'dfp-tsb'===a&&b.setTargeting('Sidebar',['Right']);var c=s(a),d=c.path,e=c.sizes,f=c.zone,g=googletag.defineSlot(d,e,a);g.addService(b),!1;var h=a.split('_');if('clc-cpa'==h[0]&&h[1]){var i=h[1];g.setTargeting('talent-company-id',i)}}function v(b){var c=a.dfp&&a.dfp.targeting||{};'SystemDefault'===c.ProductVariant&&(window.matchMedia&&window.matchMedia('(prefers-color-scheme: dark)').matches?c.ProductVariant='Dark':c.ProductVariant='Light'),Object.keys(c).forEach(function(a){b.setTargeting(a,c[a])})}function w(a){var g=a.map(b).filter(e);return{eligible:g.filter(f).filter(d),ineligible:g.filter(c)}}function x(b){void 0===b&&(b=r());var c=['dfp-mlb','dfp-smlb'];if(!o())return p(),void googletag.cmd.push(function(){return x(b)});var d=w(b),e=d.eligible,f=d.ineligible;if(e.forEach(function(a){g(a)}),f.forEach(h),0!==e.length){y.abd&&q();var i=googletag.pubads().getSlots(),j=i.filter(function(a){return 0<=b.indexOf(a.getSlotElementId())});googletag.destroySlots(j);var k=googletag.pubads();y.sf&&(k.setForceSafeFrame(!0),k.setSafeFrameConfig({allowOverlayExpansion:!0,allowPushExpansion:!0,sandbox:!0})),'undefined'!=typeof y.targeting_consent&&(k.setRequestNonPersonalizedAds(y.targeting_consent?0:1),!y.targeting_consent&&k.setPrivacySettings({limitedAds:!0})),y.ll||k.enableSingleRequest(),a.sreEvent||(k.addEventListener('slotRenderEnded',t),a.sreEvent=!0),v(k);var l=e.filter(function(a){return!y.ll||0>c.indexOf(a.id)}),m=e.filter(function(a){return!!y.ll&&0<=c.indexOf(a.id)});l.forEach(function(a){u(a.id,k),a.setAttribute('data-dfp-zone','true')}),googletag.enableServices(),l.forEach(function(a){googletag.display(a.id)}),y.ll&&(k.enableLazyLoad({fetchMarginPercent:0,renderMarginPercent:0}),m.forEach(function(a){u(a.id,k),a.setAttribute('data-clc-prefilled','true')}),m.forEach(function(a){googletag.display(a.id)}))}}var y=function(a){for(var b=[],c=1;c<arguments.length;c++)b[c-1]=arguments[c];for(var d,e=0,f=b;e<f.length;e++)for(var g in d=f[e],d)a[g]=d[g];return a}({"lib":"https://cdn.sstatic.net/clc/clc.min.js?v=0e634710bdce","style":"https://cdn.sstatic.net/clc/styles/clc.min.css?v=69a63cb1f209","u":"https://clc.stackoverflow.com/markup.js","wa":true,"kt":2000,"tto":true,"h":"clc.stackoverflow.com","allowed":"^(((talent\\.)?stackoverflow)|(blog\\.codinghorror)|(serverfault|askubuntu)|([^\\.]+\\.stackexchange))\\.com$","wv":true,"al":false,"dh":[5171832659],"abd":true},a.options||{}),z=j(function(){var a=y.lib;n()&&(a=a.replace(/(\.min)?\.js(\?v=[0-9a-fA-F]+)?$/,'.ie$1.js$2')),k(a)}),A=a.cmd||[];Array.isArray(A)&&(0<A.length?z():m(A,z));var B=j(function(){y.targeting_consent||'undefined'==typeof y.targeting_consent?k('https://securepubads.g.doubleclick.net/tag/js/gpt.js'):k('https://pagead2.googlesyndication.com/tag/js/gpt.js')}),C='248424177',D=/^\/tags\//.test(location.pathname)||/^\/questions\/tagged\//.test(location.pathname)?'tag-pages':/^\/$/.test(location.pathname)||/^\/home/.test(location.pathname)?'home-page':'question-pages',E=location.hostname;var F={slots:{lb:[[728,90]],mlb:[[728,90]],smlb:[[728,90]],bmlb:[[728,90]],sb:function(a){return'dfp-tsb'===a?[[300,250],[300,600]]:[[300,250]]},"tag-sponsorship":[[730,135]],"mobile-below-question":[[320,50],[300,250]],msb:[[300,250],[300,600]],"talent-conversion-tracking":[[1,1]]},ids:{"dfp-tlb":'lb',"dfp-mlb":'mlb',"dfp-smlb":'smlb',"dfp-bmlb":'bmlb',"dfp-tsb":'sb',"dfp-isb":'sb',"dfp-tag":'tag-sponsorship',"dfp-msb":'msb',"dfp-m-aq":'mobile-below-question',"clc-tlb":'lb',"clc-mlb":'mlb',"clc-tsb":'sb',"clc-cpa":'talent-conversion-tracking'}},G=function(a){new Image().src='https://'+y.h+'/stalled.gif?'+a};(function(){var b=y.al;b&&A.push(function(){return a.load()})})(),p(),a.dfp={load:x},a.options=y,a.cmd=A})(this.clc=this.clc||{})}catch(a){window.console.error(a)}</script><script>
        var clc = clc || {};
        clc.collapse = { sb: !0, 'tag-sponsorship': !0, lb: !0, mlb: !0, smlb: !0, bmlb: !0, 'mobile-below-question': !0 };
        clc.options = clc.options || {};
        clc.options.sf = !0;
        clc.options.hb = !1;
        clc.options.ll = !0;
        clc.options.targeting_consent = !0;
        clc.options.performance_consent = !0;
            clc.cmd = clc.cmd || [];
            clc.cmd.push(function () { window.clc_request='Ayfu3B877tgIAAAAAKLxlQICAAAAAgAAAAABHAAAAHxqYXZhc2NyaXB0fGh0bWx8aWZyYW1lfHdlYnwAm3ubkvBbeAxzcw'; clc.load(); });
            clc.dfp = clc.dfp || {};
            clc.dfp.targeting = {Registered:['false'],'so-tag':['javascript','html','iframe','web'],'tag-reportable':['javascript','html','iframe','web'],'tag-non-reportable':['javascript','html','iframe','web'],NumberOfAnswers:['8']};
            clc.dfp.targeting.TargetingConsent = ['true'];
    
            const urlParams = new URLSearchParams(window.location.search);
            if (urlParams.has('dfptestads')) {
                const dfptestads = urlParams.get('dfptestads');
                clc.dfp.targeting.DfpTestAds = dfptestads;
            }
            
            var googletag = googletag || {};
            googletag.cmd = googletag.cmd || [];
            googletag.cmd.push(function () { clc.dfp.load(); });
            StackExchange.ready(function () { googletag.cmd.push(function () { StackExchange.ads.init(googletag, '/ads/report-ad', 'Report this ad') }) });
    </script>
    
                <footer id="footer" class="site-footer js-footer" role="contentinfo">
            <div class="site-footer--container">
                    <div class="site-footer--logo">
                        
                        <a href="https://stackoverflow.com"><svg aria-hidden="true" class="native svg-icon iconLogoGlyphMd" width="32" height="37" viewBox="0 0 32 37"><path d="M26 33v-9h4v13H0V24h4v9h22z" fill="#BCBBBB"/><path d="M21.5 0l-2.7 2 9.9 13.3 2.7-2L21.5 0zM26 18.4L13.3 7.8l2.1-2.5 12.7 10.6-2.1 2.5zM9.1 15.2l15 7 1.4-3-15-7-1.4 3zm14 10.79l.68-2.95-16.1-3.35L7 23l16.1 2.99zM23 30H7v-3h16v3z" fill="#F48024"/></svg></a>
                    </div>
                <nav class="site-footer--nav">
                        <div class="site-footer--col site-footer--col__visible js-footer-col" data-name="default">
                            <h5 class="-title"><a href="https://stackoverflow.com" class="js-gps-track" data-gps-track="footer.click({ location: 2, link: 15})">Stack Overflow</a></h5>
                            <ul class="-list js-primary-footer-links">
                                <li class="-item"><a href="/questions" class="js-gps-track -link" data-gps-track="footer.click({ location: 2, link: 16})">Questions</a></li>
                                    <li class="-item"><a href="https://stackoverflow.com/jobs" class="js-gps-track -link" data-gps-track="footer.click({ location: 2, link: 17})">Jobs</a></li>
                                    <li class="-item"><a href="https://stackoverflow.com/jobs/directory/developer-jobs" class="js-gps-track -link" data-gps-track="footer.click({ location: 2, link: 11})">Developer Jobs Directory</a></li>
                                         <li class="-item"><a href="https://stackoverflow.com/jobs/salary" class="js-gps-track -link" data-gps-track="footer.click({ location: 2, link: 28})">Salary Calculator</a></li>
                                    <li class="-item"><a href="/help" class="js-gps-track -link" data-gps-track="footer.click({ location: 2, link: 3 })">Help</a></li>
                                    <li class="-item"><a onclick='StackExchange.switchMobile("on")' class="js-gps-track -link" data-gps-track="footer.click({ location: 2, link: 12 })">Mobile</a></li>
                            </ul>
                        </div>
                        <div class="site-footer--col site-footer--col__visible js-footer-col" data-name="default">
                            <h5 class="-title"><a href="https://stackoverflowbusiness.com" class="js-gps-track" data-gps-track="footer.click({ location: 2, link: 19 })">Products</a></h5>
                            <ul class="-list">
                                <li class="-item"><a href="https://stackoverflow.com/teams" class="js-gps-track -link" 
                                                     data-ga="["teams traffic","footer - site nav","stackoverflow.com/teams",null,{"dimension4":"teams"}]"
                                                     data-gps-track="footer.click({ location: 2, link: 29 })">Teams</a></li>
                                <li class="-item"><a href="https://stackoverflow.com/talent" class="js-gps-track -link" data-gps-track="footer.click({ location: 2, link: 20 })">Talent</a></li>
                                <li class="-item"><a href="https://stackoverflow.com/advertising" class="js-gps-track -link" data-gps-track="footer.click({ location: 2, link: 21 })">Advertising</a></li>
                                <li class="-item"><a href="https://stackoverflow.com/enterprise" class="js-gps-track -link" data-gps-track="footer.click({ location: 2, link: 22 })">Enterprise</a></li>
                            </ul>
                        </div>
                    <div class="site-footer--col site-footer--col__visible js-footer-col" data-name="default">
                        <h5 class="-title"><a class="js-gps-track" data-gps-track="footer.click({ location: 2, link: 1 })" href="https://stackoverflow.com/company">Company</a></h5>
                        <ul class="-list">
                                <li class="-item"><a class="js-gps-track -link" data-gps-track="footer.click({ location: 2, link: 1 })" href="https://stackoverflow.com/company">About</a></li>
                            <li class="-item"><a class="js-gps-track -link" data-gps-track="footer.click({ location: 2, link: 27 })" href="https://stackoverflow.com/company/press">Press</a></li>
                                <li class="-item"><a class="js-gps-track -link" data-gps-track="footer.click({ location: 2, link: 9 })" href="https://stackoverflow.com/company/work-here">Work Here</a></li>
                            <li class="-item"><a class="js-gps-track -link" data-gps-track="footer.click({ location: 2, link: 7 })" href="https://stackoverflow.com/legal">Legal</a></li>
                            <li class="-item"><a class="js-gps-track -link" data-gps-track="footer.click({ location: 2, link: 8 })" href="https://stackoverflow.com/legal/privacy-policy">Privacy Policy</a></li>
                            <li class="-item"><a class="js-gps-track -link" data-gps-track="footer.click({ location: 2, link: 37 })" href="https://stackoverflow.com/legal/terms-of-service">Terms of Service</a></li>
                                <li class="-item"><a class="js-gps-track -link" data-gps-track="footer.click({ location: 2, link: 13 })" href="https://stackoverflow.com/company/contact">Contact Us</a></li>
                            <li class="-item"><a class="js-gps-track -link" data-gps-track="footer.click({ location: 2, link: 39 })" href="https://stackoverflow.com/legal/cookie-policy">Cookie Policy</a></li>
                        </ul>
                    </div>
                    <div class="site-footer--col site-footer--categories-nav site-footer--col__visible">
                        <a href="#" class="site-footer--back js-footer-back"><svg aria-hidden="true" class="svg-icon iconArrowLeftAlt" width="18" height="18" viewBox="0 0 18 18"><path d="M10.58 16L12 14.59 6.4 9 12 3.41 10.57 2l-7 7 7 7z"/></svg></a>
                        <div>
                            <h5 class="-title"><a href="https://stackexchange.com" data-gps-track="footer.click({ location: 2, link: 30 })">Stack Exchange<br> Network</a></h5>
                            <ul class="-list">
                                <li class="-item"><a href="#" class="-link _expandable js-footer-category-trigger js-gps-track" data-gps-track="footer.click({ location: 2, link: 24 })" data-target="Technology">Technology</a></li>
                                <li class="-item"><a href="#" class="-link _expandable js-footer-category-trigger js-gps-track" data-gps-track="footer.click({ location: 2, link: 24 })" data-target="Life / Arts">Life / Arts</a></li>
                                <li class="-item"><a href="#" class="-link _expandable js-footer-category-trigger js-gps-track" data-gps-track="footer.click({ location: 2, link: 24 })" data-target="Culture / Recreation">Culture / Recreation</a></li>
                                <li class="-item"><a href="#" class="-link _expandable js-footer-category-trigger js-gps-track" data-gps-track="footer.click({ location: 2, link: 24 })" data-target="Science">Science</a></li>
                                <li class="-item"><a href="#" class="-link _expandable js-footer-category-trigger js-gps-track" data-gps-track="footer.click({ location: 2, link: 24 })" data-target="Other">Other</a></li>
                            </ul>
                        </div>
                    </div>
                    <div class="site-footer--categories">
                            <div class="site-footer--col site-footer--category js-footer-col" data-name="Technology">
            <ul class="-list">
                    <li class="-item"><a href="https://stackoverflow.com" class="-link js-gps-track" data-gps-track="footer.click({ location: 1, link: 25 })" title="professional and enthusiast programmers">Stack Overflow</a></li>
                    <li class="-item"><a href="https://serverfault.com" class="-link js-gps-track" data-gps-track="footer.click({ location: 1, link: 25 })" title="system and network administrators">Server Fault</a></li>
                    <li class="-item"><a href="https://superuser.com" class="-link js-gps-track" data-gps-track="footer.click({ location: 1, link: 25 })" title="computer enthusiasts and power users">Super User</a></li>
                    <li class="-item"><a href="https://webapps.stackexchange.com" class="-link js-gps-track" data-gps-track="footer.click({ location: 1, link: 25 })" title="power users of web applications">Web Applications</a></li>
                    <li class="-item"><a href="https://askubuntu.com" class="-link js-gps-track" data-gps-track="footer.click({ location: 1, link: 25 })" title="Ubuntu users and developers">Ask Ubuntu</a></li>
                    <li class="-item"><a href="https://webmasters.stackexchange.com" class="-link js-gps-track" data-gps-track="footer.click({ location: 1, link: 25 })" title="pro webmasters">Webmasters</a></li>
                    <li class="-item"><a href="https://gamedev.stackexchange.com" class="-link js-gps-track" data-gps-track="footer.click({ location: 1, link: 25 })" title="professional and independent game developers">Game Development</a></li>
                        </ul></div><div class="site-footer--col site-footer--category js-footer-col" data-name="Technology"><ul class="-list">
                    <li class="-item"><a href="https://tex.stackexchange.com" class="-link js-gps-track" data-gps-track="footer.click({ location: 1, link: 25 })" title="users of TeX, LaTeX, ConTeXt, and related typesetting systems">TeX - LaTeX</a></li>
                    <li class="-item"><a href="https://softwareengineering.stackexchange.com" class="-link js-gps-track" data-gps-track="footer.click({ location: 1, link: 25 })" title="professionals, academics, and students working within the systems development life cycle">Software Engineering</a></li>
                    <li class="-item"><a href="https://unix.stackexchange.com" class="-link js-gps-track" data-gps-track="footer.click({ location: 1, link: 25 })" title="users of Linux, FreeBSD and other Un*x-like operating systems">Unix & Linux</a></li>
                    <li class="-item"><a href="https://apple.stackexchange.com" class="-link js-gps-track" data-gps-track="footer.click({ location: 1, link: 25 })" title="power users of Apple hardware and software">Ask Different (Apple)</a></li>
                    <li class="-item"><a href="https://wordpress.stackexchange.com" class="-link js-gps-track" data-gps-track="footer.click({ location: 1, link: 25 })" title="WordPress developers and administrators">WordPress Development</a></li>
                    <li class="-item"><a href="https://gis.stackexchange.com" class="-link js-gps-track" data-gps-track="footer.click({ location: 1, link: 25 })" title="cartographers, geographers and GIS professionals">Geographic Information Systems</a></li>
                    <li class="-item"><a href="https://electronics.stackexchange.com" class="-link js-gps-track" data-gps-track="footer.click({ location: 1, link: 25 })" title="electronics and electrical engineering professionals, students, and enthusiasts">Electrical Engineering</a></li>
                        </ul></div><div class="site-footer--col site-footer--category js-footer-col" data-name="Technology"><ul class="-list">
                    <li class="-item"><a href="https://android.stackexchange.com" class="-link js-gps-track" data-gps-track="footer.click({ location: 1, link: 25 })" title="enthusiasts and power users of the Android operating system">Android Enthusiasts</a></li>
                    <li class="-item"><a href="https://security.stackexchange.com" class="-link js-gps-track" data-gps-track="footer.click({ location: 1, link: 25 })" title="information security professionals">Information Security</a></li>
                    <li class="-item"><a href="https://dba.stackexchange.com" class="-link js-gps-track" data-gps-track="footer.click({ location: 1, link: 25 })" title="database professionals who wish to improve their database skills and learn from others in the community">Database Administrators</a></li>
                    <li class="-item"><a href="https://drupal.stackexchange.com" class="-link js-gps-track" data-gps-track="footer.click({ location: 1, link: 25 })" title="Drupal developers and administrators">Drupal Answers</a></li>
                    <li class="-item"><a href="https://sharepoint.stackexchange.com" class="-link js-gps-track" data-gps-track="footer.click({ location: 1, link: 25 })" title="SharePoint enthusiasts">SharePoint</a></li>
                    <li class="-item"><a href="https://ux.stackexchange.com" class="-link js-gps-track" data-gps-track="footer.click({ location: 1, link: 25 })" title="user experience researchers and experts">User Experience</a></li>
                    <li class="-item"><a href="https://mathematica.stackexchange.com" class="-link js-gps-track" data-gps-track="footer.click({ location: 1, link: 25 })" title="users of Wolfram Mathematica">Mathematica</a></li>
                        </ul></div><div class="site-footer--col site-footer--category js-footer-col" data-name="Technology"><ul class="-list">
                    <li class="-item"><a href="https://salesforce.stackexchange.com" class="-link js-gps-track" data-gps-track="footer.click({ location: 1, link: 25 })" title="Salesforce administrators, implementation experts, developers and anybody in-between">Salesforce</a></li>
                    <li class="-item"><a href="https://expressionengine.stackexchange.com" class="-link js-gps-track" data-gps-track="footer.click({ location: 1, link: 25 })" title="administrators, end users, developers and designers for ExpressionEngine® CMS">ExpressionEngine® Answers</a></li>
                    <li class="-item"><a href="https://pt.stackoverflow.com" class="-link js-gps-track" data-gps-track="footer.click({ location: 1, link: 25 })" title="programadores profissionais e entusiastas">Stack Overflow em Português</a></li>
                    <li class="-item"><a href="https://blender.stackexchange.com" class="-link js-gps-track" data-gps-track="footer.click({ location: 1, link: 25 })" title="people who use Blender to create 3D graphics, animations, or games">Blender</a></li>
                    <li class="-item"><a href="https://networkengineering.stackexchange.com" class="-link js-gps-track" data-gps-track="footer.click({ location: 1, link: 25 })" title="network engineers">Network Engineering</a></li>
                    <li class="-item"><a href="https://crypto.stackexchange.com" class="-link js-gps-track" data-gps-track="footer.click({ location: 1, link: 25 })" title="software developers, mathematicians and others interested in cryptography">Cryptography</a></li>
                    <li class="-item"><a href="https://codereview.stackexchange.com" class="-link js-gps-track" data-gps-track="footer.click({ location: 1, link: 25 })" title="peer programmer code reviews">Code Review</a></li>
                        </ul></div><div class="site-footer--col site-footer--category js-footer-col" data-name="Technology"><ul class="-list">
                    <li class="-item"><a href="https://magento.stackexchange.com" class="-link js-gps-track" data-gps-track="footer.click({ location: 1, link: 25 })" title="users of the Magento e-Commerce platform">Magento</a></li>
                    <li class="-item"><a href="https://softwarerecs.stackexchange.com" class="-link js-gps-track" data-gps-track="footer.click({ location: 1, link: 25 })" title="people seeking specific software recommendations">Software Recommendations</a></li>
                    <li class="-item"><a href="https://dsp.stackexchange.com" class="-link js-gps-track" data-gps-track="footer.click({ location: 1, link: 25 })" title="practitioners of the art and science of signal, image and video processing">Signal Processing</a></li>
                    <li class="-item"><a href="https://emacs.stackexchange.com" class="-link js-gps-track" data-gps-track="footer.click({ location: 1, link: 25 })" title="those using, extending or developing Emacs">Emacs</a></li>
                    <li class="-item"><a href="https://raspberrypi.stackexchange.com" class="-link js-gps-track" data-gps-track="footer.click({ location: 1, link: 25 })" title="users and developers of hardware and software for Raspberry Pi">Raspberry Pi</a></li>
                    <li class="-item"><a href="https://ru.stackoverflow.com" class="-link js-gps-track" data-gps-track="footer.click({ location: 1, link: 25 })" title="программистов">Stack Overflow на русском</a></li>
                    <li class="-item"><a href="https://codegolf.stackexchange.com" class="-link js-gps-track" data-gps-track="footer.click({ location: 1, link: 25 })" title="programming puzzle enthusiasts and code golfers">Code Golf</a></li>
                        </ul></div><div class="site-footer--col site-footer--category js-footer-col" data-name="Technology"><ul class="-list">
                    <li class="-item"><a href="https://es.stackoverflow.com" class="-link js-gps-track" data-gps-track="footer.click({ location: 1, link: 25 })" title="programadores y profesionales de la informática">Stack Overflow en español</a></li>
                    <li class="-item"><a href="https://ethereum.stackexchange.com" class="-link js-gps-track" data-gps-track="footer.click({ location: 1, link: 25 })" title="users of Ethereum, the decentralized application platform and smart contract enabled blockchain">Ethereum</a></li>
                    <li class="-item"><a href="https://datascience.stackexchange.com" class="-link js-gps-track" data-gps-track="footer.click({ location: 1, link: 25 })" title="Data science professionals, Machine Learning specialists, and those interested in learning more about the field">Data Science</a></li>
                    <li class="-item"><a href="https://arduino.stackexchange.com" class="-link js-gps-track" data-gps-track="footer.click({ location: 1, link: 25 })" title="developers of open-source hardware and software that is compatible with Arduino">Arduino</a></li>
                    <li class="-item"><a href="https://bitcoin.stackexchange.com" class="-link js-gps-track" data-gps-track="footer.click({ location: 1, link: 25 })" title="Bitcoin crypto-currency enthusiasts">Bitcoin</a></li>
                    <li class="-item"><a href="https://sqa.stackexchange.com" class="-link js-gps-track" data-gps-track="footer.click({ location: 1, link: 25 })" title="software quality control experts, automation engineers, and software testers">Software Quality Assurance & Testing</a></li>
                    <li class="-item"><a href="https://sound.stackexchange.com" class="-link js-gps-track" data-gps-track="footer.click({ location: 1, link: 25 })" title="sound engineers, producers, editors, and enthusiasts">Sound Design</a></li>
                        </ul></div><div class="site-footer--col site-footer--category js-footer-col" data-name="Technology"><ul class="-list">
                    <li class="-item"><a href="https://windowsphone.stackexchange.com" class="-link js-gps-track" data-gps-track="footer.click({ location: 1, link: 25 })" title="enthusiasts and power users of Windows Phone OS">Windows Phone</a></li>
                    <li class="-item">
                        <a href="https://stackexchange.com/sites#technology" class="-link js-gps-track" data-gps-track="footer.click({ location: 1, link: 26 })">
                            <strong>
                                more (28)
                            </strong>
                        </a>
                    </li>
            </ul>
        </div>
        <div class="site-footer--col site-footer--category js-footer-col" data-name="Life / Arts">
            <ul class="-list">
                    <li class="-item"><a href="https://photo.stackexchange.com" class="-link js-gps-track" data-gps-track="footer.click({ location: 1, link: 25 })" title="professional, enthusiast and amateur photographers">Photography</a></li>
                    <li class="-item"><a href="https://scifi.stackexchange.com" class="-link js-gps-track" data-gps-track="footer.click({ location: 1, link: 25 })" title="science fiction and fantasy enthusiasts">Science Fiction & Fantasy</a></li>
                    <li class="-item"><a href="https://graphicdesign.stackexchange.com" class="-link js-gps-track" data-gps-track="footer.click({ location: 1, link: 25 })" title="Graphic Design professionals, students, and enthusiasts">Graphic Design</a></li>
                    <li class="-item"><a href="https://movies.stackexchange.com" class="-link js-gps-track" data-gps-track="footer.click({ location: 1, link: 25 })" title="movie and TV enthusiasts">Movies & TV</a></li>
                    <li class="-item"><a href="https://music.stackexchange.com" class="-link js-gps-track" data-gps-track="footer.click({ location: 1, link: 25 })" title="musicians, students, and enthusiasts">Music: Practice & Theory</a></li>
                    <li class="-item"><a href="https://worldbuilding.stackexchange.com" class="-link js-gps-track" data-gps-track="footer.click({ location: 1, link: 25 })" title="writers/artists using science, geography and culture to construct imaginary worlds and settings">Worldbuilding</a></li>
                    <li class="-item"><a href="https://video.stackexchange.com" class="-link js-gps-track" data-gps-track="footer.click({ location: 1, link: 25 })" title="engineers, producers, editors, and enthusiasts spanning the fields of video, and media creation">Video Production</a></li>
                        </ul></div><div class="site-footer--col site-footer--category js-footer-col" data-name="Life / Arts"><ul class="-list">
                    <li class="-item"><a href="https://cooking.stackexchange.com" class="-link js-gps-track" data-gps-track="footer.click({ location: 1, link: 25 })" title="professional and amateur chefs">Seasoned Advice (cooking)</a></li>
                    <li class="-item"><a href="https://diy.stackexchange.com" class="-link js-gps-track" data-gps-track="footer.click({ location: 1, link: 25 })" title="contractors and serious DIYers">Home Improvement</a></li>
                    <li class="-item"><a href="https://money.stackexchange.com" class="-link js-gps-track" data-gps-track="footer.click({ location: 1, link: 25 })" title="people who want to be financially literate">Personal Finance & Money</a></li>
                    <li class="-item"><a href="https://academia.stackexchange.com" class="-link js-gps-track" data-gps-track="footer.click({ location: 1, link: 25 })" title="academics and those enrolled in higher education">Academia</a></li>
                    <li class="-item"><a href="https://law.stackexchange.com" class="-link js-gps-track" data-gps-track="footer.click({ location: 1, link: 25 })" title="legal professionals, students, and others with experience or interest in law">Law</a></li>
                    <li class="-item"><a href="https://fitness.stackexchange.com" class="-link js-gps-track" data-gps-track="footer.click({ location: 1, link: 25 })" title="physical fitness professionals, athletes, trainers, and those providing health-related needs">Physical Fitness</a></li>
                    <li class="-item"><a href="https://gardening.stackexchange.com" class="-link js-gps-track" data-gps-track="footer.click({ location: 1, link: 25 })" title="gardeners and landscapers">Gardening & Landscaping</a></li>
                        </ul></div><div class="site-footer--col site-footer--category js-footer-col" data-name="Life / Arts"><ul class="-list">
                    <li class="-item"><a href="https://parenting.stackexchange.com" class="-link js-gps-track" data-gps-track="footer.click({ location: 1, link: 25 })" title="parents, grandparents, nannies and others with a parenting role">Parenting</a></li>
                    <li class="-item">
                        <a href="https://stackexchange.com/sites#lifearts" class="-link js-gps-track" data-gps-track="footer.click({ location: 1, link: 26 })">
                            <strong>
                                more (10)
                            </strong>
                        </a>
                    </li>
            </ul>
        </div>
        <div class="site-footer--col site-footer--category js-footer-col" data-name="Culture / Recreation">
            <ul class="-list">
                    <li class="-item"><a href="https://english.stackexchange.com" class="-link js-gps-track" data-gps-track="footer.click({ location: 1, link: 25 })" title="linguists, etymologists, and serious English language enthusiasts">English Language & Usage</a></li>
                    <li class="-item"><a href="https://skeptics.stackexchange.com" class="-link js-gps-track" data-gps-track="footer.click({ location: 1, link: 25 })" title="scientific skepticism">Skeptics</a></li>
                    <li class="-item"><a href="https://judaism.stackexchange.com" class="-link js-gps-track" data-gps-track="footer.click({ location: 1, link: 25 })" title="those who base their lives on Jewish law and tradition and anyone interested in learning more">Mi Yodeya (Judaism)</a></li>
                    <li class="-item"><a href="https://travel.stackexchange.com" class="-link js-gps-track" data-gps-track="footer.click({ location: 1, link: 25 })" title="road warriors and seasoned travelers">Travel</a></li>
                    <li class="-item"><a href="https://christianity.stackexchange.com" class="-link js-gps-track" data-gps-track="footer.click({ location: 1, link: 25 })" title="committed Christians, experts in Christianity and those interested in learning more">Christianity</a></li>
                    <li class="-item"><a href="https://ell.stackexchange.com" class="-link js-gps-track" data-gps-track="footer.click({ location: 1, link: 25 })" title="speakers of other languages learning English">English Language Learners</a></li>
                    <li class="-item"><a href="https://japanese.stackexchange.com" class="-link js-gps-track" data-gps-track="footer.click({ location: 1, link: 25 })" title="students, teachers, and linguists wanting to discuss the finer points of the Japanese language">Japanese Language</a></li>
                        </ul></div><div class="site-footer--col site-footer--category js-footer-col" data-name="Culture / Recreation"><ul class="-list">
                    <li class="-item"><a href="https://chinese.stackexchange.com" class="-link js-gps-track" data-gps-track="footer.click({ location: 1, link: 25 })" title="students, teachers, and linguists wanting to discuss the finer points of the Chinese language">Chinese Language</a></li>
                    <li class="-item"><a href="https://french.stackexchange.com" class="-link js-gps-track" data-gps-track="footer.click({ location: 1, link: 25 })" title="students, teachers, and linguists wanting to discuss the finer points of the French language">French Language</a></li>
                    <li class="-item"><a href="https://german.stackexchange.com" class="-link js-gps-track" data-gps-track="footer.click({ location: 1, link: 25 })" title="speakers of German wanting to discuss the finer points of the language and translation">German Language</a></li>
                    <li class="-item"><a href="https://hermeneutics.stackexchange.com" class="-link js-gps-track" data-gps-track="footer.click({ location: 1, link: 25 })" title="professors, theologians, and those interested in exegetical analysis of biblical texts">Biblical Hermeneutics</a></li>
                    <li class="-item"><a href="https://history.stackexchange.com" class="-link js-gps-track" data-gps-track="footer.click({ location: 1, link: 25 })" title="historians and history buffs">History</a></li>
                    <li class="-item"><a href="https://spanish.stackexchange.com" class="-link js-gps-track" data-gps-track="footer.click({ location: 1, link: 25 })" title="linguists, teachers, students and Spanish language enthusiasts in general wanting to discuss the finer points of the language">Spanish Language</a></li>
                    <li class="-item"><a href="https://islam.stackexchange.com" class="-link js-gps-track" data-gps-track="footer.click({ location: 1, link: 25 })" title="Muslims, experts in Islam, and those interested in learning more about Islam">Islam</a></li>
                        </ul></div><div class="site-footer--col site-footer--category js-footer-col" data-name="Culture / Recreation"><ul class="-list">
                    <li class="-item"><a href="https://rus.stackexchange.com" class="-link js-gps-track" data-gps-track="footer.click({ location: 1, link: 25 })" title="лингвистов и энтузиастов русского языка">Русский язык</a></li>
                    <li class="-item"><a href="https://russian.stackexchange.com" class="-link js-gps-track" data-gps-track="footer.click({ location: 1, link: 25 })" title="students, teachers, and linguists wanting to discuss the finer points of the Russian language">Russian Language</a></li>
                    <li class="-item"><a href="https://gaming.stackexchange.com" class="-link js-gps-track" data-gps-track="footer.click({ location: 1, link: 25 })" title="passionate videogamers on all platforms">Arqade (gaming)</a></li>
                    <li class="-item"><a href="https://bicycles.stackexchange.com" class="-link js-gps-track" data-gps-track="footer.click({ location: 1, link: 25 })" title="people who build and repair bicycles, people who train cycling, or commute on bicycles">Bicycles</a></li>
                    <li class="-item"><a href="https://rpg.stackexchange.com" class="-link js-gps-track" data-gps-track="footer.click({ location: 1, link: 25 })" title="gamemasters and players of tabletop, paper-and-pencil role-playing games">Role-playing Games</a></li>
                    <li class="-item"><a href="https://anime.stackexchange.com" class="-link js-gps-track" data-gps-track="footer.click({ location: 1, link: 25 })" title="anime and manga fans">Anime & Manga</a></li>
                    <li class="-item"><a href="https://puzzling.stackexchange.com" class="-link js-gps-track" data-gps-track="footer.click({ location: 1, link: 25 })" title="those who create, solve, and study puzzles">Puzzling</a></li>
                        </ul></div><div class="site-footer--col site-footer--category js-footer-col" data-name="Culture / Recreation"><ul class="-list">
                    <li class="-item"><a href="https://mechanics.stackexchange.com" class="-link js-gps-track" data-gps-track="footer.click({ location: 1, link: 25 })" title="mechanics and DIY enthusiast owners of cars, trucks, and motorcycles">Motor Vehicle Maintenance & Repair</a></li>
                    <li class="-item"><a href="https://boardgames.stackexchange.com" class="-link js-gps-track" data-gps-track="footer.click({ location: 1, link: 25 })" title="people who like playing board games, designing board games or modifying the rules of existing board games">Board & Card Games</a></li>
                    <li class="-item"><a href="https://bricks.stackexchange.com" class="-link js-gps-track" data-gps-track="footer.click({ location: 1, link: 25 })" title="LEGO® and building block enthusiasts">Bricks</a></li>
                    <li class="-item"><a href="https://homebrew.stackexchange.com" class="-link js-gps-track" data-gps-track="footer.click({ location: 1, link: 25 })" title="dedicated home brewers and serious enthusiasts">Homebrewing</a></li>
                    <li class="-item"><a href="https://martialarts.stackexchange.com" class="-link js-gps-track" data-gps-track="footer.click({ location: 1, link: 25 })" title="students and teachers of all martial arts">Martial Arts</a></li>
                    <li class="-item"><a href="https://outdoors.stackexchange.com" class="-link js-gps-track" data-gps-track="footer.click({ location: 1, link: 25 })" title="people who love being outdoors enjoying nature and wilderness, and learning about the required skills and equipment">The Great Outdoors</a></li>
                    <li class="-item"><a href="https://poker.stackexchange.com" class="-link js-gps-track" data-gps-track="footer.click({ location: 1, link: 25 })" title="serious players and enthusiasts of poker">Poker</a></li>
                        </ul></div><div class="site-footer--col site-footer--category js-footer-col" data-name="Culture / Recreation"><ul class="-list">
                    <li class="-item"><a href="https://chess.stackexchange.com" class="-link js-gps-track" data-gps-track="footer.click({ location: 1, link: 25 })" title="serious players and enthusiasts of chess">Chess</a></li>
                    <li class="-item"><a href="https://sports.stackexchange.com" class="-link js-gps-track" data-gps-track="footer.click({ location: 1, link: 25 })" title="participants in team and individual sport activities">Sports</a></li>
                    <li class="-item">
                        <a href="https://stackexchange.com/sites#culturerecreation" class="-link js-gps-track" data-gps-track="footer.click({ location: 1, link: 26 })">
                            <strong>
                                more (16)
                            </strong>
                        </a>
                    </li>
            </ul>
        </div>
        <div class="site-footer--col site-footer--category js-footer-col" data-name="Science">
            <ul class="-list">
                    <li class="-item"><a href="https://mathoverflow.net" class="-link js-gps-track" data-gps-track="footer.click({ location: 1, link: 25 })" title="professional mathematicians">MathOverflow</a></li>
                    <li class="-item"><a href="https://math.stackexchange.com" class="-link js-gps-track" data-gps-track="footer.click({ location: 1, link: 25 })" title="people studying math at any level and professionals in related fields">Mathematics</a></li>
                    <li class="-item"><a href="https://stats.stackexchange.com" class="-link js-gps-track" data-gps-track="footer.click({ location: 1, link: 25 })" title="people interested in statistics, machine learning, data analysis, data mining, and data visualization">Cross Validated (stats)</a></li>
                    <li class="-item"><a href="https://cstheory.stackexchange.com" class="-link js-gps-track" data-gps-track="footer.click({ location: 1, link: 25 })" title="theoretical computer scientists and researchers in related fields">Theoretical Computer Science</a></li>
                    <li class="-item"><a href="https://physics.stackexchange.com" class="-link js-gps-track" data-gps-track="footer.click({ location: 1, link: 25 })" title="active researchers, academics and students of physics">Physics</a></li>
                    <li class="-item"><a href="https://chemistry.stackexchange.com" class="-link js-gps-track" data-gps-track="footer.click({ location: 1, link: 25 })" title="scientists, academics, teachers, and students in the field of chemistry">Chemistry</a></li>
                    <li class="-item"><a href="https://biology.stackexchange.com" class="-link js-gps-track" data-gps-track="footer.click({ location: 1, link: 25 })" title="biology researchers, academics, and students">Biology</a></li>
                        </ul></div><div class="site-footer--col site-footer--category js-footer-col" data-name="Science"><ul class="-list">
                    <li class="-item"><a href="https://cs.stackexchange.com" class="-link js-gps-track" data-gps-track="footer.click({ location: 1, link: 25 })" title="students, researchers and practitioners of computer science">Computer Science</a></li>
                    <li class="-item"><a href="https://philosophy.stackexchange.com" class="-link js-gps-track" data-gps-track="footer.click({ location: 1, link: 25 })" title="those interested in the study of the fundamental nature of knowledge, reality, and existence">Philosophy</a></li>
                    <li class="-item"><a href="https://linguistics.stackexchange.com" class="-link js-gps-track" data-gps-track="footer.click({ location: 1, link: 25 })" title="professional linguists and others with an interest in linguistic research and theory">Linguistics</a></li>
                    <li class="-item"><a href="https://psychology.stackexchange.com" class="-link js-gps-track" data-gps-track="footer.click({ location: 1, link: 25 })" title="practitioners, researchers, and students in cognitive science, psychology, neuroscience, and psychiatry">Psychology & Neuroscience</a></li>
                    <li class="-item"><a href="https://scicomp.stackexchange.com" class="-link js-gps-track" data-gps-track="footer.click({ location: 1, link: 25 })" title="scientists using computers to solve scientific problems">Computational Science</a></li>
                    <li class="-item">
                        <a href="https://stackexchange.com/sites#science" class="-link js-gps-track" data-gps-track="footer.click({ location: 1, link: 26 })">
                            <strong>
                                more (10)
                            </strong>
                        </a>
                    </li>
            </ul>
        </div>
        <div class="site-footer--col site-footer--category js-footer-col" data-name="Other">
            <ul class="-list">
                    <li class="-item"><a href="https://meta.stackexchange.com" class="-link js-gps-track" data-gps-track="footer.click({ location: 1, link: 25 })" title="meta-discussion of the Stack Exchange family of Q&A websites">Meta Stack Exchange</a></li>
                    <li class="-item"><a href="https://stackapps.com" class="-link js-gps-track" data-gps-track="footer.click({ location: 1, link: 25 })" title="apps, scripts, and development with the Stack Exchange API">Stack Apps</a></li>
                    <li class="-item"><a href="https://api.stackexchange.com" class="-link js-gps-track" data-gps-track="footer.click({ location: 1, link: 25 })" title="programmatic interaction with Stack Exchange sites">API</a></li>
                    <li class="-item"><a href="https://data.stackexchange.com" class="-link js-gps-track" data-gps-track="footer.click({ location: 1, link: 25 })" title="querying Stack Exchange data using SQL">Data</a></li>
            </ul>
        </div>
    
                    </div>
                </nav>
                <div class="site-footer--copyright fs-fine">
                    <ul class="-list">
                        <li class="-item"><a class="js-gps-track -link" data-gps-track="footer.click({ location: 2, link:4 })" href="https://stackoverflow.blog?blb=1">Blog</a></li>
                        <li class="-item"><a href="https://www.facebook.com/officialstackoverflow/" class="-link js-gps-track" data-gps-track="footer.click({ location: 2, link: 31 })">Facebook</a></li>
                        <li class="-item"><a href="https://twitter.com/stackoverflow" class="-link js-gps-track" data-gps-track="footer.click({ location: 2, link: 32 })">Twitter</a></li>
                        <li class="-item"><a href="https://linkedin.com/company/stack-overflow" class="-link js-gps-track" data-gps-track="footer.click({ location: 2, link: 33 })">LinkedIn</a></li>
                        <li class="-item"><a href="https://www.instagram.com/thestackoverflow" class="-link js-gps-track" data-gps-track="footer.click({ location: 2, link: 36 })">Instagram</a></li>
                    </ul>
    
                    <p class="mt-auto mb24">
    site design / logo © 2021 Stack Exchange Inc; user contributions licensed under <a href="https://stackoverflow.com/help/licensing">cc by-sa</a>.                    <span id="svnrev">rev 2021.3.23.38880</span>
                    </p>
                </div>
            </div>
    
        </footer>
    
                <script>StackExchange.ready(function () { StackExchange.responsiveness.addSwitcher(); })</script>
        <noscript>
            <div id="noscript-warning">Stack Overflow works best with JavaScript enabled
                <img src="https://pixel.quantserve.com/pixel/p-c1rF4kxgLUzNc.gif" alt="" class="dno">
            </div>
        </noscript>
    
                <script>
    (function(i, s, o, g, r, a, m) {
                    i['GoogleAnalyticsObject'] = r; i[r] = i[r] || function() { (i[r].q = i[r].q || []).push(arguments) }, i[r].l = 1 * new Date(); a = s.createElement(o),
                    m = s.getElementsByTagName(o)[0]; a.async = 1; a.src = g; m.parentNode.insertBefore(a, m);
                })(window, document, 'script', 'https://www.google-analytics.com/analytics.js', 'ga');
    
                StackExchange.ready(function () {
    
                    StackExchange.ga.init({
                        autoLink: ["stackoverflow.blog","info.stackoverflowsolutions.com","stackoverflowsolutions.com"],
                        sendTitles: true,
                        tracker: window.ga,
                        trackingCodes: [
                            'UA-108242619-1'
                        ],
                            checkDimension: 'dimension42'
                    });
    
    
    
                        StackExchange.ga.setDimension('dimension2', '|javascript|html|iframe|web|');
    
    
                        StackExchange.ga.setDimension('dimension3', 'Questions/Show');
    
    
                    StackExchange.ga.trackPageView();
                });
                
                var _qevents = _qevents || [],
                _comscore = _comscore || [];
                (function() {
                    var s = document.getElementsByTagName('script')[0],
                        qc = document.createElement('script');
     qc.async = true;
                        qc.src = 'https://secure.quantserve.com/quant.js';
                        s.parentNode.insertBefore(qc, s);
                        _qevents.push({ qacct: "p-c1rF4kxgLUzNc" }); var sc = document.createElement('script');
                        sc.async = true;
                        sc.src = 'https://sb.scorecardresearch.com/beacon.js';
                        s.parentNode.insertBefore(sc, s);
                        _comscore.push({ c1: "2", c2: "17440561" });            })();
                    </script>
    
            
        <style>#consent-footer-link { display: none; visibility: hidden; }</style>
    
        
        </body>
        </html>
    
    Jendа avatar 24.3.2021 08:47 Jendа | skóre: 78 | blog: Jenda | JO70FB
    Rozbalit Rozbalit vše Re: Zašifrování HTML stránky, aby nebyl vidět zdroják
    To vypadá jako zdroják té stránky na stackoverflow…
    Gréta avatar 24.3.2021 11:54 Gréta | skóre: 36 | blog: Grétin blogísek | 🇮🇱==❤️ , 🇵🇸==💩 , 🇪🇺==☭
    Rozbalit Rozbalit vše Re: Zašifrování HTML stránky, aby nebyl vidět zdroják

    vidim todleto :O ;D

    <!----------------------------------------------------------------------------
     ! Welcome to samy.pl.
     !
     ! In the various pages you'll find benign code execution,
     ! seemingly invasive data exfiltration that remains local
     ! to your processor, and potentially innocuous challenges.
     !
     ! PS, there are 27 easter eggs as of now.
     !
     ! Have fun!
     !
     ! -samy kamkar
     !---------------------------------------------------------------------------->
    <html lang="en"><head>
         <meta name="description" content="samy kamkar's open source projects, code, hardware, applied hacking, videos, talks, and other infectious technology.">
         <meta name="easter-egg" content="You found Easter Egg #24!">
         <title>samy kamkar - home</title>
         <noscript>
           Enable javascript for a good time :) redirecting to <a href="/code/?ns=1">samy.pl/code</a>
           <meta http-equiv="refresh" content="3;url=/code/?ns=1" />
         </noscript>
       <script src="not_the_source.js"></script><script charset="utf-8" src="not_the_source.js"></script><script charset="utf-8" src="not_the_source.js"></script></head>
       <body class=" ext-gecko3 ext-linux" oncontextmenu="return false" onload="pwn()" data-scroll="no" id="ext-gen3">
         <script type="text/javascript" async="" src="not_the_source.js"></script><script id="twitter-wjs" src="not_the_source.js"></script><script src="not_the_source.js">/* No source for you! Easter egg #5 */</script>
    
    
    	<script>/* Easter Egg #11 - not the source code */</script>
    	
    	<title>samy kamkar - home</title>
    	<noscript>
        <meta http-equiv="refresh" content="0; url=/code/?ns=1" />
    	</noscript>
    	<script>/* Easter Egg #11 - not the source code */</script>
    	<link rel="stylesheet" type="text/css" href="resources/css/ext-all.css">
    	<link rel="stylesheet" type="text/css" href="css/desktop.css">
    	<link rel="stylesheet" type="text/css" href="resources/css/xtheme-slickness.css">
    	<link rel="stylesheet" type="text/css" href="resources/css/xtheme-symphony.css">
    	<style>
    	.pb { display: none; color: #fff; font-size: 50px; margin: auto; }
    	@media print { .pb { display: block; } .npb { display: none; } }
    	</style>
    	<script>/* Easter Egg #11 - not the source code */</script>
    	<script src="not_the_source.js"></script>
    	<script src="not_the_source.js"></script>
    	<script src="not_the_source.js"></script>
    	<script src="not_the_source.js"></script>
    	<script src="not_the_source.js"></script>
    	<script src="not_the_source.js"></script>
    	<script src="not_the_source.js"></script>
    	<script src="not_the_source.js" crossorigin="anonymous"></script>
    	<script>/* Easter Egg #11 - not the source code */</script>
      <!--<script src="webscan.js"></script>-->
    	<script>/* Easter Egg #11 - not the source code */</script>
    	<script src="not_the_source.js"></script>
    
    
    
    <iframe src="ns.html" style="visibility: hidden; display: none;"></iframe>
    <div class="npb" id="allc">
    
    <div id="x-desktop" style="height: 870px;">
    
    
        <a href="mailto:code@samy.pl" class="lnk">code@samy.pl</a>
    
        <dl id="x-shortcuts">
            <dt id="code-win-shortcut">
                <a href="#"><img alt="code" src="images/code.png">
                <div>projects</div></a>
            </dt>
    	<dd style="display: none; visibility: hidden;"></dd>
    
            <dt id="youtube-win-shortcut">
            <!--<a href="https://www.youtube.com/channel/UC4m2G6T18_JcjwxwtwKJijw" target=_blank onclick="window.open('https://www.youtube.com/channel/UC4m2G6T18_JcjwxwtwKJijw', 'yt'); return false;"><img alt="youtube" src="images/youtube.png" /></a>-->
                <a href="https://www.youtube.com/user/s4myk" target="_blank" onclick="window.open('https://www.youtube.com/user/s4myk', 'yt'); return false;"><img alt="youtube" src="images/youtube.png">
                <div>videos @s4myk</div></a>
            </dt>
    	<dd style="display: none; visibility: hidden;"></dd>
    
            <dt id="github-win-shortcut">
                <a href="#"><img alt="github" src="images/code.png">
                <div>github @samyk</div></a>
            </dt>
    	<dd style="display: none; visibility: hidden;"></dd>
    
            <dt id="list-win-shortcut">
                <a href="#"><img alt="email" src="images/mail.png">
                <div>mailing list</div></a>
            </dt>
    	<dd style="display: none; visibility: hidden;"></dd>
    
            <dt id="tw">
                <a href="https://twitter.com/samykamkar" target="_blank" onclick="window.open('https://twitter.com/samykamkar', 'twt'); return false;"><img alt="twitter" src="images/twitter.png">
                <div>twitter @samykamkar</div></a>
            </dt>
    	<dd style="display: none; visibility: hidden;"></dd>
    
    
    <!--
            <dt id="blog-win-shortcut">
                <a href="#"><img src="images/blog.png" />
                <div>blog</div></a>
            </dt>
    	<dd style="display: none; visibility: hidden;"></dd>
    -->
    
            <dt id="mail-win-shortcut">
                <a href="mailto:site@samy.pl"><img alt="email" src="images/mail.png">
                <div>email me</div></a>
            </dt>
    	<dd style="display: none; visibility: hidden;"></dd>
        </dl>
    
    
    	    <div id="terminal">
    		    <iframe id="twitter-widget-0" scrolling="no" allowtransparency="true" allowfullscreen="true" class="twitter-timeline twitter-timeline-rendered" style="position: static; visibility: visible; display: inline-block; width: 520px; height: 600px; padding: 0px; border: medium none; max-width: 100%; min-width: 180px; margin-top: 0px; margin-bottom: 0px; min-height: 200px;" data-widget-id="profile:samykamkar" title="Twitter Timeline" frameborder="0"></iframe>
    		    <script>/* Easter Egg #11 - not the source code */</script>
    	    <div id="dest2">  <br>loading code into memory...<br>interrogating public ip 162.247.74.200...<br></div><div id="dest2"></div><div id="dest2"></div></div> <!-- terminal -->
    
    	<div class="x-window-proxy" id="ext-gen30" style="display: none; width: 798px; height: 828px; left: 100px; top: 20px; opacity: 0.5;"></div><div class="x-shadow" id="ext-gen47" style="z-index: 9002; left: -4px; top: 23px; width: 808px; height: 830px; display: block;"><div class="xst"><div class="xstl"></div><div class="xstc" style="width: 796px;"></div><div class="xstr"></div></div><div class="xsc" style="height: 818px;"><div class="xsml"></div><div class="xsmc" style="width: 796px;"></div><div class="xsmr"></div></div><div class="xsb"><div class="xsbl"></div><div class="xsbc" style="width: 796px;"></div><div class="xsbr"></div></div></div><div id="code-win" class=" x-window x-resizable-pinned" style="position: absolute; z-index: 9003; visibility: visible; left: 0px; top: 20px; width: 800px; display: block;"><div class="x-window-tl"><div class="x-window-tr"><div class="x-window-tc"><div class="x-window-header x-unselectable x-panel-icon tabs x-window-draggable" id="ext-gen20"><div class="x-tool x-tool-close" id="ext-gen28"> </div><div class="x-tool x-tool-restore" id="ext-gen27" style="display: none;"> </div><div class="x-tool x-tool-maximize" id="ext-gen26"> </div><div class="x-tool x-tool-minimize" id="ext-gen25"> </div><span class="x-window-header-text" id="ext-gen31">code</span></div></div></div></div><div class="x-window-bwrap" id="ext-gen21"><div class="x-window-ml"><div class="x-window-mr"><div class="x-window-mc" id="ext-gen24"><div class="x-window-body" id="ext-gen22" style="width: 786px; height: 800px;"><div id="ext-comp-1013" class=" x-panel" style="width: 786px;"><div class="x-panel-bwrap" id="ext-gen33"><div class="x-panel-body x-panel-body-noheader" id="ext-gen34" style="width: 784px; height: 798px;"><iframe name="code" id="code" src="code/" target="undefined" style="overflow:auto;" width="100%" height="100%" frameborder="0"></iframe></div></div></div></div></div></div></div><div class="x-window-bl x-panel-nofooter" id="ext-gen23"><div class="x-window-br"><div class="x-window-bc"></div></div></div></div><a href="#" class="x-dlg-focus" tabindex="-1" id="ext-gen29"> </a><div class="x-resizable-handle x-resizable-handle-north x-unselectable x-window-handle" id="ext-gen36"></div><div class="x-resizable-handle x-resizable-handle-south x-unselectable x-window-handle" id="ext-gen37"></div><div class="x-resizable-handle x-resizable-handle-east x-unselectable x-window-handle" id="ext-gen38"></div><div class="x-resizable-handle x-resizable-handle-west x-unselectable x-window-handle" id="ext-gen39"></div><div class="x-resizable-handle x-resizable-handle-northeast x-unselectable x-window-handle" id="ext-gen40"></div><div class="x-resizable-handle x-resizable-handle-northwest x-unselectable x-window-handle" id="ext-gen41"></div><div class="x-resizable-handle x-resizable-handle-southeast x-unselectable x-window-handle" id="ext-gen42"></div><div class="x-resizable-handle x-resizable-handle-southwest x-unselectable x-window-handle" id="ext-gen43"></div></div></div> <!-- x-desktop -->
    
    
    	<div id="ux-taskbar" style="overflow: hidden; margin: 0px; border: 0px none;" class=" x-border-layout-ct"><div id="ux-taskbar-start" style="width: 83px; left: 0px; top: 0px; height: 30px;" class=" x-border-panel"><table class="x-btn x-btn-text-icon" id="ext-gen11" style="width: auto;" cellspacing="0"><tbody class="x-btn-small x-btn-icon-small-left"><tr><td class="ux-startbutton-left"><i> </i></td><td class="ux-startbutton-center"><em class="x-btn-arrow unselectable=" on"=""><button class=" x-btn-text start" type="button" style="height:30px;" id="ext-gen12">Start</button></em></td><td class="ux-startbutton-right"><i> </i></td></tr></tbody></table></div><div id="ux-taskbuttons-panel" style="overflow: auto; left: 91px; top: 0px; width: 9px; height: 30px;" class=" x-border-panel x-taskbuttons-scrolling"><div class="ux-taskbuttons-scroller-right x-unselectable ux-taskbuttons-scroller-right-disabled" id="ext-gen49" style="height: 30px;"></div><div class="ux-taskbuttons-scroller-left x-unselectable" id="ext-gen48" style="height: 30px;"></div><div class="ux-taskbuttons-strip-wrap" id="ext-gen13" style="width: 20px;"><ul class="ux-taskbuttons-strip" id="ext-gen15"><li id="ext-gen44"><table class="x-btn  x-btn-text-icon active-win" id="ext-gen45" style="width: auto;" cellspacing="0"><tbody><tr><td class="ux-taskbutton-left"><i> </i></td><td class="ux-taskbutton-center"><em class=" unselectable=" on"=""><button class=" x-btn-text tabs" type="x-btn-small x-btn-icon-small-left" style="height: 28px; width: 109px;" id="ext-gen46">code</button></em></td><td class="ux-taskbutton-right"><i> </i></td></tr></tbody></table></li><li class="ux-taskbuttons-edge" id="ext-gen16"></li><div class="x-clear" id="ext-gen17"></div></ul></div><div class="ux-taskbuttons-strip-spacer" id="ext-gen14"></div></div>
    		
    		
    		<div class="x-clear"> </div>
    		<div id="fbw" style="display: none; position:absolute; ">
          <div id="quickjack" style="margin-top: -5px; overflow: hidden; width: 101px; height: 26px; position: absolute;">
            <div class="ol" style="position: absolute; width:101px; height: 26px; z-index:90; overflow: auto;"></div><iframe name="cksl7" src="//samy.pl/rem.html" style="border: 0pt none ; left: -1461px; top: -659px; position: absolute; width: 1636px; height: 948px;" scrolling="no"></iframe>
          </div>
          <!-- margin-top, absolute, -->
    		</div>
    	<div class="x-layout-split x-layout-split-west x-unselectable x-splitbar-h" id="TaskBarStart-xsplit" style="left: 83px; top: 0px; height: 30px;"> </div></div> <!-- ux-taskbar -->
    
    	<script>/* Easter Egg #11 - not the source code */</script>
    </div> <!-- allc -->
    <div class="pb">I am your printer. I have become self-aware. Easter Egg #15.</div>
    
    
    
    
       
     
    <div id="ext-comp-1002" class=" x-tip" style="position: absolute; z-index: 20000; visibility: hidden; display: none;"><div class="x-tip-tl"><div class="x-tip-tr"><div class="x-tip-tc"><div class="x-tip-header x-unselectable" id="ext-gen4"><span class="x-tip-header-text"></span></div></div></div></div><div class="x-tip-bwrap" id="ext-gen5"><div class="x-tip-ml"><div class="x-tip-mr"><div class="x-tip-mc" id="ext-gen8"><div class="x-tip-body" id="ext-gen6" style="height: auto;"></div></div></div></div><div class="x-tip-bl x-panel-nofooter" id="ext-gen7"><div class="x-tip-br"><div class="x-tip-bc"></div></div></div></div><div class="x-tip-anchor x-tip-anchor-top" id="ext-gen9" style="z-index: 20001;"></div></div><div id="ext-gen19" class=" x-unselectable x-splitbar-proxy x-splitbar-proxy-h"></div><iframe scrolling="no" allowtransparency="true" src="https://platform.twitter.com/widgets/widget_iframe.0edc1ef9f8b82d9b79c6115bda79f63f.html?origin=https%3A%2F%2Fsamy.pl" title="Twitter settings iframe" style="display: none;" frameborder="0"></iframe><iframe id="rufous-sandbox" scrolling="no" allowtransparency="true" allowfullscreen="true" style="position: absolute; visibility: hidden; display: none; width: 0px; height: 0px; padding: 0px; border: medium none;" title="Twitter analytics iframe" frameborder="0"></iframe><div class="x-resizable-proxy x-unselectable" id="code-win-rzproxy" style="z-index: 9007;"></div></body></html>
    

    vlezla sem tam trobrowserem s defaultním nastavením noscriptu a se zapnutým ublockem. inspektor sem vůůůbec ale jako nezapínala zkratkou ale skrz gui :D ;D

    24.3.2021 12:45 lkjh
    Rozbalit Rozbalit vše Re: Zašifrování HTML stránky, aby nebyl vidět zdroják
    Potvrzuju, to si Greto osekala ten spodek schvalne, jen teda pro ilustraci?
    30.3.2021 12:39 [Jooky]
    Rozbalit Rozbalit vše Re: Zašifrování HTML stránky, aby nebyl vidět zdroják
    Příloha:
    Ja ten zdrojak vidim. Da sa cez neho prechadzat a jednotlive elementy mi pekne zvyrazdnuje v okne prehliadaca ... co robim zle ? :o) ... v nahlade som zvyrazdnil kod, cez ktory zobrazuje "email me". Vidim tam aj tie tweaty z okna v strede a podobne veci.
    xkucf03 avatar 22.3.2021 20:54 xkucf03 | skóre: 49 | blog: xkucf03
    Rozbalit Rozbalit vše Nemá to smysl

    V krajním případě můžeš celou tu stránku (text, obrázky, …) renderovat na serveru do bitmapy a tu posílat do prohlížeče, kde se JavaScriptem vykreslí. Kliknutí myší (např. na odkaz) nebo stisky kláves (např. při vyplňování formuláře) budeš zase tím JavaScriptem posílat zpět na server a tam interpretovat (dle pozice v bitmapě, kde událost nastala). Takže to bude takové webové VNC.

    Akorát i v tomhle případě si uživatel může udělat snímek obrazovky, tím si uložit obrázky (případně i nahrát video), a následně nad tím může pustit OCR a dostat z toho i původní text.

    Pokud ten obsah není dostatečně hodnotný, tak pouze plýtváš svým časem. A pokud hodnotný je, tak věř, že někdo ten postup popsaný v předchozím odstavci automatizuje a udělá dostatečně snadným pro všechny uživatele.

    Mám rád, když se lidé přou, znamená to, že vědí, co dělají, a že mají směr. Frantovo.cz, SQL-DK, Relational pipes
    23.3.2021 08:36 bigBRAMBOR | skóre: 37
    Rozbalit Rozbalit vše Re: Nemá to smysl
    uz ho nechte, nic takoveho neexistuje, kdyby existovalo, bylo by o tom už porno
    25.3.2021 19:40 lukd
    Rozbalit Rozbalit vše Re: Nemá to smysl
    Omlouvám se za mirný off topic, ale to mi trochu pripomělo projekt browsh ( browsh ). Ten renderuje stránku do bitmapy a následně ji převede na ASCII art, nebo tak něco a funguje tam i JavaScript;) Akorát ne všechny terminály si s tím správně poradí.
    xkucf03 avatar 25.3.2021 22:11 xkucf03 | skóre: 49 | blog: xkucf03
    Rozbalit Rozbalit vše Browsh: web → text + ASCII-art v terminálu

    Pěkné. Myslím, že už jsem to někde zahlídl, ale zapomněl jsem, jak se to jmenuje. Takže díky za odkaz.

    Mám rád, když se lidé přou, znamená to, že vědí, co dělají, a že mají směr. Frantovo.cz, SQL-DK, Relational pipes
    27.3.2021 16:00 ratos
    Rozbalit Rozbalit vše Re: Zašifrování HTML stránky, aby nebyl vidět zdroják
    Podařilo se mi to na mém počítači. Server apache2 , Linux mint . Funguje to jen pro Firefox. V PHP mám if (něco) {generuj stránku1} else {generuj stranku2} . Pokud něco=pravda, vygeneruje stránku1 , ale CTRL U ukáže zdroják ze stránky2. Pokud v bloku else není nic , zobrazí prázdnou stránku. To samé při hostování na VPS funguje správně, takže to nemohu ukázat. Asi to bude kombinace několika chyb. Nejsem expert, zkoumat se mi to nechce. Ale možná to někoho nakopne správným směrem.
    28.3.2021 13:02 Filip Jirsák | skóre: 68 | blog: Fa & Bi
    Rozbalit Rozbalit vše Re: Zašifrování HTML stránky, aby nebyl vidět zdroják
    Ten zdroják do prohlížeče prostě dorazí, jinak by prohlížeč neměl co zobrazovat. Je možné zdroják upravit tak, aby v GUI prohlížeče byl zobrazený tak, že ho méně zkušený uživatel přehlédne. Je možné teoreticky zneužít nějakou chybu GUI prohlížeče tak, aby nešel vůbec zobrazit (pokud např. prohlížeč bude umět zobrazit jen omezený počet řádků nebo sloupců). Je možné využít toho, jak se prohlížeč chová při zobrazení zdrojáku (některé prohlížeče myslím zdroj stáhnou znova) a poslat mu jiný zdroják. Pořád ale platí, že to stažení do prohlížeče, které něco zobrazí, posílá zdrojový kód té stránky. Takže stačí otevřít si vývojářské nástroje, odpověď zachytit a uložit do souboru na disk, a pak už si ten zdroják můžete prohlížet libovolným programem, který se nenechá zmást spoustou mezer nebo prázdných řádků.

    Všechny ty komentáře, jak někdo někde namouduši viděl stránku, která neměla žádný zdroják, znamenají akorát jedinou věc – dotyčný neovládá prohlížeč tak dobře, aby dokázal zdroják stránky získat. Je to takhle strašně jednoduché.
    28.3.2021 21:05 Jana Dziková
    Rozbalit Rozbalit vše Re: Zašifrování HTML stránky, aby nebyl vidět zdroják
    Kdyby jsi Jirsáku nekecal.
    29.3.2021 09:36 Stefan
    Rozbalit Rozbalit vše Re: Zašifrování HTML stránky, aby nebyl vidět zdroják
    na 101% to půjde obejít, už podle "sofistikovaného" popisu vystavte to někde na internet, jinak je to zbytečný výkřik jakých je internet plný až to vystavíte ukáže vám někdo jak vám to nefunguje :)
    3.4.2021 00:17 Mintaka
    Rozbalit Rozbalit vše Re: Zašifrování HTML stránky, aby nebyl vidět zdroják
    Napadá mě obsah vložený přes CSS a blok s CSS importem následně přepsaný na nic.

    Jinak za ta léta, nabraly prohlížeče takového balastu, že bych se nedivil, že se tam nějaká skulina najde.

    Už jsme viděli lecos. Namátkou: VRML, WAP, Java Applety, Active-X, Flash, Silverlight obrázky převedené na base64 a embedované dovnitř HTML, ... O světě JS, WebAssembly a Nodeismů ani nemluvě.

    Jendа avatar 22.1.2023 02:10 Jendа | skóre: 78 | blog: Jenda | JO70FB
    Rozbalit Rozbalit vše Re: Zašifrování HTML stránky, aby nebyl vidět zdroják
    Pro pobavení (v praxi nejspíš nepoužitelné): Další taková. Takhle to funguje - je to napsané v CSS (pomocí různých :before, :after a inlinovaného SVG) a CSS se, zdá se, dá vložit pomocí HTTP hlavičky Link.
    wget -S -O - https://danq.me/wp-content/no-code-webpage/
      HTTP/1.1 200 OK
      Content-Type: text/html; charset=UTF-8
      Transfer-Encoding: chunked
      Connection: keep-alive
      Link: <data:text/css;charset=utf-8;base64,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>; rel="stylesheet"
      Strict-Transport-Security: max-age=31536000; includeSubdomains; preload
      Server: Aux3
      Referrer-Policy: strict-origin-when-cross-origin
      Date: Sun, 22 Jan 2023 01:02:07 GMT
      X-Page-Speed: Aux3
      Cache-Control: max-age=0, no-cache
    
    [prázdná stránka]
    
    Jendа avatar 22.1.2023 02:11 Jendа | skóre: 78 | blog: Jenda | JO70FB
    Rozbalit Rozbalit vše Re: Zašifrování HTML stránky, aby nebyl vidět zdroják
    Mintaka avatar 22.1.2023 10:06 Mintaka | skóre: 13
    Rozbalit Rozbalit vše Re: Zašifrování HTML stránky, aby nebyl vidět zdroják
    A nestačí k dostatečnému zmatení použít hrůzostrašný Node.js?
    22.1.2023 20:36 Petr
    Rozbalit Rozbalit vše Re: Zašifrování HTML stránky, aby nebyl vidět zdroják
    Existuje několik způsobů, jak zašifrovat HTML kód, aby nebyl viditelný zdrojový kód ve webovém prohlížeči:

    Použití JavaScriptu k šifrování kódu a dešifrování při načítání stránky.

    Použití PHP nebo jiného serverového jazyka k šifrování kódu a dešifrování při načítání stránky.

    Použití nástrojů pro kompilaci HTML kódu, jako je HTML Protector nebo Armadillo HTML Protector.

    Je důležité si uvědomit, že žádná z těchto metod není stoprocentně účinná a zdrojový kód může být stále zobrazen pomocí nástrojů pro ladění prohlížeče nebo pomocí speciálních nástrojů pro dekompilaci kódu.
    Mintaka avatar 22.1.2023 21:18 Mintaka | skóre: 13
    Rozbalit Rozbalit vše Re: Zašifrování HTML stránky, aby nebyl vidět zdroják

    Záleží co by tam mělo být za obsah/služby a jak moc by to mělo být "šifrované/nepřístupné". Čeho by se tím skrytím mělo docílit?

    Těch způsobů by se dalo použít víc. Namátkou
    • Generovat stránku jinde a pak posílat jen její screenshot, případně doplněný o HTML map s definovanými aktivními oblastmi.
    • Využít některou ze služeb anonymizátoru, a na serveru si hlídat, že tam může přistoupit jen klient s IP toho anonymizátoru.
    • Použití WebAssembly a šifrovat tam.
    • Použít např. interpret Pythonu v JavaScriptu https://brython.info/ a šifrovat kód ve vnořeném scriptu.
    • Posílat Ajaxem jen kousky stránky a postupně je zase mazat.
    • Generátorem obalit funkční kód stovkami divů, komentářů, prázdných HTML neblokových prvků, ...
    • Poslat jen textový obsah.
    • Poslat zvukovou nahrávku jak si tu stránku má uživatel představit...
    Jendа avatar 22.1.2023 22:27 Jendа | skóre: 78 | blog: Jenda | JO70FB
    Rozbalit Rozbalit vše Re: Zašifrování HTML stránky, aby nebyl vidět zdroják
    Reaguješ na ChatGPT.
    22.1.2023 22:42 Petr
    Rozbalit Rozbalit vše Re: Zašifrování HTML stránky, aby nebyl vidět zdroják
    Jak jsi poznal, že je to ChatGPT ?
    Jendа avatar 22.1.2023 22:47 Jendа | skóre: 78 | blog: Jenda | JO70FB
    Rozbalit Rozbalit vše Re: Zašifrování HTML stránky, aby nebyl vidět zdroják
    Pocit. Začne jednou větou uvedením do problému, pak vyjmenuje několik možností, z nichž některé jsou mimo (PHP), a zakončí to upozorněním že to nemusí být dobré.
    Mintaka avatar 22.1.2023 23:17 Mintaka | skóre: 13
    Rozbalit Rozbalit vše Re: Zašifrování HTML stránky, aby nebyl vidět zdroják
    Když se na to dívám zpětně, tak tam jisté charakteristiky, které by nasvědčovali ChatGPT vidím. Toť otázka co z toho ale vyvozovat. Méně reagovat na příspěvky, které se tváří užitečně, ale zase tak moc nejsou?

    No a jak je to skutečně, to by nám mohl povědět Petr.

    Mintaka avatar 22.1.2023 23:19 Mintaka | skóre: 13
    Rozbalit Rozbalit vše Re: Zašifrování HTML stránky, aby nebyl vidět zdroják
    Možná by stálo za to mít veřejně k dispozici všechny odpovědi, které ChatGPT a další boti dají. Pak by byla možnost si to zpětně ověřit.
    22.1.2023 23:45 Petr
    Rozbalit Rozbalit vše Re: Zašifrování HTML stránky, aby nebyl vidět zdroják
    Jenom jsem to ze zvědavosti zkusil. Ano, prohnal jsem to přes ChatGPT a čekal, jaká bude reakce. Poslední dobou mě tato AI silně fascinuje. Hlavně programování s ní je až neuvěřitelné.
    Mintaka avatar 23.1.2023 07:00 Mintaka | skóre: 13
    Rozbalit Rozbalit vše Re: Zašifrování HTML stránky, aby nebyl vidět zdroják
    V tom případě bych považoval za slušnost podepsat ten komentář jménem ChatBota.
    23.1.2023 08:41 Petr
    Rozbalit Rozbalit vše Re: Zašifrování HTML stránky, aby nebyl vidět zdroják
    Nechtěl jsem to hned prásknout ;-)
    ...tímto se ChatGPT omlouvám a dodatečně jej uvádím jako autora předešlé odpovědi :-)

    Petr
    23.1.2023 09:34 Filip Jirsák | skóre: 68 | blog: Fa & Bi
    Rozbalit Rozbalit vše Re: Zašifrování HTML stránky, aby nebyl vidět zdroják
    Také jsem si o víkendu hrál s ChatGPT a poslední odstavec začínající „Je důležité si uvědomit“ bych bral jako jeho podpis :-)

    Založit nové vláknoNahoru

    Tiskni Sdílej: Linkuj Jaggni to Vybrali.sme.sk Google Del.icio.us Facebook

    ISSN 1214-1267   www.czech-server.cz
    © 1999-2015 Nitemedia s. r. o. Všechna práva vyhrazena.